Pulmonology EXAM MASTER

Pataasin ang iyong marka sa homework at exams ngayon gamit ang Quizwiz!

Case Ico-delete Highlights A 50-year-old man presents with a 4-day history of increasing exertional dyspnea. He has had a chronic cough for the past 3 years and attributes it to cigarette smoking. The cough had been productive of watery sputum, but it has changed to a yellowish color over the past week. He has no known allergies and reports no family history of asthma; on examination, he is wheezing. His temperature is 101°F, P 105/min, BP 136/86 mm Hg, and RR 30/min. Respiratory system examination reveals decreased chest wall excursion. Auscultation reveals a prolonged expiratory phase with crepitations and generalized rhonchi. Chest X-ray reveals irregular bronchovascular markings. Laboratory results reveal Hb 15 g/dL, WBC 12000/uL, and platelets 300 x 109/L. Question What are the pathogens most commonly associated with acute exacerbation of the patient's most likely diagnosis?

Correct answer: Haemophilus influenzae, Streptococcus pneumoniae, Moraxella catarrhalis Explanation Haemophilus influenzae, Streptococcus pneumoniae, and Moraxella catarrhalis are the most predominant pathogens involved in the acute exacerbation of chronic bronchitis; they are responsible for up to 60% of the cases and are the most common pathogens to be cultured from sputum in these patients. Acute exacerbations of chronic bronchitis are marked by an increase in the volume or purulence of sputum and a worsening of cough or dyspnea. Gram stain of the sputum usually shows a mixture of organisms; they are often gram-positive diplococci, Streptococcus pneumoniae, and gram-negative rods, Haemophilus influenzae. Other organisms associated with acute exacerbations include M. pneumoniae, Legionella pneumophila, atypical bacterium (e.g., Chlamydia pneumoniae), and viruses (e.g., influenza and adenovirus). C. pneumoniae is the most common atypical bacterium implicated in exacerbations and approximates 10% of the cases. Viruses have been implicated in the remaining 25-30% of the cases. Pseudomonas aeruginosa is associated with severe exacerbations that require mechanical ventilation. The less frequent cause of exacerbation includes M. pneumoniae and L. pneumophila. Sputum production in chronic bronchitis is insidious in onset, initially occurring only in the morning. The sputum is usually mucoid, but it becomes purulent during an exacerbation. The patient may also be wheezing, dyspneic, and febrile during these exacerbations. These acute exacerbations can be triggered by exposure to tobacco smoke, air pollution, viral infections, and allergies. As chronic obstructive pulmonary disease (COPD) progresses, the intervals shorten. Patients with chronic bronchitis may also present with hemoptysis, which is usually caused by mucosal erosion. On examination, there may be expiratory wheezes and crepitations. CXR reveals irregular bronchovascular markings due to repeated inflammation and scarring. Patients with acute exacerbations of chronic bronchitis should be treated with trimethoprim/sulfamethoxazole or amoxicillin/clavulanate combinations. They should also receive an annual influenza vaccination.

Case A 54-year-old man presents with a 1-year history of pneumonia, visceral Kaposi's sarcoma that is unresponsive to therapy, and chronic persistent diarrhea. He says that he uses heroin on and off because he is tired of visiting doctors all the time. He has elected to forego antiretroviral medication, chemotherapy, and prophylactic drug therapy. His caregiver mentions that his functional status has declined; he is confined to his bed for most of the day. His mental status is still intact, but activities of daily living (ADL) are notably affected. Physical exam reveals the patient has congestive heart failure. He is very thin, with recent wasting. His lab results show a viral load of 150,000 copies/ml, BUN 19 mg/dl, serum creatinine 1.3 mg/dl, CD4+ count 50 cells/mcL, and his serum albumin is repeatedly 2 mg/dl. His viral load 2 months ago was 140,000 copies/ml. Question What is the most appropriate action in this case?

Correct answer: Hospice care with the diagnosis of end stage HIV/AIDS disease Explanation This patient has end-stage HIV/AIDS, and hospice care is appropriate for him. Patients with HIV/AIDS should meet the following criteria to be eligible for hospice services: CD4+ count < 25 cells/mcL or persistent viral load >100,000 copies/ml from 2 or more assays at least 1 month apart, and at least 1 of the following conditions: Central nervous system (CNS) lymphoma, Visceral Kaposi's sarcoma unresponsive to therapy, Progressive multifocal leukoencephalopathy (PML), Cryptosporidiosis, Mycobacterium avium complex (MAC) bacteremia, untreated, refractory, or treatment refused Untreated or refractory wasting (loss of >33% lean body mass), Renal failure in the absence of dialyses, and Refractory toxoplasmosis There should also be a palliative performance scale of < 50% (the patient requires considerable assistance and frequent medical care as activity is mostly limited to bed or chair). Other supporting documentations include: Chronic persistent diarrhea for 1 year, Persistent serum albumin < 2.5 gm/dL, Concomitant substance abuse, Age greater than 50, Congestive heart failure which is symptomatic at rest, Patient has elected to forego antiretroviral and prophylactic medication related specifically to HIV. There is no indication that the patient has end-stage heart disease, end-stage pulmonary disease, or end-stage renal disease; however, pneumonia and congestive heart failure are helpful factors for making decisions regarding patients with end-stage HIV disease.

Case Ico-delete Highlights A 22-year-old woman presents with frequent sinus infections, a persistent cough productive of discolored purulent sputum, and progressive dyspnea on exertion. She also reports recurrent episodes of pneumonia, and she has been increasingly fatigued. These symptoms have evolved over several years. As a child, she had an appendectomy and surgery for bowel obstruction. Question What is the criterion standard for diagnosing the underlying etiology of her primary disease?

Correct answer: Skin sweat test Explanation The criterion standard for diagnosing cystic fibrosis (CF) is the skin sweat test. An elevation of the sweat chloride >60 mmol/L is typical of CF, but the diagnosis requires positive results on 2 separate occasions in a patient with a CF phenotype, or who has a sibling with CF. DNA analysis is currently not used for primary diagnosis due to the large number of mutations associated with CF. With so many variations, patients are most commonly diagnosed in infancy during the first 24 hours of life or the first 1-2 years, but a significant number are diagnosed after the age of 18. In recent years, every state has begun screening newborns for cystic fibrosis. If a genetic test or blood test suggests CF, the diagnosis is confirmed using a sweat test. Pulmonary function testing in CF reveals increased residual volumes (RV) with decreased forced vital capacity (FVC) and forced expiratory volume in 1 second (FEV1). These changes are seen in obstructive disease and are not specific to CF. CF is an obstructive lung disease that is progressive over time. Changes on chest X-ray seen in CF include hyperinflation in early-stage disease and bronchiectasis in late-stage disease. Although these changes are typical, they are not diagnostic for CF. Chest radiograph would identify an LRI, which is secondary to the primary disease of CF. Sputum for culture and sensitivity would identify Pseudomonas aeruginosa, which is the typical organism seen in lower respiratory infections (LRI) associated with CF, and Burkholderia cepacia, which is pathogenic for CF when identified. These organisms identify LRI, which must be treated aggressively, but these occur secondary to the underlying etiology of CF, which is a mutation of the CF gene affecting the exchange of ions across epithelia in the airways, intestines, pancreas, and sweat ducts. Alpha-1-antitrypsin deficiency (α1AT) is a genetic risk factor for early-onset COPD, but it would not account for the gastrointestinal pathology associated with CF.

Case A 50-year-old man presents with a 2-week history of not being able to see well. He is not on any medications. He has been smoking 2 packs of cigarettes a day for the past 30 years. On examination of his right eye, there is ptosis and miosis. A chest radiograph reveals a rounded opacity in the right lung field. Question What is the most likely diagnosis? Answer Choices Ico-marker Ico-abct 1 Lambert-Eaton myasthenic syndrome Ico-marker Ico-abct 2 Hypertrophic pulmonary osteoarthropathy Ico-marker Ico-abct 3 Horner syndrome Ico-marker Ico-abct 4 Ectopic adrenocorticotropic hormone (ACTH) secretion Ico-marker Ico-abct 5 Syndrome of inappropriate antidiuretic hormone secretion (SIADH)

Correct answer: Horner syndrome Explanation Horner syndrome is due to a bronchogenic carcinoma tumor mass extending to the sympathetic chain. Horner syndrome is comprised of miosis, ptosis, enophthalmos, and hemianhidrosis. In addition, the patients may also have atrophy of hand muscles. Lambert-Eaton myasthenic syndrome is an immune-mediated disorder of neuromuscular transmission resulting in impaired release of acetylcholine from nerve terminals. Patients complain of weakness of the scapular and pelvic girdles muscles, and the resultant difficulties in performing activities such as climbing stairs. Other symptoms (e.g., dry mouth, paresthesias, and sexual impotence due to autonomic dysfunction) may also be present. On examination, there is proximal muscle weakness. Deep tendon reflexes are decreased or absent. In cases of hypertrophic pulmonary osteoarthropathy (HPO), there is formation of new subperiosteal cancellous bone at the distal ends of long bones. Patients present with periarticular pain, polyarthralgia, and painful swellings at the wrists, knees, elbows, and ankles. On examination, there is clubbing of the fingers and toes. There may also be localized articular erythema, tenderness, swelling, and even effusions. X-rays may reveal subperiosteal bone formation, with periosteal elevation seen as thickening and detachment of the periosteum.

Case A 52-year-old woman, living a non-sedentary lifestyle, presents with a 5-day history of low-grade fever, flu-like syndrome, sore throat, and malaise. She has to 'catch' her breath because of pain on inspiration and when coughing. She has no known past medical or surgical history; she is not on any medication, and she has no pertinent family history. She denies any medication use, including over-the-counter medicines. On physical examination, her vitals are: temperature 100.6° F, pulse 86/min, BP 133/75 mm Hg, and RR 20 cycles/min. She has shallow breathing, resonant percussion notes, fair air entry with vesicular breath sounds, and friction rub. Her blood gas on room air is as follows: pH 7.36 PCO2 44 mm Hg PO2 100 mm Hg HCO3 26 mEq\L O2 saturation 99.8% Her chest X-ray (CXR) is normal and D-dimer assay is also normal. Question What is the most appropriate management modality for this patient?

Correct answer: NSAIDs (non-steroidal anti-inflammatory drugs) Explanation The patient has pleuritis without effusion. Pleuritis is inflammation of the pleura; it may occur with or without effusion and is characterized by a sharp pain worsened by cough and inspiration. The pain may radiate to the shoulder if the diaphragmatic pleura is affected, and pressure around the area of inflammation may produce pain. Friction rub is heard on auscultation. The pleuritic chest pain causes shallow rapid breaths. The presentation with flu-like illness, malaise, absence of history of collagen vascular disease, and CXR findings makes the most likely etiology viral in this case. Pleuritis is treated with NSAIDs. Ibuprofen can be given in the dose of 400 - 800 mg orally 3 times daily for pain management in cases of pleurisy due to viral etiology. Other examples of etiologic agents are rheumatoid arthritis, systemic lupus erythematosus, radiation, pneumonia, acute pulmonary embolism, and pneumothorax. Although a pulmonary embolism can cause pleuritis, the D-dimer assay is normal in this patient; no further evaluation by imaging modalities, such as V/Q scanning or CT scanning, is warranted in a person at low risk of a pulmonary embolism. Common risk factors are prolonged immobilization, congestive heart failure, stroke, pelvic fracture, obesity, and malignancy. Patients present with sudden shortness of breath, tachycardia, tachypnea, hypotension (if massive), and low-grade fever. There is hypoxemia on blood gas, and CXR may be normal or show a region of oligemia and wedge-shaped density. Treatment is by anticoagulation.

Case A 43-year-old man without any significant PMHx presents with acute onset of a productive cough, shortness of breath, pleuritic chest pain, and fever. His sputum is described as "thick, brown-colored, and mucoid" but without blood. He also notes associated fatigue and night sweats. He denies chills, changes in weight, a history of travel, sick or confined contacts, exposure to animals, cigarette smoking, otalgia, sore throat, swollen glands, abdominal pain, diarrhea, rashes, myalgias, and arthralgias. His physical exam is remarkable for fever, tachycardia, and tachypnea. The lung exam is noteworthy for right lower lung field increased tactile fremitus, dullness to percussion, inspiratory crackles, and bronchial breath sounds. Question What is the most likely etiologic agent responsible for this patient's presentation?

Correct answer: Streptococcus pneumoniae Explanation This patient is demonstrating signs and symptoms representative of bacterial pneumonia. Streptococcus pneumoniae is the most common cause of community-acquired pneumonia and classically causes rust-colored sputum. Travel to the Ohio or St. Lawrence River valley or contact with bird droppings and bat areas should raise suspicion of Histoplasma capsulatum pneumonia. The presentation of acute histoplasmosis consists of marked prostration, fever, but a relative scarcity of pulmonary issues despite the potential for significant chest X-ray findings. Exposure to birds (parrots, parakeets, pigeons, chickens, ducks) 7-15 days prior to the development of rapid onset fever, chills, myalgia, dry cough, and headache is consistent with psittacosis. Gram-negative pneumonias (e.g., Pseudomonas aeruginosa) occur most often in individuals who are debilitated, immunocompromised, or recently hospitalized. Individuals living in long-term care facilities where other residents are intubated are also at risk for these infections. Pseudomonas may cause greenish sputum. Tuberculosis is a caseating granulomatous infection characterized by fever, followed by night sweats, malaise, fatigue, and weight loss. Productive cough, hemoptysis, dyspnea, and pleuritic chest pain develop as the infection spreads within the lungs. Homeless patients, immigrants, those with HIV, those who use drugs, alcoholic patients, and elderly patients are at the greatest risk.

Case A 45-year-old man presents with significant weight loss (10 pounds in 4 months), cough with hemoptysis, and pleuritic chest pain. The chest X-ray shows ill-defined opacities in both the lungs; the opacities have a reticulonodular pattern. A transbronchial biopsy is performed, and it microscopically shows a few epithelioid cells with necrotic debris. Question What is the most likely diagnosis?

Correct answer: Tuberculosis Explanation The epithelioid cells suggest that it is a granuloma; the hemoptysis suggests that the granuloma has eroded enough parenchyma and involved a bronchus. A granulomatous infection, along with hemoptysis, is typical for tuberculosis. The history of significant weight loss and hemoptysis is also typical of tuberculosis, but it can also be seen in other cases. Aspergillosis is more likely to produce a fungus ball, which is defined on an X-ray. The inflammatory reaction would vary from acute to mixed to granulomatous. Squamous cell carcinoma is usually a central mass lesion. It is unlikely to produce epithelioid cells, which are seen in the biopsy of this patient. Pneumocystis carinii pneumonia has a granulomatous pattern in rare cases, but the granulomas are small and unlikely to produce hemoptysis. Oat cell (or small cell) carcinoma is centrally located and is unlikely to be found with epithelioid cells.

Case Ico-delete Highlights A 52-year-old woman with a history of tobacco use was diagnosed with non-small cell bronchogenic carcinoma after a biopsy of a palpable supraclavicular lymph node. She was proceeding with her work up, but she now presents after a week of worsening dyspnea, marked facial edema, and neck distention that has progressed over the past 36 hours. Chest X-ray and MRI confirms a right hilar mass with a small pleural effusion in the right hemithorax. Question What next step is most appropriate for this patient?

Correct answer: Radiation therapy Explanation The patient is presenting with signs and symptoms of superior vena cava (SVC) syndrome, caused by obstruction of blood flow through the SVC due to compression, thrombosis, or infiltration from a right upper hilar malignancy. Obstruction of venous drainage at the upper thorax causes venous distention. Morbidity may result from irreversible thrombosis, central nervous system damage, or pulmonary complications. The most common cause of obstruction is malignant disease, with bronchogenic carcinoma at the top of the list. Signs and symptoms of SVC syndrome include swelling and cyanosis in the face and upper trunk, dyspnea, dysphagia, neck vein distension, and headache. Cerebral edema is a potential complication. In patients who present with serious airway or central nervous system systems, the preferred management strategies are SVC stenting and/or radiation therapy. Diuretics, steroids, and thrombolytics are sometimes used as temporizing measures, but their effectiveness is questionable. The diagnosis of pericardial effusion is not supported by the history, physical exam, and chest film; therefore, pericardiocentesis is not indicated. Direct surgical interventions including mediastinoscopy or thoracotomy are ineffective in this case. The small pleural effusion present on chest x-ray is unlikely to account for the patient's symptoms, and an in-office pleurocentesis would not be advisable.

Case A 23-month-old boy is brought in by his distraught mother. Both are crying. It seems that while crawling on the floor, the child found a penny and put it in his mouth. Before the mother could get to him, he swallowed it. She put her finger in the baby's mouth and could not find anything. She immediately drove to the emergency department. It has been about an hour since the ingestion. When you see the child, he is just finishing a 4-ounce bottle. When the mother takes it away, he cries. His color is pink. Pulse ox is 98%. Chest sounds are clear and present bilaterally. Heart sounds are clear, and he has a sinus tachycardia. Abdomen is soft and flat. There is no tenderness. On rectal examination, brown stool is found, and there is no blood. A plain X-ray shows an object about the size of a penny present in a slightly distended stomach. The lungs are clear, and the heart shadow is in the midline and appropriate size and shape. Question What should occur next regarding evaluation and treatment of this patient?

Correct answer: Repeat an abdominal X-ray in an hour. Explanation The critical issue in treating and managing swallowed foreign bodies is their size and their ability to pass through the pylorus. There are 5 areas of natural constriction in the pediatric age group where foreign bodies can get trapped: cricopharyngeal narrowing at the level of C6 the thoracic inlet aortic arch tracheal bifurcation hiatal narrowing When the foreign body has passed the pylorus, it will almost always work its way down and be passed in the stool. Of course, this is contingent on the object's presence prior to GI surgery or anomalies. Hence, the second X-ray of the abdomen to confirm its passage beyond the pylorus is warranted. Some may argue that the second X-ray is not necessary and the mother can take the child home and check the stool for the penny, only returning if it is not passed in 24 hours; the safest thing to do, however, is to confirm passage beyond the pylorus.

Case Ico-delete Highlights A newborn infant is delivered at 30 weeks since the last menstrual period. At birth, the infant weighs 1,500gm, but otherwise appears normal. Soon after birth, the infant becomes cyanotic and breathes with a grunting noise. Chest X-rays reveal dense lungs with significant atelectasis but no cardiovascular abnormalities. Question What is the most likely diagnosis?

Correct answer: Respiratory distress syndrome Explanation The leading cause of death among premature infants is respiratory distress syndrome. A fetus 30 weeks from the last menstrual period is actually 28 weeks old. At 28 weeks, type II pneumocytes first differentiate and begin to secrete surfactant. Cyanosis, difficulty breathing, and collapsed lungs are all associated with respiratory distress syndrome. In cases of congenital diaphragmatic hernia, the chest X-ray would reveal abdominal viscera in the pleural cavity. Coarctation of the aorta and Tetralogy of Fallot are incorrect. They would be associated with cardiac abnormalities. Bilateral renal agenesis would result in Potter syndrome, but no particular difficulty in breathing.

Question A woman is diagnosed with primary tuberculosis. She is 5 months pregnant. What drug should be avoided in this patient?

Correct answer: Streptomycin Explanation The correct response is streptomycin. Tuberculosis in pregnancy is treated with an initial regimen of isoniazid, rifampin, and ethambutol. Because the teratogenicity of pyrazinamide is not determined, pyrazinamide is added only if a resistance to the other drugs is documented/suspected, and susceptibility to pyrazinamide is likely. Streptomycin is contraindicated in pregnancy because it may cause congenital defects. Isoniazid, rifampin, and ethambutol are considered safe to use during pregnancy.

Case A 53-year-old woman presents with a 2-year history of chronic cough. The cough produces large volumes of grossly purulent sputum. She has a history of recurrent respiratory infections; they resulted in 5 hospitalizations in the past year. She also had similar complaints during the previous year. Shortness of breath limits her daily activity considerably. Upon pulmonary examination, bilateral breath sounds are audible, with inspiratory and expiratory crackles at the lung bases. Chest X-rays reveal increased lung volumes, non-flattened diaphragm, and thickened bronchial walls. Question What is the initial diagnosis?

Correct answer: Chronic bronchitis Explanation The clinical picture is suggestive of chronic bronchitis. Chronic bronchitis is defined by a clinical history of productive cough for 3 months of the year for 2 consecutive years. Smoking is the leading cause. The principle pathologic feature is airway injury and narrowing, hypertrophy of the airway mucous glands, infiltrate of inflammatory cells, and loss of ciliated epithelium. The cough produces thick, often purulent sputum because of the ongoing local inflammation and the high likelihood of bacterial colonization and infection. The increased mucous production and defective mucociliary escalator function lead to inspiratory and expiratory crackles. On imaging, common findings are hyperinflation of lung volumes, relatively depressed diaphragm, and parallel linear densities of thickened bronchial walls known as tram track lines. Asthma produces wheezing lung sounds. They are not heard in this patient. A productive cough is a rare clinical manifestation of emphysema, and the CXR is more descriptive of bronchitis. Pulmonary edema may present with mild exertional dyspnea or a nonproductive cough, although a frothy or blood-tinged sputum may be seen. Pulmonary fibrosis is a restrictive lung disease with the clinical features of progressive dyspnea. It is typically accompanied by a dry, persistent hacking cough.

Question A 3-year-old boy presents with extreme difficulty in breathing. Recently, the child came to the country with his parents from India. Past medical history reveals a 2-day history of sore throat and mild fever, and he has had some difficulty in swallowing. His condition deteriorated this afternoon. For some time, he has been sitting with his head still and jaw pushed forward, and he has been breathing shallowly. The physical examination reveals moderate respiratory distress with inspiratory stridor and inspiratory retractions of the supraclavicular and intercostal spaces. It is difficult to examine his throat, as the boy will not open his mouth. What is the best way to confirm the diagnosis and initially manage this patient?

Correct answer: Endotracheal intubation Explanation The clinical picture is suggestive of epiglottitis due to H. influenzae type B infection. Hib immunization is not routine in many parts of the world. The illness is characterized by a very sore throat, dysphagia, toxaemia, and rapidly progressive airway obstruction. Total airway obstruction is imminent by the time stridor appears. The child's appearance is most deceptive. The child does not appear to be in grave danger because the airway obstruction is masked by measures the child adopts to protect the airway; the child sits, the head is kept still and jaw pushed slightly forward, there is reluctance to open the mouth wide, and breathing is shallow with slow deliberate inspirations. The child will also be reluctant to speak (speech is soft), reluctant to swallow, and may drool. This is a medical emergency and no effort should be made to see the epiglottis, as it may precipitate complete obstruction and the patient may choke to death. There is no role for conservative therapy unless proper airway is maintained. The examination should only be done during operation under general anesthesia. After confirmation of epiglottitis, an endotracheal tube is passed, which is kept in situ for 24 to 72 hours until the fever and sore throat subside under proper antibiotic therapy, steroids, and oxygen. Once the airway is maintained, a blood culture and blood count can be taken, and intravenous chloramphenicol can be administered. Ampicillin may be substituted if the blood culture is sterile or grows sensitive organisms. Oral steroid (prednisolone) is given until the tube is out. If the child becomes gravely ill, or if he undergoes respiratory arrest, tracheostomy may be required.

Case A 30-year-old man presents with a 2-month history of coughing, and a 2-day history of coughing blood. He has been losing weight and sweating at night. On physical examination, the patient appears wasted, tachypneic, has bronchial breath sounds in the right upper lobe, and crepitations in the left upper lobe and right mid-zone. His direct sputum result comes back positive for acid-fast bacilli with Ziehl-Neelsen stain. His sputum is sent for culture and treatment is started. Refer to the image. Question Ico-delete Highlights Retrobulbar neuritis is the predominant adverse effect of what drug?

Correct answer: Ethambutol Explanation The correct response is ethambutol . This patient has all the signs and symptoms of pulmonary tuberculosis (TB). Direct sputum examination by Ziehl-Nielsen stain also helps the diagnosis, but it is still not confirmatory. Sputum needs to be cultured to check what kind of mycobacterium is causing this disease. It is important to start the treatment as soon as the culture is sent. The standard treatment for adult respiratory/pulmonary TB includes a complete 6-month regimen comprising of 2 months initial phase with 4 drugs: rifampin, isoniazid, pyrazinamide, and ethambutol. This is followed by a 4-month continuation phase consisting of 2 drugs: rifampin and isoniazid. Irrespective of the bacteriological status of the sputum, this is the recommended standard treatment for respiratory tuberculosis (including isolated pleural effusion or mediastinal lymphadenopathy). The 4th drug, ethambutol, may be omitted in patients with a low risk of resistance to isoniazid. Ethambutol should be started in individuals who are known or suspected to be HIV-positive, in those who have had previous treatment, and in immigrants and refugees of any ethnic group who are considered to have a significantly higher risk of resistance to isoniazid and other drugs. Like most medications, antituberculosis drugs also have some side effects. Since treatment is long-term, it is essential that patients are warned about and checked for side effects. If side effects are not explained well to the patient, it will decrease the compliance. The adverse effect of ethambutol is retrobulbar neuritis. The important side effects of anti-tubercular drugs are: INH: hepatotoxicity, peripheral neuritis, cutaneous hypersensitivity, (rarely) optic neuritis RMP: hepatotoxicity, nephrotoxicity, red discoloration of the body fluids, ''flu-syndrome', and thrombocytopenic purpura PZA: hepatotoxicity, hyperuricemia ETH: retrobulbar neuritis STM: nephrotoxicity, ototoxicity

Case A 72-year-old man is evaluated following admission to the hospital for a 1-year history of progressive dyspnea, nonproductive cough, weight loss, low-grade fevers, fatigue, and myalgias. His past medical history is remarkable for atrial fibrillation (for which he takes amiodarone), hypercholesterolemia, and recurrent urinary tract infections, for which his urologist prescribed nitrofurantoin on a chronic, prophylactic basis. He denies cigarette use, a history of murmurs or coronary artery disease, chills, fatigue, rhinitis, otalgia, chest pain, wheezing, hemoptysis, syncope, abdominal pain, rashes, peripheral edema, diaphoresis, arthralgias, vomiting, and urinary complaints. A bedside echocardiogram and electrocardiograms are unremarkable for abnormalities; a chest X-ray revealed peripheral reticular opacities at the lung bases and a generalized honeycombing pattern. Question What is the most likely diagnosis?

Correct answer: Idiopathic pulmonary fibrosis Explanation This patient's most likely diagnosis is idiopathic pulmonary fibrosis. Amiodarone, bleomycin, and nitrofurantoin are notable medications associated with pulmonary fibrosis. Most patients with idiopathic pulmonary fibrosis present with a gradual onset, which is often greater than 6 months. The clinical symptoms of idiopathic pulmonary fibrosis are nonspecific; symptoms often precede the diagnosis by a median of 1 - 2 years. Most patients present with exertional dyspnea and a nonproductive cough. Associated constitutional symptoms are uncommon. The physical examination reveals fine bibasilar inspiratory crackles (Velcro crackles) and digital clubbing in 25 - 50% of cases. Typical chest X-ray findings include peripheral reticular opacities (netlike linear and curvilinear densities) that are predominantly located at the lung bases, honeycombing (coarse reticular pattern), and lower lobe volume loss. The diagnosis of pulmonary embolism almost always occurs with underlying predisposing conditions present; venous thrombosis may result from a generalized hypercoagulable state, venous endothelial injury, or local stasis (Virchow triad). Most commonly, the initial manifestations of pulmonary embolism include an abrupt dyspnea and chest pain. Tachycardia and hypoxia are the most common clinical signs. Associated manifestations include fever, hypotension, cyanosis, pleural friction rub, and findings consistent with pulmonary consolidation. ECG most often shows tachycardia and various ST-T wave abnormalities, which are not specific for PE. Common chest radiographic abnormalities include atelectasis, pleural effusion, parenchymal opacities, and elevation of a hemidiaphragm. The classic radiographic findings of pulmonary infarction include a wedge-shaped, pleura-based triangular opacity with an apex pointing toward the hilus (Hampton hump) or decreased vascularity (Westermark sign). Sarcoidosis is a multisystem inflammatory disease of unknown etiology that manifests as non-caseating granulomas, predominantly in the lungs and intrathoracic lymph nodes. The presentation commonly includes systemic complaints of fever, anorexia, and arthralgias, as well as pulmonary complaints such as dyspnea on exertion, cough, chest pain, and (rarely) hemoptysis. Extrapulmonary findings are common, and include erythema nodosum, lower-extremity panniculitis, lupus pernio, a facial violaceous rash, maculopapular plaques, granulomatous uveitis, conjunctival lesions, scleral plaques, cardiomyopathy, and cranial nerve palsies. The chest X-ray commonly reveals air trapping, lymphadenopathy, and infiltrates. Goodpasture's disease is a condition of glomerulonephritis, with or without pulmonary hemorrhage, and the presence of circulating anti-glomerular basement membrane (anti-GBM) antibodies. Constitutional symptoms, such as malaise, chills and fever, and/or arthralgias, may precede or be concurrent with pulmonary or renal manifestations. Hemoptysis, cough, dyspnea, and shortness of breath describe pulmonary involvement, while hematuria, edema, high blood pressure, and uremia signify renal affliction. Significant anemia and chest pain may also occur. Wegener granulomatosis is a rare multisystem autoimmune disease of unknown etiology; its hallmark features include necrotizing granulomatous inflammation and pauci-immune vasculitis in small- and medium-sized blood vessels. A wide spectrum of extrapulmonary manifestations, which include recurrent respiratory infection in adults and upper and lower respiratory tract problems in children, is expected. Manifestations include constitutional complaints, conjunctivitis, episcleritis, uveitis, optic nerve vasculitis, retinal artery occlusion, nasolacrimal duct occlusion, proptosis, chronic sinusitis, epistaxis, saddle nose deformity, serous otitis media, hearing loss, strawberry gingival hyperplasia, stridor, myalgias, arthritis, arthralgias, glomerulonephritis, and renal failure. Later-onset findings reveal neuropathic and cranial nerve abnormalities. Other manifestations include CNS small- to medium-sized vessel vasculitis, a palpable purpura, splanchnic vasculitis, myocardial infarction, and/or cardiac friction rubs. References

Case Ico-delete Highlights A 35-year-old woman presents with a progressive 5-hour history of shortness of breath, cough, and wheezing. This morning she felt that she was 'catching a cold' because of fever, sore throat, and thin, purulent rhinorrhea for which she took aspirin. Her past medical history is significant for persistent rhinitis resistant to therapy; this was diagnosed as nonallergic rhinitis with eosinophilia. Question What finding on the examination will confirm your diagnosis?

Correct answer: Nasal polyposis Explanation Your patient most probably has aspirin-exacerbated respiratory disease, also called aspirin triad, or Samter's triad. It is characterized by nasal polyposis, nonallergic induced asthma, and aspirin sensitivity. Clinical symptoms of aspirin-sensitive patients are characterized by mucosal inflammation and rhinitis, severe asthma precipitated by aspirin ingestion, and aggressive bilateral nasal polyposis. Fever is not a specific sign. Laryngeal edema, rash, and cervical adenopathy are not characteristic of aspirin triad.

Case A 72-year-old man is evaluated at the bedside following hospital admission for a 1-year history of progressive dyspnea, nonproductive cough, weight loss, low-grade fevers, fatigue, and myalgias. His past medical history is remarkable for atrial fibrillation, for which he takes amiodarone, hypercholesterolemia, and recurrent urinary tract infections, for which his urologist prescribed nitrofurantoin on a chronic, prophylactic basis. He denies any cigarette use, history of murmurs or coronary artery disease, chills, fatigue, rhinitis, otalgia, chest pain, wheezing, hemoptysis, syncope, abdominal pain, rashes, peripheral edema, diaphoresis, arthralgias, vomiting, or urinary complaints. Arterial blood gas analysis demonstrated a PaO2 of 50 mmHg and pulse oximetry of SpO2 of 87%; bedside echocardiogram and electrocardiograms are unremarkable for abnormalities; a chest x-ray revealed peripheral reticular opacities at the lung bases and a generalized honeycombing pattern. Question What treatment is the most beneficial in the management of this patient?

Correct answer: Oxygen Explanation This patient's most likely diagnosis is idiopathic pulmonary fibrosis (IPF). It is defined as a specific form of chronic, progressive fibrosing interstitial pneumonia of unknown cause, primarily occurring in older adults, limited to the lungs, and associated with the histopathologic and/or radiologic pattern of usual interstitial pneumonia (UIP). Patients with hypoxemia (PaO2 < 55 mmHg or oxygen saturation as measured using pulse oximetry [SpO2] < 88%) at rest or with exercise should be prescribed oxygen therapy to maintain a saturation of at least 90% at rest, with sleep, and with exertion. Corticosteroids have not undergone appropriate trials to indicate a benefit for treating idiopathic pulmonary fibrosis. Current guidelines recommend against using corticosteroid therapy alone. Colchicine has been shown to inhibit fibroblast proliferation and collagen synthesis in vitro; however, evidence-based guidelines recommend that patients with idiopathic pulmonary fibrosis should not be treated with colchicine. Evidence-based guidelines recommend that the majority of patients with IPF should not be treated with N-acetylcysteine monotherapy. Bosentan is an endothelin receptor A and B antagonist that is approved for the treatment of pulmonary hypertension. While bosentan has been shown to have antifibrotic effects in an animal model of pulmonary fibrosis, current evidence-based guidelines recommend that patients with idiopathic pulmonary fibrosis should not be treated with bosentan.

Case A 22-year-old man presents with sudden onset of shortness of breath and right-sided chest pain. Symptoms began abruptly yesterday. He felt well prior to the onset of symptoms. He denies fever, hemoptysis, and upper respiratory symptoms. He smokes one pack per day; he has an otherwise non-contributory past medical history. On physical exam, the patient is in mild respiratory distress. He has a slightly elevated heart rate and respiratory rate. He is normotensive. His trachea appears deviated to the left. On pulmonary exam, breath sounds are diminished on the right. Hyperresonance is noted on percussion of the right chest compared to the left. Other than tachycardia, his cardiovascular exam is normal. Question What test finding is most diagnostic for your suspected diagnosis of this patient

Correct answer: Pleural line on chest X-ray (CXR) Explanation This patient is presenting with a spontaneous primary pneumothorax. A finding of a pleural line on chest x-ray (CXR) is diagnostic for this condition. A pneumothorax is a condition in which air is introduced into the lung cavity, either spontaneously or by trauma. In young people without known pulmonary pathology (especially more common in smokers), a pneumothorax may occur spontaneously. In older patients, other causes, such as a lung tumor or severe pulmonary disease, can lead to the collapse of the lung. As the lung collapses, the line from the edge of the pleura may be visible on CXR. Blunting of costophrenic angles on CXR would indicate some type of pleural effusion or fluid in the lungs. This patient's history and physical are not consistent with a pleural effusion (other than some dyspnea may be present). Specifically, percussion over a pleural effusion should produce a dullness, not a hyperresonance. Increased pH on ABG can certainly be seen in cases of respiratory alkalosis, associated with a pneumothorax. However, increases in pH can occur with numerous other conditions; therefore, an increase is not diagnostic for tension pneumothorax. Oxygen saturation may drop below 90% in cases of a pneumothorax; conversely, the oxygen saturation may be normal in milder cases of pneumothorax. However, as with the ABG, decreased oxygen saturation is a common finding with various pulmonary conditions; it alone is not diagnostic for pneumothorax. A sputum smear positive for acid-fast bacilli indicates a Mycobacterium pulmonary infection, such as tuberculosis. Classically, tuberculosis does not produce tracheal shift or hyperresonance on exam. This patient's history is not consistent with a pulmonary infection either.

Case A 35-year-old African American woman reports to the office with cough and dyspnea that have been bothering the patient for about 6-9 months. The patient denies chest pain, her BP is 138/90 mm Hg, and no obvious edema of extremities has been found. She also denies drugs, smoking, and alcohol. Patient's serum ACE levels are increased four-fold, and gamma globulin is 18%. There was no reaction to tuberculin test. Histology sample obtained during bronchoscopy revealed the presence of granulomas. Refer to the image. Question What is the most likely diagnosis?

Correct answer: Sarcoidosis Explanation Increased ACE levels, hypergammaglobulinemia, granulomatosis, and areas of consolidation in the mediastinum seen on MRI in the absence of a reaction to tuberculin are features of sarcoidosis in an African American woman. Bronchial asthma usually shows a barrel-like chest, and it does not result in a demonstrated elevation of ACE. Hodgkin's lymphoma is more common in men; the presence of Reed-Sternberg cells and sometimes lymphocytes, along with fibrosis in histological samples, points out the condition. Chronic bronchitis does not have ACE activity or radiographic signs of consolidation in the mediastinum. Hypersensitivity pneumonitis is characterized by numerous eosinophils in histological samples, along with radiographic signs of patchy fibrosis. Wegener's granulomatosis usually has the same signs of sarcoidosis, except for the histological picture. There, necrotic tissue is surrounded by areas of granulation, with plasma cells, lymphocytes, and large mononuclear cells.

Question A 4-year-old boy presents with a recurrent cough. The boy is small for his age, but his mother insists that he eats well. She also mentions that he frequently passes stool that is oily and foul smelling. He has had pneumonia several times, and the doctor notes that he is wheezing. What disorder does the boy have?

Correct answer: Cystic fibrosis Explanation Cystic fibrosis is one of the most common genetic disorders in the Caucasian population, with a carrier rate of approximately 1 in 28 Caucasians (1 in 31 Americans). It is inherited in an autosomal recessive fashion, and it is caused by mutations in the CFTR gene, which is located on the long arm of chromosome 7. The protein regulates Cl and Na transport across the epithelial membrane, and disruption leads to the formation and excretion of abnormally thick mucous, leading to severe pulmonary and digestive difficulties. Presence of the mucous in the lung leads to small airway obstruction. Infection is secondary; eventually, the bronchial walls become thick, the patient suffers from chronic hypoxemia, and muscular atrophy occurs. Due to pancreatic insufficiency, which is manifested as frequent passage of greasy, foul-smelling stool, abdominal protuberance, and deficiency of fat-soluble vitamins, affected individuals suffer from inadequate weight gain (apparent as early as 4 to 6 weeks of age) and growth retardation; these symptoms occur despite a good appetite. Patients also suffer from excessive sweating when stressed, and 10% develop insulin dependent diabetes. Average life span is early thirties. Although mutations in the CFTR gene have been identified, they are too numerous to provide a feasible DNA diagnostic test. However, a sweat test is available diagnostically. Alagille syndrome is a genetic disorder inherited in an autosomal dominant fashion. It is caused by mutations in the JAG1 gene, localized to chromosome 20p12. It is characterized by liver, heart, skeletal, and ophthalmologic abnormalities. Liver problems are usually observed within the first 3 months of life, and they include cholestasis, jaundice, pruritus, and paucity of bile ducts. Heart problems range from murmur to peripheral or pulmonic stenosis or tetralogy of Fallot. Skeletal and ophthalmologic abnormalities do not usually cause medical problems, but they can aid in diagnosis. They include butterfly vertebrae and posterior embryotoxon. A characteristic facies is also common among affected individuals. The face is shaped as an inverted triangle - prominent forehead, deep-set eyes, moderate hypertelorism, pointed chin, and a straight nose with a bulbous tip. These individuals also tend to show pancreatic insufficiency and growth failure. Diagnosis is based on clinical findings, although deletion studies are available clinically. Direct mutation analysis is available on a research basis only. The most common symptoms seen in 22q11-deletion syndrome include congenital heart disease (particularly conotruncal malformations), palatal abnormalities, hypocalcemia, immune deficiency, characteristic facial features, and learning difficulties. Most patients affected with this deletion syndrome can be diagnosed with fluorescence in situ hybridization (FISH) using DNA probes from the DiGeorge chromosomal region (DGCR). 94 percent of patients with the deletion have had a de novo event. 6 percent of patients inherit the deletion from a parent. Galactosemia is an autosomal recessive disorder of carbohydrate metabolism. Symptoms include hypoglycemia, jaundice, failure to thrive, vomiting, and sepsis. It is a progressive disorder if not diagnosed and treated, and it eventually leads to intellectual disability and death. In MOST states of the United States, this deficiency is included in newborn screening, and it can therefore be recognized before the infant leaves the hospital. William's syndrome is a microdeletion syndrome caused by a contiguous gene deletion of a critical region on chromosome 7q11. Patients share a common phenotype that includes mild intellectual disability, cardiovascular disease, connective tissue abnormalities, and a distinctive facies. The most common cardiac finding is supravalvar aortic stenosis. Connective tissue abnormalities include joint laxity, loose skin, hoarse voice, and tendency for hernia. Affected individuals usually have bitemporal narrowing, periorbital fullness, a short nose with a bulbous tip, malar hypoplasia, long philtrum, full lips, wide mouth with a small lower jaw, and prominent earlobes. Teeth tend to be small and widely spaced. Affected individuals tend to be overly friendly, anxious, and have attention deficit disorder. Growth pattern shows prenatal growth deficiency, failure to thrive in the first 4 years, and a brief pubertal spurt. Average height as an adult is less than the third percentile.

Question A 5-month-old infant presents with severe respiratory symptoms that include apnea and asphyxia; she is admitted to the hospital. 3 days before her admission, she developed rhinitis and a cough with wheezing. The mother became concerned when the child became lethargic. The physical examination reveals an agitated child with a persistent cough. She has a body temperature of 39°C, pulse of 190 BPM with a noted tachycardia, respirations of 76/min, and a blood pressure of 89/58 mm Hg. Her throat is clear, and auscultations of her lungs are significant for expiratory wheezing and rhonchi. Chest radiographs are positive for interstitial infiltrates and hyperexpansion. Blood gases reveal a relative hypoxemia and acidosis. The child is placed in isolation and subsequently intubated. A nasopharyngeal aspirate is sent to the laboratory for a stat rapid diagnostic test; it is positive. The child is treated with ribavirin. Based on the clinical presentation and treatment, what is the most likely diagnosis?

orrect answer: Respiratory syncytial virus Explanation Respiratory syncytial virus (RSV) is the major cause of lower respiratory tract illness (i.e., pneumonia, bronchiolitis, and tracheobronchitis) in young children. The virus belongs to the Paramyxoviridae family and to the genus Pneumovirus. The virus is very contagious, and illnesses tend to peak in the winter or spring. Wheezing, rhonchi, rales, interstitial infiltration, and hyperinflation may be present on chest radiographs. Fevers are usually seen and range between 38° to 40° C. Hypoxemia may be profound in children. There is a need for the measurement of an infant's arterial oxygen saturation because clinically the illness is difficult to assess. Because of its contagious nature, infants in hospitals are placed in isolation in order to prevent outbreaks within the hospital. The diagnosis of RSV can be made with reasonable accuracy based on the clinical and epidemiologic findings. Confirmation can be made by viral isolation (unfortunately this is time consuming) or by rapid diagnostic tests based that are available and provide results in about 10 minutes. Ribavirin, an antiviral agent, is available for treatment of RSV in infants where the diagnosis is made in the early stages of the disease. It has been shown to decrease viral shedding and increase oxygenation. Bordetella pertussis is a fastidious Gram-negative bacillus that is the causative agent of whooping cough. The organism is difficult to recover in cultures; PCR testing and DFA are the tests of choice, especially because results are acquired within hours instead of days. The organism specifically binds to ciliated epithelial cells. Since the nasopharynx is lined with ciliated epithelial cells, specimens obtained from this site are more reliable at obtaining valid results then any other specimen source. The infection is limited to the upper airways and pneumonia is a rare occurrence; thus, chest radiographs are usually normal. Children with whooping cough have paroxysms of coughing. When they gasp for breath the sound of this inspiration is the 'whoop' of whooping cough. Abnormal oxygen exchange is common and can cause the child to turn red, and sometimes blue. Repetitive coughing can lead to vomiting or choking on respiratory secretions. Haemophilus influenzae is a small Gram-negative coccobacillus that requires 2 supplements for growth (factor X and factor V); it is non-motile; grows best at 5 - 10% carbon dioxide, and grows on chocolate agar (because of the availability of X and V factors), but not on BAP or MacConkey agar. Pneumonia due to H. influenzae Type b typically occurs in patients between 4 months and 4 years old, patients with primary lung disease, and patients that are alcoholics. Radiologic findings are those of a segmented, lobar, bronchopneumonic, or interstitial pattern (listed in descending order of frequency). Cavitation is rare. Pleural effusion occurs in about 50% of cases, and the fluid is usually found to be sterile when cultured. Streptococcus pyogenes is a Gram-positive coccus; it is catalase-negative, beta-hemolytic on blood agar, and appears as chains on Gram stain. Definitive identification to distinguish it from other beta-hemolytic streptococci is the detection of its specific 'A' antigen by latex agglutination techniques. Pneumonia with this organism is extremely rare; it is associated in children with streptococcal pharyngitis, scarlet fever, and streptococcal pyoderma. Bacteremia is uncommon. It is universally sensitive to penicillin. Corynebacterium diphtheriae is an aerobic, Gram-positive rod that is club-shaped. It causes the disease diphtheria. Selective media (cystine tellurite agar) is used for the isolation and identification of the bacteria; the organism produces black colonies. The organism produces a toxin (diphtheria toxin) that is the major virulence factor; it enters the circulation and inhibits protein synthesis in a variety of tissues (with heart, nerves, and kidneys being particularly targeted). The disease will usually occur in individuals that have not been properly immunized (especially children). On physical examination, the patient will have a pseudomembrane formed at the back of the throat. This pseudomembrane is composed of bacteria, fibrin, dead epithelial cells, and red and white blood cells. Aspiration of this pseudomembrane can cause death by suffocation. In unvaccinated children, the mortality rate is approximately 20%. Treatment is usually with both antibiotics and diphtheria antitoxin.

Case Ico-delete Highlights A 3-year-old girl presents with her mother for an acute visit. The mother states that the chief issue is a persistent cough. About 4 weeks ago, she had a low-grade fever with rhinorrhea and cough for several days. The rhinorrhea and fever have since resolved, but the cough has continued. It bothers her primarily during the night and when she wakes first thing in the morning. Her medical history is significant only for mild seasonal allergies that appeared after she turned 3. Her vaccines are current, and her parents deny recent travel. Mom described the patient as a thriving, happy child who attends preschool 3 days a week. For the past few weeks, she has been tired and fussy during the day because the cough keeps her up at night. On exam, she appears overall healthy. She is in no respiratory distress, and her lungs are clear to auscultation. Question What intervention will most likely cut down on the coughing and improve the girl's sleep?

Correct answer: Begin daily montelukast. Explanation This girl's symptoms and history of allergic rhinitis are consistent with cough-variant asthma. Respiratory symptoms are characteristically worse at night and often triggered by respiratory (likely viral) infections. Since this patient has a history of allergic rhinitis, chances are that she is being exposed to allergens that are accentuating her symptoms. For this reason, it would be more beneficial to initiate treatment that would be both alleviating signs and symptoms of the suspected allergic rhinitis and potential asthma. The most effective option would be montelukast 4 mg granule packets. Montelukast is the best option because it is leukotriene inhibitor that is prescribed to not only treat asthma and prevent bronchospasms but also to treat perennial allergies and children as young as 6 months and adults; it also treats seasonal allergies in kids as young as 2 years. Diphenhydramine, Robitussin, and cutting down on milk are unnecessary and not appropriate choices. They are ineffective in treating a cough caused by asthma. Fluids and humidification are always a good recommendation, but in this case, they will not directly cut down on the cough if seasonal and/or perennial allergies are contributing factors.

Case Ico-delete Highlights A 76-year-old Caucasian man with a 90 pack per year smoking history presents with progressive fatigue, tachypnea, exertional dyspnea, cough, and lower extremity edema. Inspection of his chest and abdomen reveals an increased chest diameter, labored respiratory efforts with retractions and cyanosis, left parasternal and subxiphoid heaves, hepatojugular reflux, and a pulsatile liver. Additionally, there is scattered wheezes and crackles in his lungs and bilateral lower extremity edema. Question What physical exam findings would be most consistent with the underlying diagnosis?

Correct answer: Distended neck veins with prominent a or v waves Explanation The correct response is distended neck veins with prominent a or v waves. This patient's diagnosis is chronic cor pulmonale. Cor pulmonale is a change in the structure and function of the right ventricle and is caused by a primary disorder of the lungs or the pulmonary blood vessels. It is estimated to account for 6-7% of all types of adult heart disease in the United States, with chronic obstructive pulmonary disease (COPD) due to chronic bronchitis or emphysema the most common cause (the causative factor in more than 50% of cases). Cor pulmonale usually presents chronically; however, 2 main conditions can cause acute cor pulmonale: pulmonary embolism (more common) and acute respiratory distress syndrome (ARDS). In chronic cor pulmonale, right ventricular (RV) hypertrophy (RVH) generally predominates. Common symptoms are fatigue, tachypnea, exertional dyspnea, and cough. Other symptoms include anginal chest pain (due to right ventricular ischemia), hemoptysis, and, rarely, hoarseness due to compression of the left recurrent laryngeal nerve by a dilated pulmonary artery. Anorexia, right upper quadrant abdominal discomfort, and jaundice may occur due to passive hepatic congestion. Physical exam findings reflect the underlying lung disease or pulmonary hypertension, right ventricular hypertrophy (RVH), and RV failure. Expected signs include an increase in chest diameter, labored respiratory efforts with retractions of the chest wall, distended neck veins with prominent a or v waves, and cyanosis. Wheezes and crackles may be audible upon auscultation due to underlying lung disease. S2 heart sound splitting with an accentuated pulmonic component may be found early, while a systolic ejection murmur with sharp ejection click over the region of the pulmonary artery may be heard in advanced disease, along with a diastolic pulmonary regurgitation murmur. Other cardiac findings include third and fourth sounds and the systolic murmur of tricuspid regurgitation. RVH is characterized by a left parasternal or subxiphoid heave. Hepatojugular reflux and pulsatile liver are signs of RV failure with systemic venous congestion. On percussion, hyperresonance of the lungs may be a sign of underlying COPD; ascites can be seen in severe disease. Examination of the lower extremities reveals evidence of pitting edema. Dullness to percussion and increased tactile fremitus of the lungs suggests a lung consolidation, as in pneumonia, pulmonary edema, or pulmonary hemorrhage. Pericardial friction rubs suggest inflammation of the pericardial sac, while a laterally displaced and enlarged point of maximal impulse occurs with left ventricular hypertrophy, cardiomyopathy, ischemic heart disease, and congestive heart failure. As opposed to the pulmonary nature of Cor Pulmonale, in heart failure the etiology is cardiac. The heart doesn't pump blood with normal efficiency; subsequent organ congestion is caused by cardiac-related elevation of pulmonary or systemic venous pressures. The finding of a systolic ejection murmur located at the aortic valve area most likely suggests a diagnosis of aortic stenosis.

Case A 65-year-old man presents with dry mouth and difficulties rising from a chair, climbing stairs, and walking. His symptoms started about 1 year ago, are worse in the morning, and improve with exercise. He is a heavy smoker and was recently diagnosed with lung cancer. Question What is the most likely underlying mechanism of his weakness?

Correct answer: Paraneoplastic Explanation Paraneoplastic syndrome (Lambert-Eaton syndrome), resembling myasthenia gravis, occurs in some people with small cell carcinoma of the lungs. It usually manifests as progressive weakness in the large muscles. Lambert-Eaton syndrome is caused by the inhibition of voltage-gated calcium channels on the presynaptic membrane of the neuromuscular junction; this prevents the release of acetylcholine. As the muscles continue to contract, acetylcholine can build up in sufficient quantities for the strength to get better; weakness improves after repetitive muscle contraction. Although the underlying mechanism is autoimmune, Lambert-Eaton syndrome in this patient is regarded as paraneoplastic because it is a consequence of a cancer, not due to a local presence of cancer cells. Tumor infiltration is a local manifestation of a tumor associated with the production of various types of extracellular matrix-degrading enzymes. Local infiltration of the nerves excludes variegated symptoms of autonomic nervous system disturbance and fluctuating weakness in several muscle groups. Secondary spinal cord tumors usually follow hematogenous spread to the vertebral bodies, epidural expansion, and/or intramedullary metastasis. Subsequent symptoms of compression (i.e., pain, radicular, or medullar symptoms) gradually worsen, not improve with exercise. Nicotine poisoning is not likely; it is impossible to overdose on nicotine through smoking alone. Smoking causes vascular disease, cancer, lung disease, peptic ulcer, and reproductive disturbances (e.g., premature birth). Nicotine may contribute to tobacco-related disease, but direct causation has not been determined because nicotine is consumed simultaneously with a multitude of other potentially harmful substances that occur in tobacco smoke. The effects of nicotine on nerves and muscles are generally dose-dependent; they also occur in nicotine-tolerant individuals. Initially, nicotine has a short-lived stimulatory phase followed by a longer inhibitory phase which leads to a neuromuscular blockade. Neuromuscular symptoms include hypotonia, decreased deep tendon reflexes, weakness, fasciculations, and paralysis of muscles (including respiratory muscles). Cholinergic effects on the autonomic nervous system, often observed initially, include diaphoresis, salivation, lacrimation, increased bronchial secretions, miosis, and later mydriasis. Nicotine acts on the sympathetic ganglia, chemoreceptors of the aorta, and carotid bodies; it affects the adrenal medulla, releasing catecholamines.

Case A 22-year-old man presents with a sudden onset of shortness of breath and right-sided chest pain. Symptoms began yesterday, and he felt well prior to the onset of symptoms. He denies fever, hemoptysis, and upper respiratory symptoms. He is a 1 pack-per-day smoker; otherwise, he has a noncontributory past medical history. On physical exam, the patient is in mild respiratory distress, with a slightly elevated heart rate and respiratory rate. He is normotensive. His trachea appears deviated to the left. On pulmonary exam, breath sounds are diminished on the right. Hyperresonance is noted on percussion of the right chest compared to the left. Other than tachycardia, his cardiovascular exam is normal. A chest x-ray is obtained, and a pleural line is visible. Question What is the most likely diagnosis?

Correct answer: Pneumothorax Explanation This patient is presenting with a spontaneous primary pneumothorax. A finding of a pleural line on chest x-ray (CXR) is diagnostic for this condition. A pneumothorax is a condition in which air is introduced into the lung cavity, either spontaneously or by trauma. In young people without a known pulmonary pathology (more common in smokers especially), a pneumothorax may occur spontaneously. In older patients, other causes, such as a lung tumor or severe pulmonary disease, can lead to the collapse of the lung. As the lung collapses, the line from the edge of the pleura may be visible on CXR. Bronchiectasis is a chronic condition of the large bronchi associated with coughing, dyspnea, and wheezing. Cystic fibrosis is a common cause of bronchiectasis. This patient does not have a cough, wheezing, or a history of chronic pulmonary disease. Bronchitis is a rather vague term referring to acute or chronic inflammation of the bronchi. A viral infection is a common cause of acute bronchitis. Chronic obstructive pulmonary disease (COPD) is a common cause of chronic bronchitis. This patient's history and physical do not support any bronchial inflammation; they are not suggestive of an acute infection. A foreign body in the bronchus can cause acute shortness of breath, chest pain, and some of this patient's physical findings. Typically, an adult patient should be able to provide a history that no foreign bodies were aspirated; an x-ray would usually identify a foreign body. Furthermore, a foreign body would not produce the pleural line unless it caused a pneumothorax; therefore, an airway foreign body is unlikely in this patient. Pneumonia can cause chest pain and shortness of breath. However, the patient would likely feel more ill; there would be a history of malaise, fatigue, fever, chills, cough, and possibly nausea. The patient's x-ray and physical exam findings are not found with pneumonia. If pneumonia causes percussive changes on the lung exam, it produces dullness to percussion, not hyperresonance.

Case A 59-year-old man presents with a 3-month history of progressive exertional dyspnea. He also complains of dry cough, but he denies any history of fever, chest pain, or weight loss. You discover that he works in the ceramic industry at the outskirts of town. He is a non-smoker and drinks alcohol very occasionally. Examination reveals bibasilar crackles, and pulmonary function test indicates FEV1 of 67%, FVC of 73%, and TLC of 75% with DLCO of 65%. Chest x-ray shows "eggshell calcification" of hilar lymph nodes. Question What is the most likely cause of the patient's symptoms?

Correct answer: Silicosis Explanation Silicosis is the chronic fibrosing disease of the lungs produced by the prolonged and extensive exposure to free crystalline silica dust. It is the most prevalent occupational disease in the world. It requires years of exposure to crystalline silica to develop, as is found in mining, glassmaking, sand blasting, sand cutting, ceramics, glass manufacturing, and brickyards. It has a latency period of 20-30 years. Pulmonary function reveals a restrictive pattern with decreased DLCO. Small and discrete nodules are present in chest x-rays of simple silicosis. In advanced cases, large masses of dense, fibrous tissue are seen. In about 5-10% of cases, the nodes will calcify circumferentially, producing so-called "eggshell" calcification. Silica ingested by alveolar macrophages renders them ineffective, so a +PPD in these patients makes the diagnosis of latent tuberculosis infection, which should be treated with anti-tuberculosis medications. All patients with suspected silicosis should have yearly TB skin test and screening for lung malignancy. Sarcoidosis is an idiopathic inflammatory disorder that may cause noncaseating granuloma formation in the lungs. Some patients report symptoms such as cough but others are asymptomatic. Calcified lymph nodes are not a common finding of the disease. Sarcoid is less likely given that the patient has a history of silica exposure. HIV infection may result in reduced immunity and pneumonia, but pneumonia would not explain the calcified hilar lymph nodes. Mesothelioma is a malignancy involving the pleural lining of the lung. It is typically associated with asbestos, not silica exposure. Chest x-ray typically shows pleural thickening and effusions but not calcified hilar lymph nodes. Tuberculosis of the lungs results in the formation of caseating granulomas. Hilar lymph node enlargement may occur with acute presentations. Cavitations may also be present. In acute infections, other symptoms such as cough and weight loss are present.

Question Spirometry is conducted preoperatively on a 50-year-old woman about to undergo a cholecystectomy. The patient is a chronic smoker with a history of recent respiratory infection. What spirometry findings pose an increased risk for respiratory failure?

Correct answer: FVC<50% and FEV1 <50% Explanation Patients with history of chronic smoking have increased chances of developing respiratory failure with FEV1<50% and FVC<50%. A pulmonary function test is one of the important tests in a person undergoing abdominal and thoracic surgery. FEV1 < 50% is the greatest predictor of postoperative function and mortality. Both FEV1 and FVC < 50% predicted are used to predict respiratory failure postoperatively. Having FEV1 and FVC decreased lower than 50% predicted is important because it labels a person as at risk for the surgery. Pneumonia is an infective disease of the respiratory tract and is best diagnosed on chest radiograph.

Case A 72-year-old man is evaluated at bedside following hospital admission for a 1-year history of progressive dyspnea, weight loss, low-grade fevers, fatigue, and myalgias. He has a past medical history of coronary artery disease, COPD, obstructive sleep apnea, GERD, atrial fibrillation (for which he takes verapamil), and recurrent urinary tract infections (for which his urologist prescribed nitrofurantoin on a chronic, prophylactic basis). He denies any cigarette use, history of murmurs, chest pain, wheezing, hemoptysis, syncope, abdominal pain, rashes, peripheral edema, diaphoresis, or vomiting. His physical exam is remarkable for obesity, bilateral basilar crackles, and digital clubbing. A chest X-ray revealed peripheral reticular opacities at the lung bases and a generalized honeycombing pattern. Question What health maintenance statement is correct regarding this patient?

Correct answer: Evaluation by a pulmonologist and lung transplantation specialists should be made. Explanation Any patient suspected of having idiopathic pulmonary fibrosis, interstitial lung disease, or any another idiopathic interstitial pneumonia should be referred to a pulmonologist for further evaluation and management. Any patient diagnosed with idiopathic pulmonary fibrosis or probable idiopathic pulmonary fibrosis should be referred for lung transplantation evaluation, regardless of the vital capacity. The goal of any disease management strategy should include assessment and treatment of comorbid medical conditions. Common comorbid medical conditions found in patients with idiopathic pulmonary fibrosis (IPF) include chronic obstructive pulmonary disease, obstructive sleep apnea, gastroesophageal reflux disease, and coronary artery disease. Therefore, if any of these comorbid illnesses are present, they should be managed according to current practice guidelines. Vaccination against influenza and pneumococcal infection should be encouraged in all patients with idiopathic pulmonary fibrosis. Any patient with idiopathic pulmonary fibrosis who is overweight should be encouraged to meet with a nutritionist and make dietary changes to achieve ideal body weight. Maintaining adequate nutritional intake is important for quality of life in patients with idiopathic pulmonary fibrosis. Improving quality of life is an important goal in disease management. Deconditioning and subsequent functional impairment is a common problem in patients with idiopathic pulmonary fibrosis and negatively impacts quality of life. Patients with idiopathic pulmonary fibrosis should be evaluated for pulmonary rehabilitation and encouraged to participate in regular exercise to maintain a maximal degree of musculoskeletal conditioning.

Case Ico-delete Highlights A 70-year-old man with type II diabetes mellitus, hyperlipidemia, homocysteinemia, and metabolic syndrome presents with a 5-month history of excessive daytime sleepiness, a lack of refreshing sleep, a depressed mood, and an inability to focus at work as well as while driving. Additionally, he has been told by his wife that he snores rather loudly while sleeping. He denies fever, chills, headache, cold intolerance, weight loss, hair changes, hoarseness, dysphagia, chest pain, edema, palpitations, or changes in his bowel habits. On physical exam, he is found to be hypertensive. He has elevated BMI with abdominal obesity, and has an enlarged neck circumference; no other abnormalities are noted. Question What pathological mechanism best accounts for this patient's presentation?

Correct answer: Reduced inspiratory patency of the upper airway due to relaxation of the airway muscles Explanation This patient's history and physical exam are remarkable for obstructive sleep apnea/hypopnea syndrome (OSAHS), which is caused by the airway being sucked closed on inspiration during sleep. This occurs as the upper-airway dilating muscles relax during sleep. In patients with OSAHS, the dilating muscles fail to oppose negative pressure within the airway during inspiration. Although these patients have narrow upper airways already during wakefulness, increased muscle dilating activity helps to maintain airway patency. However, during sleep, muscle tone falls and the airway further narrows; snoring may commence before the airway occludes, and apnea results. Apneas and hypopneas terminate when the subject arouses (i.e., briefly wakes up). When it occurs in the setting of breast cancer, lung cancer, or lymphoma, the presence of dyspnea; facial, neck, trunk, or extremity swelling; headache; and venous distension, this suggests the possibility of superior cava syndrome. In Hashimoto's thyroiditis, there is a marked lymphocytic infiltration of the thyroid with germinal center formation, atrophy of the thyroid follicles accompanied by oxyphil metaplasia, absence of colloid, and mild to moderate fibrosis. The predominant pathologic event of chronic bronchitis is an inflammatory process in the airways, with mucosal thickening and mucus hypersecretion, resulting in diffuse airflow obstruction.

Case A 25-year-old male basketball player presents with acute onset shortness of breath associated with right-sided chest pain. The pain is unaffected by position and is worse with inspiration. He was grocery shopping when it started. He denies chest trauma. He had an upper respiratory infection earlier in the month that had resolved without incident. He smokes 1 pack of cigarettes per day and has no significant past medical history. On PE, he is afebrile; BP is 138/80 mm Hg, and pulse is 124; R is 24; and pulse oximetry is 94% on room air with mild respiratory distress. Trachea is midline. Lungs are clear to auscultation bilaterally with no wheezing or rhonchi; tactile fremitus and percussion are equal throughout posterior lung fields. Heart is tachycardic with normal S1 and S2, no murmur, rubs, or gallops. Question What is the most likely diagnosis?

Correct answer: Spontaneous pneumothorax Explanation The patient has a primary spontaneous pneumothorax (PSP). There is a higher incidence in male patients and it usually occurs in age 20-40. PSP is heavily associated with smoking. Patient presentation and physical exam findings will vary depending on the size of the pneumothorax. Most commonly, patients present with unilateral chest pain, dyspnea, and cough occurring with minimal activity in addition to tachycardia and tachypnea. A large PSP may become a tension pneumothorax, and the patient may have a deviated trachea and no breath sounds on affected side with increased resonance to percussion with respiratory distress and shock. Imaging of choice for a PSP is a chest radiograph, usually an upright inspiratory film. A white visceral pleural line is diagnostic. Supine or lateral decubitus views can also be used. Pericarditis is a differential diagnosis, but the chest pain is usually relieved with positioning. Pulmonary embolism is another differential, but this patient does not have risk factors for hypercoagulability, venous stasis, or vascular injury. A patient with community-acquired pneumonia (CAP) would usually be febrile and complain of associated symptoms such as a productive cough and malaise. Acute chest syndrome is associated with sickle cell disease.

Case A 68-year-old African-American woman with a past medical history of obstructive sleep apnea, hypertension, and COPD presents due to chronic, progressive dyspnea, which initially occurred upon exertion, but over the last 8 months has also been noted at rest. She is maintained on home oxygen for COPD. There is associated fatigue, substernal exertional chest pain, and 2 episodes of syncope, which occurred during exertion. She denies other symptoms. Physical exam reveals an oxygen saturation of 90%, a left parasternal lift, narrow splitting of the second heart sound, accentuation of the pulmonary component of the second heart sound, an early systolic ejection click, and an S4 gallop. No murmurs are identifiable. There is also +1 pitting edema noted bilaterally to the lower extremities to the mid-calf level. Bloodwork was remarkable for a hemoglobin level of 19 gm/dL and a hematocrit of 68%. An EKG and chest x-ray were performed. Question Which health maintenance statement is correct regarding this patient?

Correct answer: Expected complications include syncope, hypoxia, pedal edema, and hepatic congestion. Explanation This patient has a presentation most consistent with cor pulmonale. The electrocardiogram demonstrates right axis deviation, right ventricular hypertrophy, and right atrial enlargement. The chest x-ray indicates cardiac enlargement, with prominence of the pulmonary artery, right atrium, and right ventricle. Patient education regarding the importance of adherence to medical therapy is vital, because appropriate treatment of both hypoxia and underlying medical illness can improve mortality and morbidity. Complications of cor pulmonale include syncope, hypoxia, pedal edema, passive hepatic congestion, and death. Patients with cor pulmonale generally require close attention in the outpatient setting. It is appropriate to regularly assess the patient's oxygen needs and pulmonary function. Consider a formal program of pulmonary rehabilitation, as many patients benefit from this therapy. Phlebotomy is indicated in patients with chronic cor pulmonale and chronic hypoxia, causing severe polycythemia (defined as hematocrit of 65% or more). Phlebotomy results in a decrease in mean pulmonary artery pressure, a decrease in mean pulmonary vascular resistance, and an improvement in exercise performance in such patients.

Question A 40-year-old Asian-American man presents with a 3-day history of nausea and vomiting. He notes his health is good. He was started on a new medication 1 month ago for a positive PPD. What is a true statement regarding this patient?

Correct answer: His AST and ALT would be elevated Explanation The patient's AST and ALT would be elevated. Isoniazid (INH) toxicity should be suspected in any patient who is on this medicine and presents with abdominal pain. INH is not metabolized by the kidney; therefore, it should not affect kidney function. GGT is a nonspecific liver marker that goes up in any liver inflammation, so you would expect it elevated in INH-induced hepatitis. Blood work is suggested for any patient who is older than 35. The intervals to check should be at 1, 3, 6, and 9 months. INH can cause peripheral neuropathy by blocking the metabolism of pyridoxine. Gastroparesis alone is not typical for a peripheral neuropathy presentation.

Case Ico-delete Highlights An 18-month-old boy presents with a 2-day history of runny nose, slight cough, and low-grade fever. Over the last 24 hours, however, his condition has worsened; the child is tachypneic on exam. He is wheezing, and his breaths are associated with nasal flaring and chest retraction. Question What is the most common etiology of this patient's condition?

Correct answer: Respiratory syncytial virus Explanation This child presents with bronchiolitis. In approximately 70% of cases of bronchiolitis in children under the age of 2 years, respiratory syncytial virus (RSV) is the causative agent. The remaining cases are usually the result of other viruses, including influenza, parainfluenza, and adenovirus. Bronchiolitis is a lower respiratory tract infection marked by classic upper respiratory symptoms (e.g., rhinorrhea, cough); these are followed by progressive respiratory distress, including wheezing and stridor. Respiratory distress occurs because of the narrowing of the airway caused by inflammation. RSV infection occurs in adults as well, but it does not usually cause respiratory distress; the size of the adult airway makes it more capable of accommodating the inflammatory response elicited by the infection. Haemophilus influenzae infection is associated with epiglottitis, an upper respiratory infection that can cause life-threatening airway obstruction in children. Both Haemophilus influenzae and Streptococcus pneumoniae cause several other types of infections including otitis media, meningitis, and pneumonia. The advent of vaccines against both of these bacterial agents has significantly reduced the number of severe infections caused by them.

Case Ico-delete Highlights A 15-year-old girl with a history of mild asthma has had worsening episodes of cough, wheezing, and increasing bloody sputum over the past 5 months. She denies any weight loss, decreased appetite, lethargy, or unusual travel. She has increased her bronchodilator use, but she has not sought further care until now. Her mother has also noted occasional facial flushing with sweating that sometimes appears when she feels stressed, but not always. They were of brief duration at first, but seem to be lasting longer now. On exam, her respiratory rate is 32 breaths/min. with obvious respiratory distress, temperature is 98.6 F, heart rate 84 bpm and BP 114/76 mm Hg, oxygen saturation is 94%. Her throat is clear and on auscultation, breath sounds over the left hemithorax are diminished without retractions or wheezes, there are few fine crackles at the base. Right side is clear. The remainder of the exam is normal. Chest X-ray reveals a round area of increased opacification near the right hilar region. Complete blood count shows a normal white count and differential. Question What diagnosis would most easily explain the girl's symptoms?

Correct answer: Bronchial carcinoid tumor Explanation Bronchial carcinoid tumors, though rare in children, are the most common primary malignant lung tumor in children. They tend to arise in the perihilar region. Carcinoid tumors are rare neuroendocrine tumors occurring most often in the GI tract, especially the appendix. They can be associated with systemic symptoms due to the release of a variety of hormones. One of these hormones, serotonin, is thought to cause carcinoid syndrome. This consists of episodic flushing, wheezing and diarrhea, and it is noted in less than 1% of patients with bronchial carcinoid tumors. It is more common for midgut primary tumors. Patients may also present with recurrent pneumonia, cough, hemoptysis, wheezing, asthma, or chest pain. Metastasis, if present, usually occurs in mediastinal lymph nodes, the liver, bone, or skin. Resection is the preferred treatment for localized tumors; survival rates, when localized, are greater than 90%. Pulmonary embolism is extremely rare in pediatric patients, but it may be considered in a young female patient using oral contraceptives or in one who has had a recent abortion. It is also possible in a young male with recent leg trauma. Presenting symptoms include dyspnea, fever, pleuritic pain, cough, and hemoptysis. Bacterial pneumonia patients typically present with high temperatures and associated pleural effusions, along with a high percentage of band forms on lab study. Most commonly, it is secondary to acute viral bronchitis occurring during minor upper respiratory infection, or it is complicating an underlying chronic illness. Pulmonary hemosiderosis can occur as primary lung disease, or it can be secondary to cardiovascular or systemic disease. It is characterized by repeated episodes of intra-alveolar bleeding leading to abnormal accumulation of iron as hemosiderin in alveolar macrophages with subsequent fibrosis and severe anemia. Nonspecific recurrent or chronic pulmonary symptoms such as cough, dyspnea, tachypnea, and wheezing are observed most often. Hemoptysis may or may not occur in children. Vascular malformations include congenital structural anomalies between arteries and veins within blood vessels of the lungs, resulting in blood shunting and poor oxygenation. Many patients have no symptoms; others have dyspnea, shortness of breath with exertion, or bloody sputum. Long-term changes include clubbing, low oxygen saturation, and elevated hematocrit. Chest X-ray will usually reveal abnormal blood vessels. In addition, a murmur may be heard on auscultation.

Case A 75-year-old man is recovering from a pneumonia caused by Streptococcus pneumoniae; his condition suddenly deteriorates. He presents after developing a persistent fever, chills, cough, and diaphoresis. A CBC reveals leukocytosis with a left-shift. A chest X-ray demonstrates an air-fluid level in the pleural space, which is suggestive of an abscess. Question What organism caused the patient's deterioration?

Correct answer: Staphylococcus aureus Explanation The clinical picture is suggestive of a pleural empyema. Of the organisms listed, empyemas are commonly caused by Staphylococcus aureus, S. pneumoniae, S. pyogenes, and H. influenza in the adult patient. The other organisms listed either do not cause an empyema, or they infrequently cause an empyema.

Case A 3-year-old girl presents with a 2-day history of a sore throat and fever. This morning, she was hoarse and seemed to be having more difficulty breathing. On exam, she appears to be in distress and has an oral temperature of 100.0°F. Tympanic membranes are pink but not bulging. Nares are patent without rhinorrhea. She has a barking cough, stridor at rest, and nasal flaring. Question What treatment is most appropriate in the care of this child?

Correct answer: Admit patient, start humidified oxygen and intermittent racemic epinephrine Explanation This child is in acute respiratory distress as depicted by her stridor at rest, nasal flaring, and state of distress.The situation warrants admission, treatment, and careful observation. Corticosteroids via nebulizer may also be effective. Specific antiviral treatments and parainfluenza vaccines are not currently available. Parainfluenza virus will not respond to antibiotic treatment. Although home care with a vaporizer can be effective for patients with milder cases of croup, this child is severely ill.

Case A 31-year-old HIV-positive woman presents for ongoing care. She was diagnosed with HIV 2 years ago, and she began antiretroviral therapy. Her CD4 T cell count is 400 cells/mL, and she has a history of oral candidiasis. As part of her evaluation, a tuberculin skin test (TST) is performed using 5 TU of purified protein derivative (PPD). The test site is examined 48 hours later and the skin reaction is measured. Question What is the minimum diameter of induration at which this test result should be considered positive in this patient?

Correct answer: 5 mm Explanation The correct response is 5 mm. Persons with HIV should be tested yearly for tuberculosis using the purified protein derivative (PPD) skin test, also referred to as the Mantoux test. In those with HIV, and in certain other cases (refer to the table), an induration of more than, or equal to, 5 mm is considered positive. Preventive therapy should be prescribed for all patients having a positive PPD. Those with a positive skin test (or high-risk exposure) should undergo prophylaxis. A common regimen consists of isoniazid (INH) and pyridoxine daily, usually for at least 1 year. CDC recommends INH (300 mg/day) for a period of 12 months to any HIV-infected persons with positive TST (>5 mm), along with supplemental pyridoxine (25 - 50 mg/day) to prevent peripheral neuropathy. Both isoniazid-resistant and multidrug-resistant strains of Mycobacterium tuberculosis are becoming more prevalent. Classifying positive TST reactions: Interpretation of the tuberculin skin test*: Induration (dia.) Positive in persons with >5 mm - HIV infection - Persons with chest X-ray findings consistent with prior TB - Close contacts of a person with infectious TB - Patients with organ transplant and other immunosuppressed persons >10 mm - Medical risk factors such as chronic renal disease, diabetes, gastrectomy, and silicosis - Residents/employees of high-risk congregate settings (jails, nursing homes, hospitals and other long-term facilities for elderly) - IV drug users - Mycobacteriology laboratory personnel >15 mm - Healthy persons without known risk factors * 5 TU PPD

Case A 6-year-old boy with Down's syndrome is brought to the emergency department after a drowning accident in a lake. Submersion time is unknown, although he was missing for some time. He was resuscitated and intubated by emergency medical technicians during transport. On arrival, his heart rate is 76 beats/min, BP 104/72 mm Hg, and rectal temperature is 82 F (28° C). He remains comatose, exhibiting non-purposeful flexion withdrawal to pain, His pupils are equal and reactive. Question What is the best way of treating the patient's drowning-related hypothermia?

Correct answer: Active internal warming Explanation Active internal warming is the gold standard for warming patients who have cardiac arrest or ongoing cardiovascular instability due to hypothermia, and it is most commonly achieved with cardiopulmonary bypass or extracorporeal life support. Active internal warming by arteriovenous circulation, esophageal rewarming tubes, and body cavity lavage are also methods that have been described as being effective. Passive rewarming, in which the patient is kept warm and protected from wind and cold, can be initiated at the scene. Due the patient's significant hypothermia, passive rewarming is inadequate. Active external warming by use of such measures as warmed air or heat packs may cause surface vasodilatation, leading to cardiovascular instability. Maintaining hypothermia may be a reasonable therapeutic option for an internal temperature of 90° to 93° F (32° to 34°C) for 12 to 24 hours, for children who remain comatose but who are cardiovascularly stable following return of spontaneous circulation. Rewarming should always be provided to children who have severe hypothermia of < 82° F (28° C), and considered for moderate hypothermia of 82° to 90° F (28° to 32° C) for children who have cardiovascular instability or shock-related coagulopathy. References:

Case A 33-year-old man presents with shortness of breath, wheezing, mild fever, and fatigue. He has had several similar episodes in the past, and each previous episode began after a cold that moved into his chest. Over the past several weeks, he has had a productive cough most mornings. He smokes on a social basis. Question What is the most likely diagnosis?

Correct answer: Acute asthmatic bronchitis Explanation The clinical picture is suggestive of acute asthmatic bronchitis. Typical symptoms include a persistent cough with mucus that can become thicker and more profuse, as well as dyspnea, mild fever, and chest pains. It is commonly caused by a virus infecting the lining of the bronchial tree. Wheezing can occur for several weeks. In this particular patient, the cold (viral infection) moved to his lungs and induced acute asthmatic bronchitis. Productive cough is not seen in chronic emphysema patients. Chronic bronchitis is defined as a clinical history of productive cough for 3 months of the year for 2 consecutive years, which is not described in this patient. Cor pulmonale has similar symptoms to bronchitis and emphysema, but it also has elevated jugular venous pressures, parasternal lift, edema, hepatomegaly, and ascites, which are not seen in this patient. Bronchiectasis presents with dyspnea, wheezing, chronic cough with copious mucous production, hemoptysis, and pleuritic chest pain. Hemoptysis and pleuritic chest pain are not present in this patient.

Case A 4-year-old boy presents with a 1-month history of weight loss, fevers, cough, and night sweats. He and his family moved to the United States from Africa 3 months ago. He is a thin, pale boy in no acute distress. His heart rate and rhythm are regular, his lungs are clear to auscultation, and he has no organomegaly. Question What initial tests would most likely have the most value?

Correct answer: Chest X-ray and tuberculin skin test Explanation This boy presents with classic symptoms of tuberculosis. Clinical manifestations often appear 1-6 months after infection, which corresponds to the timing of the child's recent immigration from a high-risk continent. The CDC suggests that TB diagnosis in the pediatric population is challenging, as children are less likely to have a positive result. They recommend using a combination of positive tuberculin skin test, chest X-ray, and clinical signs and symptoms for diagnosis. The tuberculin skin test consists of 5 tuberculin units of purified protein derivative injected intradermally into the forearm. Measurement of the resulting induration indicates the likelihood of tuberculosis infection. The chest X-ray in a child with tuberculosis will range from normal presentation to containing diverse abnormalities, such as hilar lymphadenopathy; subcarinal, paratracheal, or mediastinal node; infiltrate; pleural effusion; cavitary lesions; or miliary disease. Hepatitis B symptoms include nausea, malaise, and jaundice. Syphilis symptoms include chancres, rash, and lymphadenopathy. Acute illness due to Schistosoma infection presents as fever, malaise, cough, abdominal pain, diarrhea, hepatosplenomegaly, and lymphadenopathy.

Case A 14-year-old boy presents with worsening shortness of breath; it most often occurs when he plays soccer. He often awakens in the middle of the night due to 'attacks': he starts to feel anxious because he feels like he cannot breathe, and he experiences chest tightness. He suffers from a dry cough, especially after playing sports. His mother has put a humidifier in his room and has him use his sister's inhaler, which seems to help temporarily; the boy has been using it 5 - 6 times daily. The family history is significant for asthma in his sister, father, and 3 other paternal relatives. The patient and his mother are not aware of any allergies. He denies fever, chills, and chest pain. In between "attacks", he feels well and normal. The patient's past medical history is noncontributory. There are no known medical conditions; he has no drug allergies, and he has not had any surgeries. Other than the aforementioned inhaler, he does not take any medications. Question In addition to his own albuterol inhaler, what medication should be prescribed for this patient?

Correct answer: Daily low-dose inhaled budesonide Explanation This patient is exhibiting persistent asthma symptoms. Using the stepwise approach to treatment, the 2nd step (after using a short-acting beta-agonist, such as albuterol) is to add a low-dose inhaled corticosteroid. Alternatives include cromolyn, a leukotriene receptor antagonist, nedocromil, and theophylline. For this patient, daily low-dose inhaled budesonide would be most appropriate. He should then be monitored for response; if necessary, the medication should be adjusted. The patient and his mother should receive patient education on asthma, the use of peak flow meters, and the proper use of medications. A burst of oral prednisone can be useful in the management of an acute exacerbation of asthma. However, due to the side effects of systemic steroids, its use is discouraged. This patient is not in acute distress and should start with inhaled steroids, which pose fewer side effects than oral/systemic steroids. Daily-inhaled salmeterol, a long-acting beta-agonist (or LABA), should be added to an inhaled corticosteroid if low-medium doses of the inhaled steroid alone are unable to control symptoms. The salmeterol is not recommended as a stand-alone therapy, and it should only be used with other asthma control medications. Daily oral zileuton, a 5-lipoxygenase inhibitor, is extremely expensive and is dosed 4 times daily. It should not be recommended initially when the preferred agent (low-dose inhaled corticosteroid) has not yet been tried. Additionally, compliance in teens can be difficult, so a 4-times-daily medication is not ideal. Subcutaneous injections of omalizumab, which is an immunomodulator, are an option in step 5 and 6 patients, who have failed to achieve symptom control with multiple and high-dose medications. Omalizumab is not appropriate in this patient's case.

Question During your well newborn morning nursery rounds, a nurse tells you about an abnormal finding on an antenatal ultrasound that was done. The rest of the maternal history is unremarkable. The infant was born last night. She was born at term and without any complications, and she seems to be feeding and transitioning well; however, she has not passed stool yet. What antenatal ultrasound finding might be a marker for cystic fibrosis?

Correct answer: Echogenic bowel Explanation Echogenic bowel, which is sometimes seen on antenatal ultrasound (US) that is done during the 2nd trimester, may be a marker for aneuploidy, congenital infections such as toxoplasmosis and cytomegalovirus, rare intestinal disorders, and cystic fibrosis. However, most infants who exhibit this finding are normal, and most will pass stool by 48 hours. A delay should prompt a repeat exam on abdomen and rectum as well as assessment of adequacy of feeding. If the delay persists beyond 48 hours, a barium enema to evaluate for Hirschsprung's disease is indicated, and a consult for rectal biopsy should be considered. Newborns with cystic fibrosis also may present with meconium plug syndrome, a usually transient disorder of the newborn colon causing delayed passage of meconium and intestinal dilatation, although meconium ileus is more frequent and characteristic Choroid plexus cysts may be commonly seen on US prior to 24 weeks' gestation. There is a possible association of ultrasound detected choroid plexus cyst and chromosomal problems, such as trisomy-18 and trisomy-21. If it is not associated with other anomalies, they are unlikely to be of any significance. Follow-up is not needed if results of the physical examination are normal. An echogenic intracardiac focus or a bright spot seen near the papillary muscle of the left ventricle has a correlation with trisomy-21, but it is a normal finding in most cases. If there are no clinical findings suggestive of Down's syndrome, or there is a normal antenatal karyotype, the family can be reassured that no further testing is needed. Mild enlargement or asymmetry of the cerebral ventricles might have some significance. The obstetric service will usually have performed serial US, karyotype, and cytomegalovirus and toxoplasmosis testing. A physical exam and cranial US should be done after delivery to document the brain anatomy and size of the ventricles. Any abnormal findings should be referred for consultation. A mild degree of ventricular enlargement often represents normal variation; however, if enlarged or asymmetric, it may also place an infant at risk for neurodevelopmental problems. Hydronephrosis is found on 4.5% of pregnancies. Generally accepted criteria for fetal hydronephrosis are greater than 4 mm anteroposterior diameter renal pelvic dilatation in the second trimester, and greater than 7 mm dilation in the third trimester. Postnatal US should be done on infants who meet the criteria, although normal findings do not rule out vesicoureteral reflux. 10-30% of newborns with antenatal hydronephrosis have vesicoureteral reflux and are at risk of renal scarring from urinary tract infection.

Case Ico-delete Highlights A 55-year-old woman presents with a slight cough she has had for about a week. She is a nonsmoker, and she does not remember having a fever or feeling sick. The patient currently works as a third-grade teacher and has done this for 30 years. She has no past medical history of significant pulmonary diagnoses. Auscultation of the chest reveals clear lung fields. A chest X-ray shows a subpleural "coin lesion" in the right upper lobe. Question What is the most likely diagnosis?

Correct answer: Granuloma Explanation The most likely diagnosis is granuloma. The differential diagnosis of a solitary coin lesion includes: Lung carcinoma (most commonly an adenocarcinoma) Granuloma Hamartoma A small cell carcinoma (or oat cell carcinoma) tends to spread very quickly and does not remain localized. Moreover, it is most common with a history of smoking. Silicosis results from years of inhaling dust containing silica, and it produces diffusely scattered nodules in the lungs. Bronchiectasis results from inflammation, destruction, and dilatation of the bronchi; it does not produce a round discreet lesion. Exogenous lipid pneumonia is due to aspiration of a substance that is oily or contains lipids. It is not typically localized.

Case A 55-year-old woman presents with a several-month history of increasing cough and dyspnea. She also has increased serum urea, nitrogen, and serum creatinine. A chest X-ray shows multiple bilateral small nodules. A renal biopsy shows a focal necrotizing vasculitis; her antineutrophil cytoplasmic autoantibody (ANCA) test is positive at 1:160. Question What additional finding would be most likely to occur?

Correct answer: Hemoptysis Explanation The history of upper airway involvement (cough with dyspnea), the nodular pattern on X-ray, and the renal biopsy showing focal necrotizing vasculitis suggests the diagnosis of Wegener's granulomatosis. Pulmonary (as well as upper respiratory tract) involvement with Wegener's granulomatosis commonly leads to hemorrhage, which manifests as hemoptysis. Although the etiology is unknown, it is believed to represent an antigen-triggered immunologic reaction. Pulmonary manifestations include cough, dyspnea, chest discomfort, and hemoptysis. In most cases, the extrarenal disease precedes the onset of renal disease. Although the heart may be involved with Wegener's granulomatosis, it does not usually lead to symptoms of ischemia (angina). The vasculitis does not typically involve the pericardium extensively; therefore, hemorrhagic pericarditis would not be a finding. The renal failure could lead to a fibrinous pericarditis. Endocarditis is not a feature of Wegener's granulomatosis. The vasculitis of Wegener's granulomatosis involves very small arteries and capillaries. Hemothorax is more typical of trauma.

Case A 22-year-old woman presents with shortness of breath. She has a history of intermittent wheezing while exercising. On examination, you find that her pulse rate is increased; there are diffuse wheezes on pulmonary auscultation. Her oxygen saturation is 95%. Question What would be the most effective therapy in this patient?

Correct answer: Inhaled albuterol Explanation In the case of an acute attack, asthmatic patients who lack signs of impending ventilatory collapse should be treated with inhaled aerosolized β2 agonist (albuterol) every 20 minutes by nebulizer for 3 doses, with reduced frequency thereafter. Inhaled steroids (beclomethasone) have a delayed action as compared to the α2 agonist; therefore, they are not the first choice in case of an acute attack. Intramuscular therapy also shows a delayed response as compared to inhalation of β2 drugs. Cromolyn sodium is not effective in an emergency. It is used to prevent attacks. Cromolyn sodium is an anti-inflammatory medication. It works by preventing the release of substances in the body that cause inflammation. I.V aminophylline is also less effective as compared to inhaled albuterol. Aminophylline is a bronchodilator. Aminophylline works in several ways; it relaxes muscles in the lungs and chest to allow more air in, decreases the sensitivity of the lungs to allergens and other substances that cause inflammation, and increases the contractions of the diaphragm to draw more air into the lungs.

Case Ico-delete Highlights A 3-year-old boy presents for his annual well-child check. He and his family moved to the United States from Africa 4 months ago. He is a thin boy in no acute distress. His heart rate and rhythm are regular. His lungs are clear, and he has no hepatosplenomegaly. Due to his recent immigration, he is given a tuberculin skin test. He has no known chronic medical conditions and no known contacts with tuberculosis disease. The test shows an induration of 6 mm. Question What is the next step in the management of this patient?

Correct answer: Normal well-child care Explanation For a child who has immigrated from a high-prevalence area of the world but has no close contacts with tuberculosis disease, is not immunocompromised, and has no signs or symptoms of the disease, the induration must be 10 mm or greater to be positive. Therefore, this child has a negative test and should receive normal well-child care. If the test were positive, he would require a chest X-ray and further workup. If he were found to have latent infection without active disease, he would be treated with isoniazid alone. If he were found to have a pulmonary or extrapulmonary disease, except meningitis, he would be treated with isoniazid, rifampin, pyrazinamide, and ethambutol. Gastric aspirate sampling is one way of isolating the organism if a patient has active tuberculosis disease.

Case A 55-year-old woman was diagnosed with small cell cancer of the lung 2 months ago; she now presents with increasing dysphagia, respiratory difficulties, and weakness of the upper limb. Her vital signs are pulse 85/min, BP 120/90 mm Hg, resp. 12/min, and temp. 37.7° C. On examination, she has ptosis of both eyes, and she reports diplopia. Her pupillary responses are normal. The strength in the muscles of her arm on testing is 2/5; on repeated testing, the strength improves to 4/5. Sensation is intact in both upper limbs. Question Of the following treatments, what would be most effective in treating the patient's symptoms?

Correct answer: Plasmapheresis Explanation Lambert-Eaton syndrome is caused by antibodies to the presynaptic calcium channel, which decreases the release of acetylcholine. Repeated stimulation of the nerve ending increases the intracellular calcium concentration, which allows enough acetylcholine to be released to cause muscle contraction. Of the treatments listed, plasmapheresis would be most effective at removing the circulating antibodies. Other therapies include immunosuppressants (e.g., prednisone or azathioprine). The Lambert-Eaton (or myasthenic syndrome) may occur as a paraneoplastic syndrome of small cell lung cancer. The presentation is similar to myasthenia gravis; however, in myasthenia gravis, the weakness is made worse by sustained movement while it is improved in the Lambert-Eaton syndrome. Myasthenia gravis is caused by antibodies to the acetylcholine receptor; consequently, the treatment of myasthenia gravis lies in increasing the acetylcholine concentration in the synaptic cleft using an acetylcholinesterase inhibitor (e.g., neostigmine). Acetylcholinesterase inhibitors are sometimes used to treat Lambert-Eaton syndrome, but they are rarely effective alone. Atropine is a cholinergic antagonist at muscarinic, not nicotinic receptors; therefore it would not be useful in this case. Amikacin is an aminoglycoside; it can cause weakness of skeletal muscles. Cisapride is a promotility agent use for dysphagia or gastroesophageal reflux. It has been withdrawn from the United States market, and it is available only to patients who meet specific criteria.

Case A 30-year-old immunocompromised patient presents with a 2-week history of breathlessness and a nonproductive dry cough. The patient is afebrile, pulse is 100, and BP is 110/70 mm Hg. On auscultation, scattered rales all over the chest are heard. A chest X-ray shows diffuse air-space and interstitial shadowing in both lungs. The shadowing is more prevalent in the apical region. Question What is the most likely diagnosis?

Correct answer: Pneumocystis pneumoniae Explanation The patient being immunocompromised narrows the choices to Pneumocystis pneumoniae and tuberculosis; diffuse infiltrate is more common in Pneumocystis infections of the lung. The fungus Pneumocystis jirovecii causes pneumocystis pneumonia. Pneumocystis pneumoniae is the most common opportunistic infection in patients with acquired immunodeficiency syndrome (AIDS). Tuberculosis shows localized infiltrate; it is commonly seen at the apices with hilar lymphadenopathy. Bacterial pneumonia will have an acute history of 2-4 days with fever and a nonproductive cough. The X-ray will show lobar infiltrate; it is rarely found throughout the lung. Viral pneumonia will have a fever before the onset of the respiratory symptoms. Diagnosis is usually by isolating the organisms as well as serological tests. The presence of cavities in the lung indicates bacterial causes, fungal (Mycoplasma) causes, tuberculous infection, or neoplasm.

Case Ico-delete Highlights A 45-year-old man presents with a 2-day history of sharp left-sided chest pain. It is aggravated by taking a deep breath. He denies any trauma to the chest. On examination, he is febrile, dyspneic, and has rales on auscultation. Chest X-ray reveals consolidation in the left lower lobe. Question What is the most likely cause of his chest pain?

Correct answer: Pneumonia Explanation Pneumonia often causes a sharp pain that is aggravated by inspiration. Other signs and symptoms of pneumonia (e.g., cough, fever, dyspnea, and rales on auscultation) may also be present. A consolidation on chest X-ray is also consistent with the diagnosis of pneumonia. Esophageal reflux pain is a substernal pain that is usually described as burning. It may last for up to 1 hour. It is relieved by antacids and may be aggravated by alcohol and postprandial recumbency. There are typically no X-ray findings associated with reflux. Aortic dissection pain is of abrupt onset; it is usually described as severe and tearing. It may radiate from the anterior chest wall posteriorly to the interscapular region. On examination, the blood pressure may be elevated. A pericardial rub and the diastolic murmur of aortic insufficiency radiating along the right sternal border may be auscultated. The mediastinum may be widened on chest X-ray. Pulmonary embolism pain is usually of abrupt onset and pleuritic in nature. It has a duration of several minutes to a few hours. There may be predisposing factors, such as deep venous thrombosis or long periods of immobilization. Patients may also report hemoptysis. On examination, the patients are dyspneic and tachypneic with tachycardia. Unstable angina pain is usually described as tightness or pressure on the chest; it lasts for 10-20 minutes. It is usually left-sided or retrosternal, and it may radiate to the jaw and neck. It often occurs at rest, but it may be precipitated by physical exertion and emotional stress. On examination, a systolic murmur of mitral regurgitation may be auscultated during the pain.

Case A 28-year-old woman of Norwegian descent presents with cough, dyspnea, joint pain, fever, fatigue, and weakness. Her history is negative for occupational and environmental exposure. A previous tuberculin skin test was negative. Chest X-ray shows interstitial infiltrate and bilateral hilar lymphadenopathy. Ophthalmological investigation (slit lamp) reveals a clinically silent uveitis. Blood tests show hypercalcemia, and transbronchial biopsy shows non-caseating granulomas. Therapy with systemic corticosteroids is initiated. Question What skin lesions would be most likely revealed in this patient's legs upon physical examination?

Correct answer: Erythema nodosum Explanation This patient presents with classical findings of sarcoidosis. Erythema nodosum (EN) may occur in up to 39% of sarcoidosis cases, characterized by tender erythematous nodules most commonly located on the anterior tibial areas. EN is usually related to the acute presentation of systemic sarcoidosis (fever, malaise, polyarthralgias, and bilateral hilar lymphadenopathy) and has a self-limited course of about 1 month. EN is a non-specific skin lesion that is also observed in rheumatic fever, inflammatory bowel disease, infections (coccidioidomycosis, histoplasmosis, α-hemolytic streptococci, tuberculosis, and syphilis), patients with hypersensitivity reactions to drugs (oral contraceptives, NSAIDs), and pregnancy. Despite all these associations, EN is typically idiopathic in approximately 55% of the cases. An EN biopsy should be avoided, as histopathology shows non-specific subcutaneous inflammation and vasculitis. While lupus pernio is a skin manifestation of sarcoidosis, it is unusual on the legs; it is a specific cutaneous manifestation of chronic systemic sarcoidosis. Lupus pernio is characterized by red-to-purple indurated papules, plaques, or nodules; they are most commonly seen on the mid-face, particularly the alar rim of the nose and the hands. Biopsy shows granulomas and is diagnostic of sarcoidosis. Erythematous patches and plaques can be seen in multiple situations, including systemic lupus erythematosus (SLE) and dermatomyositis (DM). The classical SLE rash has a butterfly shape and is localized on the cheeks and the nose ridge. In DM, the rash is localized on the face, neck, back, and shoulders (shawl sign). Like erythema nodosum, pretibial myxedema is usually located on the anterior tibial areas. Pretibial myxedema is characterized by areas of a waxy, discolored induration resultant of glycosaminoglycans accumulation in Grave's disease. Erythema multiforme presents with multiple types of lesions, including macules, papules, and vesicles; however, the classic description is the "target lesion," which is characterized by multiple erythematous rings around a dusky blue center (commonly located on the trunk or extremities). It is usually seen in drug reactions (penicillin, sulfa drugs, phenytoin, and aspirin), but it can also be associated with infections (Mycoplasma pneumoniae, HSV), cancers (lymphoma, leukemia), and autoimmune conditions (collagen diseases, vasculitis). It is part of a spectrum that includes Steven-Johnson syndrome and toxic epidermal necrolysis. Sarcoidosis most commonly affects people of African and Northern European ancestry.

Case A 58-year-old man presents to the emergency department for a 4-hour history of chest pain and shortness of breath. He is a long-haul truck driver and noted the symptoms started while he was driving. He admits a mild cough, with some blood in his sputum. The chest pain seems to be associated with breathing and gets worse with deeper inspiration. He denies fever or chills. He is a smoker. He reports no known medical conditions, no medication use, and no prior surgeries. On physical exam, the patient is mildly obese, tachypneic (respiratory rate of 22), and tachycardic (pulse of 112). He appears to be in mild distress. Lungs are normal to auscultation and percussion. Heart exam is normal. His left lower leg has some dependent edema and tenderness. The patient thinks he strained a muscle or bumped it, but he had been too worried about his shortness of breath and failed to mention in his history. The remainder of his exam is normal. He was immediately placed on oxygen at arrival. Several tests results are available. Test Result Oxygen saturation 92% on oxygen Complete blood count Normal Comprehensive metabolic panel Normal Electrocardiogram (ECG) Normal Troponin and CK-MB Normal Chest x-ray Normal D-dimer Elevated Question Assuming all modalities are available to this patient, which of the following interventions is most appropriate for his current condition?

Correct answer: Anticoagulation Explanation This patient is presenting with a likely pulmonary embolism (PE). Acute dyspnea, cough (with or without hemoptysis), chest pain, and tachycardia/tachypnea are common presenting signs/symptoms of a PE. If further testing was to be done, a CT pulmonary angiography could be done. However, with a strong clinical suspicion, the diagnosis can be made and anticoagulation, the cornerstone of PE treatment, can be initiated. There are options in prediction rules for PE. This patient would score "high" on the Wells criteria, with clinical signs/symptoms of deep venous thrombosis -3.0 points; alternative diagnosis less likely than PE - 3.0 points; heart rate >100 beats/minute -1.5 points; and hemoptysis - 1.0 points. Unfractionated heparin and low molecular weight heparin are the most common choices. The patient's tender leg should also be evaluated for a possible deep venous thrombosis, although it would be treated with the same anticoagulation as the PE. Depending on the patient's hemodynamic stability and extent of PE, fibrinolytic therapy (thrombolysis) may be considered. Chest tube decompression may be used to treat a pneumothorax. A pneumothorax may present with chest pain and dyspnea, but the D-dimer would not be elevated. The lung exam would be expected to be abnormal, and a pleural line may have been shown on the chest x-ray. Hyperbaric oxygen therapy is useful for a variety of conditions, ranging from non-healing skin wounds to carbon monoxide poisoning and air embolism. It is not a primary treatment for PE. Inferior vena cava filters can be used in the management of PE, mostly in individuals with recurrent PE, to prevent future embolism. This patient did not have a prior history of PE. The anticoagulants are first-line treatment. Percutaneous coronary intervention (PCI) is a treatment for acute ST-elevation myocardial infarction (STEMI), especially if it can be performed very rapidly after symptom onset. It does not have a role in the treatment of PE. Myocardial infarction (MI) would have initially been on the differential for a patient with chest pain and dyspnea, but the tests have helped rule MI out.

Case A 72-year-old man presents with progressive shortness of breath over the years. He denies chest pain or a history of smoking. The patient was in the construction business for many years, and before that he worked as a shipbuilder. Chest X-ray reveals marked interstitial fibrosis and calcified pleural plaques on the lateral chest wall. Question What is the most likely diagnosis?

Correct answer: Asbestosis Explanation The correct answer is asbestosis. Asbestosis is an interstitial fibrosis that occurs from lung irritation caused by exposure to asbestos; it is often seen in patients who have an extensive work history of shipbuilding and construction. Pleural calcifications or plaques are typically seen with this disease. Treatment for this disease is supportive. Silicosis, siderosis, byssinosis, and tuberculosis do not cause the calcified pleural plaques that are a typical presentation for asbestosis. Silicosis is often found in patients with occupations such as rock mining or sandblasting. Siderosis is often found in patients with occupations in mining and welding. Byssinosis is found in patients who work in textiles.

Case A 40-year-old man, a chronic alcoholic, presents with a cough that is productive of large amounts of fetid sputum. He developed the cough about 2 weeks ago, and it has gradually worsened over time. The sputum is now foul-smelling and copious. He has had high intermittent pyrexia for 4 days. His vitals are as follows: Temperature 102°F, PR 108\min, RR 24\min, BP 140\80 mm Hg. On auscultation, there is pleural rub and diminished air entry on the right side. The chest x-ray shows a large dense opacity on the right side with a fluid level. The patient is diagnosed with lung abscess. Question What statement regarding lung abscesses is true?

Correct answer: Aspiration is the most common cause Explanation A lung abscess is the result of a microbial infection causing cavities containing necrotic pulmonary tissue to develop in the lung. Aspiration of oropharyngeal material is the most common cause. It is always secondary and is due to an infection reaching the lungs from elsewhere. Predisposing factors include alcohol abuse, seizures, coma, and stroke. About 65% of cases are caused by anaerobic bacteria; the rest are caused by a mixture of both aerobic and anaerobic organisms. Symptoms include cough, foul-smelling sputum, fever, and sometimes hemoptysis. A chest x-ray can demonstrate a cavity with fluid in it. Other associated conditions, such as empyema or pulmonary infarction, are best demonstrated on CT scan. Knowledge of the underlying cause of the abscess can assist in deciding on the right antibiotic course. A sputum culture, although routinely done, is usually a contaminated specimen (by the organisms in the mouth); therefore, it is not very useful. Although improvement is expected in 3 - 5 days, therapy is generally continued for 4 - 6 weeks; the length of the therapy depends on further x-ray findings.

Question Ico-delete Highlights A 7-year-old boy presents with a 1-week history of wheezing and dyspnea on any exertion (with productive cough). On physical examination, bilateral rhonchi are heard. After a few days of treatment, spirometry is done on the patient. The findings are shown in the table. Total lung capacity (TLC) is 111% on spirometry. What is the most likely diagnosis?

Correct answer: Asthma Explanation The spirometry result in this patient shows reduction in both forced vital capacity (FVC) and forced expiratory volume in 1 second (FEV1), but FEV1 is more affected than FVC. Therefore, there is a decrease in the FEV1/FVC ratio below the predicted levels. The TLC is normal (80-120%). This concludes that this is an obstructive disorder and excludes the restrictive diseases listed. There is a marked improvement with bronchodilators. In reversible airway obstruction such as in asthma, there is a rise in the FEV1 and/or FVC by approximately 12% from pre- to post-bronchodilator testing. This reversibility is characteristic of bronchial asthma. The spirometry findings are not suggestive of restrictive lung diseases such as pneumonia, pleural effusion, kyphoscoliosis, and tuberculosis.

Case A 22-year-old man presents with a 1-week history of fever, chills, dry or mildly productive cough, and chest wall discomfort. He is on no medications, but he has a history of mild asthma for which he does not need chronic therapy. He states that he has been under a lot of pressure at work and has not been sleeping very well. He decided to seek medical attention today because he has not been improving and is concerned he might have pneumonia. On examination, vitals include a temperature of 102.6°F, BP of 126/76 mm Hg, pulse of 82 beats/min, and RR of 20/min. Lung exam reveals coarse rhonchi throughout the lung fields, with rales in the right lower lobe and egophony demonstrated over the right lower lobe. The results of a PA and lateral chest X-ray reveal right lower lobe infiltration. Question What would be the best empiric medication for this patient's illness?

Correct answer: Azithromycin 500 mg first dose, then 25 0mg daily for 4 days Explanation The correct response is azithromycin 500 mg first dose, then 250 mg daily for 4 days. Azithromycin is a macrolide that is effective against almost all causes of community-acquired pneumonia (CAP), especially Mycoplasma pneumoniae, which may be suspected in this young patient. For empiric treatment of CAP, doxycycline and macrolides are preferable in patients under 50 and those without comorbidities. Fluoroquinolones (e.g., levofloxacin) are typically indicated in patients over 50 and those with significant comorbidities. Ciprofloxacin is not indicated in the empiric treatment of CAP because it does not have broad enough coverage. Amoxicillin and cephalexin are not appropriate alternatives; they are not effective against Mycoplasma sp. Trimethoprim/sulfamethoxazole is not indicated for empiric treatment.

Case A term infant is apneic at birth. After providing warmth and positioning and clearing the airway, the infant is still apneic and has central cyanosis; his heart rate is 80 beats per minute. Question What is the next appropriate step for resuscitation of this newborn? Answer Choices Ico-marker Ico-abct 1 Stimulate the infant by flicking the soles Ico-marker Ico-abct 2 Begin positive pressure ventilation Ico-marker Ico-abct 3 Start chest compressions Ico-marker Ico-abct 4 Administer epinephrine Ico-marker Ico-abct 5 Begin positive pressure ventilation and chest compressions

Correct answer: Begin positive pressure ventilation Explanation The initial steps like providing warmth, positioning, clearing the airway, drying, removing wet linen, and evaluating the infant takes about 30 seconds. If the infant is still apneic and the heart rate is less than 100/minute, the infant's breathing should be assisted by providing positive pressure ventilation with a bag and mask. When a newborn or fetus is deprived of oxygen, there is an initial period of rapid breathing followed by a period of primary apnea during which stimulation like drying or flicking the sole will cause resumption of breathing. If oxygen deprivation continues, the infant will have gasping irregular breathing efforts and will then go into secondary apnea. At this point, stimulation will not restart the infant's breathing; therefore, positive pressure ventilation must be provided for the respiration to start. This infant will require positive pressure ventilation because he is in secondary apnea. He will not respond to stimulation by flicking the soles or free flow of oxygen. After providing PPV for 30 seconds, evaluate the heart rate. If it is below 60 beats per minute, you should support circulation by providing chest compressions (CC). Since the heart rate is 80/min, chest compressions are not required at present. Epinephrine is administered if the heart rate remains below 60/min after giving PPV with bag and mask/endotracheal tube and bag for 30 seconds, followed by PPV and CC for another 30 seconds. At present, it is not required in the above infant; however, effectiveness of PPV and chest compressions should be checked frequently. Ventilation of the lungs is the single most important and most effective step in the cardiopulmonary resuscitation of the asphyxiated newborn. Once this is accomplished, heart rate, blood pressure, and pulmonary blood flow improve spontaneously. However, if oxygen levels have become very low, cardiac output may have to be supported by chest compressions and by administering epinephrine.

Case A 53-year-old man with a 40 pack-year smoking history presents with a 10-month history of an intermittent cough with productive sputum. He admits to progressive exertional shortness of breath, which recently has limited his activity to climbing 1 flight of stairs or walking 3 city blocks. He denies diaphoresis, fever, chills, chest pain, palpitations, audible wheezing, pleurisy, peripheral edema, hemoptysis, abdominal pain, reflux, regurgitation, diarrhea, melena, or hematochezia. He also denies travel, sick contacts, and drug or alcohol use. His general survey reveals an overweight male with an odor of smoke and nicotine staining of his fingernails. His nails also demonstrate digital clubbing. His pulmonary exam reveals a prolonged expiratory phase, barrel chest, poor diaphragmatic excursion, and wheezing to auscultation. Pulmonary function testing shows airflow obstruction with a reduction in FEV1 and FEV1/FVC ratio; increases in total lung capacity, functional residual capacity, and residual volume were noted. Question What intervention has been demonstrated to influence the natural history of this patient's illness?

Correct answer: Behavioral modification - smoking cessation Explanation This patient's diagnosis is most consistent with chronic obstructive pulmonary disease. Only 3 interventions: smoking cessation, oxygen therapy in chronically hypoxemic patients, and lung volume reduction surgery in selected patients with emphysema have been demonstrated to influence the natural history of patients with COPD. All other current therapies are directed at improving symptoms and decreasing the frequency and severity of exacerbations. Diuretics are not indicated in the management of COPD. Long-acting inhaled -agonists (e.g., salmeterol) and anticholinergic agents, such as tiotropium or ipratropium bromide, have been shown to improve symptoms and reduce exacerbations; however, it has not been demonstrated that either influences the rate of decline in FEV1. Despite the frequent implication of bacterial infection in COPD exacerbations, chronic suppressive, or "rotating" antibiotics, are not beneficial in patients with COPD.

Case A 42-year-old man presents to the Emergency Department with a 3-day a history of fever with chills, cough with greenish-yellow sputum, and chest pain. He finds himself short of breath after walking 1 block. He has body aches, headache, and lack of appetite. Since this morning, he has been nauseated, and he has thrown up 3 times. His past medical history is significant for an appendectomy at age 16 and right knee arthroscopy 3 years prior to presentation. He has no drug allergies. He is not presently on any medications. Family history is significant for coronary artery disease in his father. On exam, he has a temperature of 101.1°F, pulse rate is 104/min, BP is 110/67 mm Hg, and SPO2 is 91%. Chest X-ray shows consolidation in the right lower lobe of the lung. EKG has sinus tachycardia; troponin is 0.1, BUN is 17 mg/dL, creatinine is 1.2 mg/dL, electrolytes are normal, and urinalysis is normal. Patient was further admitted as inpatient due to the severity of his symptoms. Question What regimen should you start pending blood and sputum cultures?

Correct answer: Ceftriaxone 1 g IV Q24H and azithromycin 500 mg IV Q24H Explanation The correct response is ceftriaxone 1 g IC Q24H and azithromycin 500 mg IC Q24H. The patient is possibly suffering from community-acquired pneumonia (CAP). He was admitted to the hospital because of vomiting; as a result, he would be unable to keep oral antibiotics down, so IV antibiotics are required. CAP is defined as a pneumonia that starts outside the hospital or nursing home or occurs within 48 hours of being admitted to a hospital, without any history of residence in a long-term facility for 2 weeks or more prior to the onset of symptoms. The most common causative organism is Streptococcus pneumoniae. Other organisms include Haemophilus influenzae (especially in smokers), atypical organisms (e.g., Mycoplasma pneumoniae, chlamydia, Legionella), and viruses—most commonly influenza, but occasionally parainfluenza, respiratory syncytial virus, adenovirus, etc. Staphylococcus aureus is found in patients with influenza, cystic fibrosis, bronchiectasis, nursing home residents, and drug abusers. Klebsiella pneumoniae may be isolated in alcoholics and diabetics. Anaerobic bacteria can cause pneumonia in patients with poor dental hygiene or aspiration. Pseudomonas is another bacteria found in cystic fibrosis, bronchiectasis, and institutionalized patients. A complete blood count with differential, basic metabolic panel, pre-antibiotic sputum, and blood cultures should be done in all hospitalized patients. Sputum culture is recommended in non-hospitalized patients. When sputum is not provided by a patient and culture is necessary, then transtracheal aspiration, fiberoptic bronchoscopy, and transthoracic needle aspiration may be done to obtain samples. Chest X-ray is helpful in many ways. Usual findings include lobar consolidation, patchy infiltrates, pleural effusion, cavitations, and other pulmonary abnormalities. Radiography helps in diagnosis and is also useful in assessing severity and response to treatment. Lung infiltrates may take up to 6 weeks to clear on X-ray. Pleural tap may be done in patients to rule out empyema; it may also be done for diagnostic reasons. The causative organism directs treatment. A combination of ceftriaxone and azithromycin covers gram-positive and atypical organisms. Levofloxacin IV is also a good monotherapy in CAP, but it is not recommended in order to avoid drug resistance to quinolones. It should be used only in cases where the other 2 medications are contraindicated, such as in the case of a drug allergy, but it is an invaluable antibiotic in cases of severe healthcare-acquired pneumonia (HCAP). Treatment should be continued for 7-14 days. Levofloxacin PO is not recommended in this case due to the patient vomiting. If this patient had not been vomiting and was overall stable, outpatient treatment with oral levofloxacin for 7-14 days would have been an option. It would not have been highly recommended, however, due to the danger of resistant organisms. In such a case, oral azithromycin for 7-10 days would also be a good option. Clarithromycin and doxycycline for 7-10 days may also be used. Ceftazidime and vancomycin combination is used in HCAP; its coverage includes gram-negative organisms and methicillin-resistant Staphylococcus aureus (MRSA).

Case A 30-year-old woman presents because she recently had a PPD skin test; the transverse diameter of the induration was 14mm. The patient denies ever having tuberculosis and she is asymptomatic now. However, she expresses some anxiety about the result of the skin test because, for the last 6 months, she has worked as a nurse for a home resident. Her patient is a vent-dependent tetraplegic. She remembers having the BCG vaccine in childhood. On clinical examination, there are no abnormalities. Question What is the most appropriate next step?

Correct answer: Chest X-ray Explanation Any PPD skin test should be performed and interpreted only in healthy, asymptomatic individuals, with a negative chest X-ray for active tuberculosis (TB). A positive result is considered if the transverse diameter of the induration is greater then 15 mm for low-risk individuals, 10 mm for high-risk individuals, and 5 mm for HIV-positive individuals or close contacts of patients with active TB. A health care worker is considered a high-risk individual. In this case, the next step in the management is performing a chest X-ray to look for old or active TB lesions. Patients with active TB should be placed on respiratory isolation for 2 weeks, or until 3 negative sputum results are obtained. They should follow the standard medication regimen for 6 or 9 months. Prophylaxis with isoniazid for 6, 9, or 12 months should be considered for any patient with suspicion of latent TB (positive skin test result), because the risk to develop active TB is about 10% for the next 2 years. Current indications for prophylaxis are: Any recent converters in the last 2 years from a negative skin test to a positive skin test, regardless of the patient's age Any patient with a positive skin test who had TB or has an abnormal chest X-ray with signs of old TB lesions, regardless of the patient's age Any patient younger then 35 years with a positive skin test and a normal chest X-ray Reassurance would not be appropriate before obtaining a chest X-ray that would orient the management toward a diagnostic workup; it is also not appropriate before treatment for active TB or isoniazid prophylaxis. Sputum examination is not appropriate for an asymptomatic patient

Case A 46-year-old man is a known asthmatic and has been on a carefully titrated dose of theophylline for 14 years without any complications. Recently, he sought medical attention for malaise, persistent cough, and weight loss. He was diagnosed with tuberculosis. During visits to the clinic for diagnosis and initiation of treatment for tuberculosis, he was also discovered to be hypertensive and to have acid peptic disease. He was put on medication for all 3 conditions. After 2 weeks of therapy, the patient had a seizure and was rushed to the emergency department. Reports showed a tearing of the bridging veins between the dural sinuses and an increase in the plasma theophylline concentration in the large cerebral veins. Question What drug is most likely to have caused the theophylline toxicity?

Correct answer: Cimetidine Explanation Theophylline is metabolized via the cytochrome P450 oxidative drug metabolizing system. Cimetidine is the most likely cause of the theophylline toxicity because it inhibits cytochrome P450. Theophylline's metabolism would be impaired, leading to toxic plasma levels. As the case clearly describes a supra therapeutic level of theophylline, cimetidine is the best answer as it relates to this complication. This patient is unlikely to have been put on atenolol; it is contraindicated in asthmatic patients. Hydrochlorothiazide and prazosin should not alter levels of theophylline. Rifampin is a cytochrome P450 inducer and would lower plasma levels of theophylline.

Case Ico-delete Highlights A 57-year-old man presents with progressive dyspnea on exertion and left lumbar colic. He has a history of hypertension as well as a 40 pack-year history of smoking. He denies cough, orthopnea, and paroxysmal nocturnal dyspnea. He has some mild ankle swelling, but he has no history of congestive heart failure. The only medication he is on is amlodipine. His vital signs are as follows: temperature 99.8°F, pulse 92/min, respiration 22/min, and BP 128/88 mm Hg. Of significance on physical examination is the absence of breath sounds in the left lower lung zone. Laboratory data reveals WBC 1000/μL with 70% segmented neutrophils, serum glucose 106 mg/dL, sodium 138 mmol/L, chloride 102 mmol/L, potassium 4.2 mmol/L, bicarbonate 22 mmol/L, BUN 32 mmol/L, creatinine 1.2 mmol/L, protein 8.2 g/dL, amylase 56 U/dL, and LDH 250 U/mL. Thoracentesis is done and pleural fluid analysis shows WBC 910/μL, RBC 14/μL, LDH 108 U/mL, protein 2.6 g/dL, glucose 82 mg/dL, and creatinine 1.2 mmol/L. Question What is a possible cause of this condition?

Correct answer: Cirrhosis Explanation Pleural effusion occurs because of the accumulation of excess fluid in the pleural space. Effusions may be transudative or exudative. Transudates have fluid protein less than 50% of the serum protein and LDH less than 60% of serum LDH. Exudates, on the other hand, have protein more than 3 g/dL and LDH greater than 200, or 50 and 60% of the serum values, respectively. Causes of transudative effusions include congestive heart failure, cirrhosis, nephrotic syndrome, superior vena cava obstruction, and urinothorax. Causes of exudative effusions include cancer, tuberculosis, esophageal rupture, pulmonary embolism, connective tissue disorders such as rheumatoid arthritis, pancreatitis, and pneumonias. The patient in the scenario has a transudative fluid collection. Of the options listed, cirrhosis causes a transudative fluid collection, so it is the most likely possible diagnosis. All the other options are associated with the development of an exudative fluid collection, making them less likely diagnoses.

Case Ico-delete Highlights You are evaluating a 78-year-old man who lives in a nursing home due to moderate Parkinsonism. The patient does not have any specific complaints, but nursing home staff have noticed that he does not seem as alert as usual and is spending more time in his room sleeping. He is also eating less and has dropped 2 lb in the last 2 weeks. The patient's temp is 100.8°F; pulse 100 bpm; respiration 25/min; and bp 120/70 mm Hg. On exam, he is aware of the place, but not the exact time or date. His HEENT exam is unremarkable. His cardiac exam reveals tachycardia, but no murmurs or rubs. His lung sounds are decreased at the bases. Question Based on this presentation, what should be included as part of the initial workup?

Correct answer: Complete blood count Explanation This patient is presenting with altered mental status, fatigue, weight loss, and fever, which are concerning for the presence of an underlying infection. The correct response is complete blood count. Many conditions, including infections and myocardial infarction, have subtle and atypical presentations in the elderly. The first signs are often nonspecific, such as a change in mental status and fatigue. Physicians taking care of older patients must be especially vigilant for this reason. Fever is an important warning sign, but the febrile response may be attenuated in older patients. It is often advised that any temperature over 99°F be considered elevated, warranting further workup. In addition to physical examination, the standard fever workup includes complete blood count, urinalysis and culture, and chest X-ray. Erythrocyte sedimentation rate may be included if there is concern for osteomyelitis, endocarditis, temporal arteritis, or other specific disease entity. Other specific imaging may be warranted if there is suspicion of an intra-abdominal process or because of findings on physical exam.

A 55-year-old woman presents with dyspnea, cough, and a feeling of fullness in her head. Clinical examination shows facial and neck swelling, dilated venous channels over the upper part of her body, facial flushing, and cyanosis. She was recently diagnosed with adenocarcinoma of the lung. Question Her symptoms are most probably of what nature?

Correct answer: Compressive Explanation The correct response is compressive. This patient has a clinical picture consistent with superior vena cava compression, which is not uncommon in the presence of a right upper lobe tumor. Because the superior vena cava has a thin wall coupled with a low intravascular pressure, and is surrounded by rigid structures (i.e., sternum, trachea, right bronchus, aorta, pulmonary artery, and perihilar and paratracheal lymph nodes), it is relatively easy to compress. The subsequent obstruction to flow causes an increased venous pressure which results in interstitial edema and retrograde collateral flow. An immunological reaction is not likely the cause of symptoms that are strongly suggestive of a well-defined compressive lesion. This patient has a well-defined syndrome pointing towards specific organ involvement. Constitutional symptoms (e.g., fatigue, malaise, weight loss, fever, chills, and night sweats) are nonspecific manifestations of a vast number of diseases and conditions. In the presence of a nearby primary tumor, it is unlikely that a metastatic tumor cell from a distant location would cause superior vena cava compression. Correct answer: Compressive Explanation The correct response is compressive. This patient has a clinical picture consistent with superior vena cava compression, which is not uncommon in the presence of a right upper lobe tumor. Because the superior vena cava has a thin wall coupled with a low intravascular pressure, and is surrounded by rigid structures (i.e., sternum, trachea, right bronchus, aorta, pulmonary artery, and perihilar and paratracheal lymph nodes), it is relatively easy to compress. The subsequent obstruction to flow causes an increased venous pressure which results in interstitial edema and retrograde collateral flow. An immunological reaction is not likely the cause of symptoms that are strongly suggestive of a well-defined compressive lesion. This patient has a well-defined syndrome pointing towards specific organ involvement. Constitutional symptoms (e.g., fatigue, malaise, weight loss, fever, chills, and night sweats) are nonspecific manifestations of a vast number of diseases and conditions. In the presence of a nearby primary tumor, it is unlikely that a metastatic tumor cell from a distant location would cause superior vena cava compression.

Case Ico-delete Highlights A 69-year-old man presents with shortness of breath associated with a nonproductive cough. He notes stabbing chest pain that increases with inspiration. On exam, there is dullness to percussion over the left side. EKG and cardiac enzymes are normal. Chest X-ray confirms presence of pleural effusion. Thoracentesis is performed. Pleural fluid is straw-colored on exam with protein 2 g/dL, glucose is equivalent to serum level, and total nucleated cell count is 300/mm with predominantly lymphocytes. Question What factor is most likely contributing to the patient's pleural effusion?

Correct answer: Congestive heart failure Explanation Congestive heart failure is the most common cause of transudative pleural effusion. In transudative pleural effusion, pleural fluid function is increased, but the patient has normal capillaries, causing increased hydrostatic pressure and/or decreased oncotic pressure. Thoracentesis will reveal protein less than 3 g/dL, glucose levels equivalent to serum glucose, and a total nucleated cell count level less than 500/mm with predominantly lymphocytes. Pulmonary embolism, pneumonia, pancreatitis, and malignancy are all causes of exudative pleural effusion.

Case Ico-delete Highlights A 72-year-old man presents due to worsening shortness of breath, orthopnea, and chest pain; symptoms have been occurring for the past few weeks. The patient admits to some chronic heart problems as well as fatigue, dyspnea, and a non-productive cough, but he feels like symptoms have worsened recently. He denies fever, chills, and a productive cough. On physical exam, the man has mildly increased respiratory effort, but he does not appear in distress. He is barrel-chested. His breath sounds are diminished bilaterally, with dullness to percussion over right and left lower lungs. No pleural friction rub is noted. On cardiovascular exam, an S3 gallop and mild tachycardia (110 bpm) are noted. Clubbing of the fingers, dependent edema in the lower extremities, and jugular venous distention are also noted. His cardiac enzymes and electrocardiogram demonstrate no acute cardiac pathology. Pleural fluid and cardiomegaly are found on the chest X-ray, and a thoracentesis is performed. The pleural fluid is generally clear in color, testing negative for chylomicrons and triglycerides. It has low levels of red blood cells, white blood cells, protein, and lactate dehydrogenase (LDH). Question What should be recommended as most useful for this patient as a tertiary level of preventive medicine?

Correct answer: Control of his heart failure to prevent pulmonary complications Explanation Preventive medicine is often classified into primary, secondary, and tertiary methods. Primary methods are meant to prevent disease occurrence altogether. Secondary methods are intended to detect disease at a very early stage, before complications. Tertiary methods may occur after the patient already has some significant illness, and they are aimed at preventing further complications from the disease. In this patient's case, control of his heart failure to prevent pulmonary complications is a recommended tertiary preventive medicine. This patient's chronic heart failure has caused a pleural effusion. Better control of his heart condition will prevent pulmonary complications. Administration of the influenza vaccine is a recommended primary method of preventive medicine for this patient because it prevents the illness altogether. It is recommended for almost all adults, and it is especially recommended for the patient in this case, but it is not classified as tertiary prevention. Checking a prostate-specific antigen (PSA) to detect prostate cancer at an early stage is considered a secondary level of prevention; it is aimed at detecting the illness at a very early and treatable stage. PSA screening for prostate cancer is controversial, and because this patient is much more likely to succumb to complications from his heart failure and pleural effusion than prostate cancer, it should not be recommended at this time. Daily low-dose aspirin has been often recommended as primary prevention of stroke and myocardial infarction in high-risk individuals as well as secondary prevention. While aspirin therapy may offer some mild benefit to this patient (who has a history and physical consistent with chronic congestive heart failure), it is a not a top priority. The provider should not recommend calcium supplementation to reduce risk of fractures in this patient. Calcium supplementation has recently been linked to an increased risk of cardiovascular events. Except for specific conditions, widespread calcium supplementation is not recommended. It would also be a primary prevention technique, intended to prevent bone loss and fracture before disease was present.

Case A 69-year-old man presents with dyspnea on exertion that has been slowly progressive over the course of the last year. He notes impairment in climbing stairs and walking short distances. His review of systems is positive for fatigue, palpitations, intermittent retrosternal chest pain, swelling of his lower extremities, dizziness, and "feeling faint." His associated symptoms are also known to occur upon exertion. He denies any fever, chills, weight changes, cough, abdominal pain, early satiety, nausea, vomiting, diarrhea, changes in his urine color or odor, flank pain, hematuria, or dysuria. He denies any cigarette, alcohol, or drug use. His cardiac exam is remarkable for an increased pulmonic component of the second heart sound (P2), wide, inspiratory splitting of S2 over the cardiac apex, right-sided S3 and S4 gallops, a left parasternal lift, a loud diastolic murmur that increases with inspiration and diminishes with the Valsalva maneuver, prominent "A" waves in jugular venous pulsations, and increased JVD. He has an enlarged liver with hepatojugular reflux, peripheral edema, and ascites. A bedside EKG analysis revealed peaked P waves, rightward axis deviation, and prominent R waves in the early V leads. Question What is the most likely diagnosis?

Correct answer: Cor pulmonale Explanation The most appropriate diagnosis in this scenario is cor pulmonale, often referred to as pulmonary heart disease. It occurs due to right ventricle structure and/or function alteration due to chronic lung disease, and it's usually triggered by pulmonary hypertension. Symptoms that present are due to the underlying pulmonary pathology. Dyspnea is the most common symptom, as well as abdominal pain and ascites. Signs found on physical assessment include tachypnea, elevated jugular venous pressure, hepatomegaly, and lower extremity edema. Evidence of prominent V waves in the jugular venous pulse may be found, as well as an RV heave palpable along the left sternal border or even in the epigastrium. Cyanosis is a late finding in the course of cor pulmonale. Diagnostically, ECG findings will display P pulmonale, right axis deviation, and right ventricle hypertrophy. Myocardial infarctions occur at rest and most commonly in the early morning. The pain is similar to angina in location and radiation, but it may be more severe and builds up rapidly or in waves to maximum intensity over a few minutes or longer. Associated symptoms include diaphoresis, weakness, apprehension, and a feeling of impending doom; patients may move about, seeking a position of comfort, preferring not to lie quietly. Light-headedness, syncope, dyspnea, orthopnea, cough, wheezing, nausea and vomiting, or abdominal bloating may occur. Physical exam findings may include fever, anxiousness, diaphoresis, bradycardia or tachycardia, low cardiac output, or arrhythmia. There may be hypertension (in hypertensive patients) or low in patients with shock. Respiratory distress, jugular venous distention, a Kussmaul sign, soft heart sounds, and atrial gallops (S4) or ventricular gallops (S3) usually indicate heart failure. Primary biliary cirrhosis is most common in middle-aged females and is characterized by fatigue, pruritus, hepatosplenomegaly, xanthomatous lesions on the skin, eyelids, and tendons, jaundice, and steatorrhea. Signs of portal hypertension are late findings. Other findings include orthostatic hypotension and cognitive dysfunction. Left-sided or forward failure may account for many of the clinical manifestations of heart failure, such as mental confusion from decreased cerebral perfusion, fatigue and weakness from decreased skeletal muscle perfusion, and sodium and water retention with secondary venous congestion from decreased renal perfusion. Isolated left-side heart failure is associated with dyspnea, fatigue, weakness, cough, paroxysmal nocturnal dyspnea, and orthopnea in the absence of peripheral edema, jugular venous distention (JVD), or hepatojugular reflux. Chest pain, dyspnea, and tachypnea are the most frequent signs and symptoms of pulmonary embolism. Other manifestations may include tachycardia, pleurisy, low-grade fever, apprehension, and productive cough with blood-tinged sputum. Massive PE may manifest as sudden collapse, crushing substernal chest pain, shock, diaphoresis, hypotension, distended neck veins, and loss of consciousness.

Case A 28-year-old man presents with a 2-hour history of chest tightness, coughing, and wheezing. The history determines that he has had many such attacks in recent years, usually brought on by emotional factors or exertion; the attacks are generally treatable by self-medication at home. He has a history of hay fever, and other members of the family have had similar symptoms. Physical examination reveals dyspnea, orthopnea, and cyanosis. High-pitched, sibilant rhonchi occur on inspiration and expiration, and some coarse crepitations are audible. Pulse is 130/min and regular. An emergency arterial PCO2 is 65 mm Hg. Question What will likely be shown by pulmonary function studies during an acute attack?

Correct answer: Decreased FEV1 Explanation The correct response is decreased FEV1. The patient suffers from extrinsic asthma, which is an obstructive lung disease. There is, therefore, reduction of both the vital capacity and the FEV1; there is more reduction of the latter, causing a reduction of the FEV1: FVC ratio. The total lung capacity (TLC) may, however, be increased due to air being trapped in the lung. This causes an increase in the residual volume(RV). Bronchial obstruction leads to an increase in airway resistance. In severe asthma, the patient first hyperventilates to maintain his PaO2, and as a result, he has a low PaCO2. As he tires, he hypoventilates; this causes his PaCO2 to become normal and then rise. This elevation of PaCO2 is of grave significance, as it indicates that the resting ventilation is maximal and that reduction through fatigue may be lethal.

Case A 45-year-old man presents with a 2-year history of worsening dyspnea and a 1-month history of dry cough. The patient gives no history of fever, chills, chest pain, or wheezing. History is significant for smoking (25 cigarettes/day for more than 22 years). A chest X-ray shows hyperinflated lungs with bullae, tubular heart, flattened diaphragm, and no areas of consolidation. Pulmonary function tests (PFT) reveal a decrease in forced expiratory volume in 1 second (FEV1) along with reduction of FEV1/FVC (forced vital capacity) ratio. Question These findings are characteristic of what condition?

Correct answer: Emphysema Explanation The findings described suggest that the patient is suffering from emphysema. Emphysema is a chronic obstructive airway disease; it is characterized by abnormal, permanent enlargement of the airways distal to the terminal bronchioles and destruction of their walls. Cigarette smoking is a major contributor in the development of emphysema. Other causes include air pollution, alpha-1 antitrypsin deficiency, and occupational exposure. The classical symptomatology includes prolonged progressive dyspnea with late-onset non-productive cough, occasional mucopurulent relapses, and eventual cachexia and respiratory failure. The patients are usually thin and have a barrel-shaped chest. There is tachypnea with pursed-lip breathing and use of accessory muscles; they may also adopt a tripod sitting position. A chest X-ray shows hyperinflated lungs with bullae, tubular heart, and flattened diaphragm. Forced expiratory spirometry quantifies airway obstruction. Pulmonary function tests (PFT) reveal a decrease in forced expiratory volume in 1 second (FEV1) along with reduction of FEV1/FVC (forced vital capacity) ratio. Treatment includes general measures, such as cessation of smoking, breathing exercises, symptomatic treatment like antibiotics to control secondary infection, bronchodilators, corticosteroids, and oxygen therapy. Lung volume reduction surgery and lung transplantation may be required depending on the severity. Congestive cardiac failure presents with dyspnea, cough, orthopnea, weakness, abdominal discomfort, and edema in lower portions of the body. The chest X-ray may show cardiac enlargement. There may be pulmonary congestion with audible rales. Echocardiography confirms the diagnosis. Chronic bronchitis produces obstructive changes and bronchial inflammation. Chronic bronchitis is characterized by chronic productive cough for ≥3 months in each of 2 successive years for which other causes, such as infection with Mycobacterium tuberculosis, carcinoma of the lung, or chronic heart failure, have been excluded. Chest X-ray shows increased bronchovascular markings and cardiomegaly. Bronchiectasis is a condition characterized by abnormal permanent distortion of the conducting bronchi or airways, most often secondary to an infectious process. Patients with bronchiectasis often present with chronic cough with mucopurulent sputum production that lasts months to years. CT, especially HRCT (high-resolution computed tomography), helps in confirming the diagnosis. Diffuse alveolar damage, or adult respiratory distress syndrome, is characterized by fulminant interstitial and alveolar edema that develops after an initial lung injury. It results from increased alveolar-capillary permeability that is not cardiogenic in origin. Injury can occur directly or as part of a generalized systemic acute inflammatory process.

Case A 3-year-old boy has been on antibiotics for 3 days for pneumonia. In the past 24 hours, he has developed dyspnea, fever, chills, and left-sided chest pain. On exam, he has diminished breath sounds and dullness to percussion on the left. A chest X-ray shows fluid in the pleural space. Question What is the most likely diagnosis?

Correct answer: Empyema Explanation A patient with a pleural effusion and signs of worsening pneumonia is at high risk of having an empyema. Patients with empyema often appear ill with fever, tachycardia, dyspnea, and pleuritic chest pain. Empyema is an infection and pus in the pleural space; it is most commonly caused by pneumonia due to Streptococcus pneumoniae. H. influenzae, GAS, gram-negative organisms, tuberculosis, fungi, and malignancy are less common causes. Empyema may also be caused by the rupture of a lung abscess into the pleural space, trauma, or thoracic surgery. Thoracentesis should be performed and a culture sent. If pus is obtained, a chest tube is left in for drainage. Appropriate antibiotic therapy should be instituted.

Case A 3-year-old child presents with sudden onset of high-grade fever, sore throat, pain during swallowing, and drooling of saliva. There is no history of a cough. The child appears toxic and is dyspneic with inspiratory stridor. The child is sitting upright, leaning forward with chin up and mouth open. Suprasternal and intercostal retractions are present. Chest is clinically clear. Blood count shows polymorphonuclear leukocytosis. Lateral radiograph of upper airway shows "thumb sign". Question Ico-delete Highlights What is the most likely diagnosis?

Correct answer: Epiglottitis Explanation The most likely diagnosis is acute epiglottitis, as the child has presented with sudden high fever, sore throat, dysphagia, drooling, and dyspnea. Cough is usually absent in epiglottitis. The diagnosis is further supported by the presence of polymorphonuclear leukocytosis and lateral radiograph of the upper airway showing the "thumb sign," which is due to swelling and inflammation of the epiglottis and other supraglottic structures, especially the aryepiglottic folds. Haemophilus influenzae type b is the most common cause of acute epiglottitis. Stridor is a late finding and suggests severe airway obstruction. Rapid airway management is essential and includes nasotracheal intubation and oxygenation. Examination of the throat with a tongue depressor should be avoided; it can precipitate a fatal laryngospasm. Antibiotics commonly used for treatment are cefotaxime, ceftriaxone, or meropenem pending culture and sensitivity reports. Laryngotracheobronchitis (Croup) usually starts with rhinorrhea, cough, and low-grade fever. After 2-3 days, the child develops the characteristic "barking" cough, hoarseness of voice, and a biphasic stridor. In epiglottitis, the stridor is inspiratory. Cough is usually absent in epiglottitis, as seen in the above patient. Temperature ranges from 102 degrees F to 104 degrees F. Croup is most commonly caused by viruses (influenza A and B, adenovirus, and respiratory syncytial virus). Throat examination shows moderately inflamed pharynx, rhinitis, and mild tachypnea with subcostal and intercostal retractions. Epiglottitis has a more acute and rapid course than croup. X-ray chest shows the characteristic narrowing of the subglottic region and is known as the "steeple sign." Bacterial tracheitis is more commonly caused by Staphylococcus aureus. It often follows a viral upper respiratory tract illness. Mean age is between 5 and 7 years. After a few days, the child develops a brassy cough, biphasic stridor, high fever, and toxicity; unlike epiglottitis, the child can lie flat, does not drool, and does not have dysphagia. Bacterial tracheitis is a life-threatening emergency, as respiratory obstruction may develop due to mucosal edema and thick purulent secretions, which can be adherent and membranous. Bronchoscopy is both diagnostic and therapeutic because it shows the purulent secretion that can be mechanically debrided. Retropharyngeal abscess occurs as a complication of bacterial infection of the upper respiratory tract and commonly occurs in children between 3 and 4 years of age. It occurs due to spread of infection from the upper respiratory tract to the retropharyngeal lymph nodes, which suppurate and form an abscess. It may present with fever, reduced mobility of the neck, stridor, dysphagia, and drooling. Child may have respiratory distress or obstructive sleep apnea. Physical examination can reveal bulging of the posterior pharyngeal wall. Cervical lymphadenopathy may also be present. Lateral neck radiograph shows a soft tissue bulge in the posterior pharyngeal wall. Laryngomalacia is the most common congenital anomaly of the larynx. The infant commonly presents with inspiratory stridor, which is aggravated when the infant is crying, feeding, or is placed in supine position. It typically starts at 4 - 6 weeks of age. The infant is otherwise usually happy and thriving. Stridor is due to partial collapse of a flaccid supraglottic airway. Condition is self-limiting and resolves by 18 months of age.

Case A 23-year-old woman presents with an anaphylactic reaction after being stung by several bees. She complains of wheezing and shortness of breath. On examination, the patient is in acute distress. BP is 98/56 mm Hg, PR 110/min, RR 28/min, and temperature 98.7°F. She is immediately treated with supplemental oxygen. Question In treating this condition further, what drug is required most urgently?

Correct answer: Epinephrine Explanation The correct answer is epinephrine, as this is the drug of choice in anaphylactic reactions. The definite indications are hypotension, bronchospasm, and urticaria. Epinephrine has significant beta-agonistic effects that cause bronchodilation, and alpha-agonistic effects that can reverse systemic hypotension. IM route is preferred over SC, as the time for maximum serum concentration is shorter by IM route. Patients with a previous history of similar insect bite or asthma must be given epinephrine. Side effects such as arrhythmias and angle-closure glaucoma can occur. Self-injectable preparations are available that can also be used to renew prescriptions; however, fluid replacement (especially when the patient is in shock) and airway maintenance are also important. Diphenhydramine is an antihistaminic and H1 blocker that also must be used in all bee sting cases. It affords symptomatic relief from pruritus. Glucagon is required only if the patient is on beta-blockers. In such patients, it exerts its effects by inotropic and chronotropic properties, which are independent of beta-receptors. Theophylline may be required in resistant bronchospasm, but it is not first-line therapy for anaphylaxis. Cimetidine is an H2 blocker, which may have an additive effect with H1 blockers in anaphylaxis. Methylprednisolone, a corticosteroid, is used in all generalized reactions for its anti-inflammatory property.

Question A 16-year-old boy presents with a dry cough of 1 week duration, pharyngitis, and malaise. Examination shows a low-grade fever (38.5° C), and chest auscultation reveals sibilant rhonchi. Chest films show a patchy bronchopneumonia. What is the best course of treatment for this clinical picture?

Correct answer: Erythromycin Explanation The correct response is erythromycin. A clinical syndrome that includes low-grade fever, malaise, and nonproductive cough is characteristicof atypical pneumonia. The highest prevalence of infection is in the young, with children older than 5 years and young adults at most risk. Atypical pneumonia is most often caused by infection with Mycoplasma pneumoniae, Chlamydia pneumoniae, and Legionella pneumophila. Outpatients without coexisting illness, or patients over 60 years of age, are unlikely to have pneumonia caused by these 3 organisms. Macrolide antimicrobials provide suitable empiric therapy for atypical pneumonia. Erythromycin and tetracycline have been shown in clinical trials to be efficacious for the treatment of pneumonia caused by M. pneumoniae. Fluoroquinolones have been shown effective in M. pneumoniae infections. β-Lactams are ineffective therapies because M. pneumoniae lack a cell wall. C. pneumoniae should be suspected in atypical pneumonia, particularly when accompanied by pharyngitis. Large, controlled trials comparing antimicrobial agents have not been conducted, but erythromycin and other macrolides, tetracycline, and fluoroquinolones all appear effective. L. pneumophila causes 1% to 15% of atypical pneumonia cases. The presentation of pneumonia caused by this agent is varied and nonspecific. Determination of L. pneumophila susceptibility to antimicrobials has been hindered by a lack of suitable assays for intracellular pathogens. Erythromycin, which has been the drug of choice for treatment of atypical pneumonia caused by L. pneumophila, has limited effectiveness in some cases. Azithromycin and the quinolones, with superior intracellular levels, have proven effective by experience. The efficacy of azithromycin, erythromycin, and quinolones has not been compared in clinical trials of L. pneumophila pneumonia. Streptomycin, not part of routine antimicrobial therapy for pneumonia, is part of combined therapy of tuberculosis; it may be used in combination with other agents for treatment of endocarditis, plague, tularemia, and brucellosis. Trimethoprim-sulfamethoxazole, sometimes used to treat acute exacerbations of bronchitis, is not effective against these organisms. Like amoxicillin, the cephalosporins have no activity against Mycoplasma.

Case Ico-delete Highlights An immigrant worker presents with a chronic cough, blood-stained sputum, and night sweats. Physical examination reveals a wasted middle-aged man with bronchial breathings on the right upper lobe. His sputum examination under microscope shows acid-fast staining rods, and his PPD is 15 mm. Based on clinical findings, sputum examination, and PPD result, pulmonary tuberculosis is diagnosed. Treatment is started after sending the sputum for culture. The patient comes back 4 weeks later for a check up, and he mentions not being able to see clearly and not being able to distinguish the color blue from green. Question What drug is most likely causing this side effect?

Correct answer: Ethambutol Explanation The correct response is ethambutol. Important side effects of TB drugs are: Ethambutol: optic neuritis and difficulty distinguishing blue from green Isoniazid: hepatitis, and peripheral neuropathy (pyridoxine is the treatment for peripheral neuropathy) Rifampicin: hepatotoxic, hypersensitivity reactions, and red coloring of the urine and other secretions (saliva, tears, stool) Pyrazinamide: hepatotoxic (rare), and hyperuricemia Streptomycin: ototoxic, and nephrotoxic.

Case A 24-year-old man undergoes a routine medical check-up to become a volunteer in the ER. PPD skin test shows an induration diameter of 9 mm. Sputum and chest X-ray are done; they are negative for tuberculosis. He is otherwise healthy and has no fever, cough, or other symptoms of Mycobacterium tuberculosis infection. Question What is the most appropriate explanation for the doubtful tuberculin test in this patient?

Correct answer: He was given BCG vaccination earlier in life. Explanation The patient was given BCG vaccination earlier in life. The Mantoux tuberculin skin test (TST) is performed to determine whether the person is infected with Mycobacterium tuberculosis. The TST is performed by injecting 0.1 mL of tuberculin purified protein derivative (PPD) into the inner surface of the forearm. Skin test results should be read 48-72 hours after administration of the PPD. Generally, in immunocompetent individuals, any palpable induration ≥10 mm is considered a positive reaction. In the case of tuberculosis suspects or close contacts of individuals with tuberculosis, an induration ≥5 mm should be interpreted as a positive reaction. An induration measuring 5-9 mm is considered a doubtful reaction. In case of doubtful reaction, the possibility of skin sensitivity due to previous immunization or atypical mycobacterium exposure should be considered. Though prior BCG vaccination increases the risk of a reactive PPD, this effect is known to be inconsistent. Studies have shown that reactions >10 mm should not be attributed to prior BCG vaccination. Induration ≥5 mm should be considered positive in HIV-positive patients, patients with known contact with active TB, patients with a history of organ transplant on immunosuppressive therapy, and patients with fibrotic lesions on chest X-ray. Positive tuberculin test indicates exposure of the immune system to tuberculous protein, either in the form of BCG vaccine, an active tuberculous infection, or a chronic tuberculous infection. Proper vaccination history could have unmasked the cause of the indeterminant reaction to tuberculin test in this patient. Other causes, such as a wrong sputum and chest X-ray report, inadequate sputum samples, chronic cavitary lesion, or active tubercular infection, are not the cause of doubtful reaction in this patient.

Case A 58-year-old woman with a past medical history of hypertension, hyperlipidemia, breast cancer, hip fractures, and coronary artery disease is being evaluated for acute-onset, severe left-sided pleuritic chest pain over the course of the last 2 hours. The pain is associated with feelings of anxiety, hemoptysis, shortness of breath, and nausea. She "feels warm", but denies chills, palpitations, wheezing, edema, vomiting, abdominal pain, abnormal bowel habits, or dietary intolerances. She admits to a 30 pack/year smoking history, but denies drug or alcohol use. Upon physical exam, she is found to be febrile, hypotensive, tachycardic, tachypnic, diaphoretic, and in acute painful distress. There is perioral cyanosis and a pleural friction rub to the left lung fields; the remainder of the exam is normal. Question What is the most appropriate therapeutic intervention for this patient at this time?

Correct answer: Heparin Explanation This patient's presentation is significant for a pulmonary embolism. Predisposing underlying conditions are almost always present; venous thrombosis may result from a generalized hypercoagulable state, venous endothelial injury, or local stasis (Virchow triad). Most commonly, the initial manifestations of pulmonary embolism include an abrupt dyspnea and chest pain. Tachycardia and hypoxia are the most common clinical signs. Associated manifestations include fever, hypotension, cyanosis, pleural friction rub, and findings consistent with pulmonary consolidation. Anticoagulation is the foundation for successful treatment of DVT and PE. Unfractionated heparin, low-molecular-weight heparin, or fondaparinux all achieve effective anticoagulation immediately.

Case A 40-year-old man presents in January with a 4-day history of fevers, chills, myalgias, headache, productive cough, and mild sinus congestion. He has no significant past medical history. His physical examination reveals a temperature of 101.7°F, pulse 96/min, respiratory rate 20/min, and blood pressure of 128/80 mm Hg. There is mild maxillary sinus tenderness. His oral cavity and oropharynx are clear. His tympanic membranes are pearly gray, with normal light reflex. His chest is clear to auscultation; cardiac exam is unremarkable. Question What is the best course of therapy for this patient?

Correct answer: Recommend bed rest, analgesics, and nasal decongestants. Explanation The best course of treatment is supportive and symptomatic care with hydration, analgesics, and decongestants. This patient presents with classic symptoms of an influenza infection. He has no evidence of a severe bacterial infection, so antibiotics are not indicated. It is important to note that the flu can cause sinus congestion as well as rhinorrhea. Although amantadine and zanamivir are approved for treatment of influenza infections, they have only been found to be helpful if treatment is begun within 48 hours of the initial symptoms. If prescribed within this period, the duration of illness can be shortened by about 1 day.

Case You have just taken over the management of a 55-year-old man with COPD who was admitted 3 days earlier for community-acquired pneumonia. He currently feels somewhat better, and he has been afebrile for the last 24 hours. Leukocytes count 5,600/μL serum glucose 106 mg/dL Segmented neutrophils 75% sodium 138 mmol/L Hemoglobin 19g/dL chloride 102 mmol/L Platelets 245,000/μL potassium 4.2 mmol/L Arterial blood gas PH 7.25 bicarbonate 29 mmol/L PCO2 55 BUN 18 mmol/L PO2 57 creatinine 1.0 mmol/L HCO3 29 O2 sat 88% Question What has been shown to improve life expectancy in a patient like this?

Correct answer: Home oxygen Explanation The only therapy that has been shown to decrease mortality and morbidity in COPD patients with chronic hypoxemia is oxygen therapy. It is indicated in patients with COPD with PO2 <55 or O2 saturation <88% or PO2 56-59 and O2 saturation <90% if there is evidence of cor pulmonale, signs of congestive heart failure, or hematocrit ≥56. The patient has COPD and a PO2 57. Given the clinical scenario, he is a candidate for home O2 therapy. Antibiotics are only used in acute COPD exacerbations to treat superimposed infections, especially when there is a change in sputum quantity or color. Their long-term use has not been shown to improve mortality or morbidity. Likewise, inhaled glucocorticoids and bronchodilators, such as β-agonist and theophylline, may provide symptomatic relief but do not improve life expectancy. Acetazolamide is a diuretic that is not indicated in the management of COPD.

Question A 3-year-old girl presents with a rapid onset of high fever and noisy breathing that developed in the last 8 hours. The patient has missed standard immunizations. The child denies having a sore throat, but she tells you that she cannot eat or drink anything because of pain; when she tells you this, she points with her finger to a spot deep inside her mouth. On examination, the child has a muffled voice, drooling, fever, tachycardia, tachypnea, and she adopts a leaning forward position with the neck extended. Stridor and retractions of the chest wall are noted on inspirations, but the lungs are clear on auscultation. During the examination, the child becomes cyanotic and prostrated. What is the most appropriate next step in patient management?

Correct answer: Immediate intubation Explanation Epiglottitis, an infection most commonly caused by Haemophilus influenzae, is characterized by inflammation and rapidly progressive edema of epiglottis and contiguous tissue. Children of ages 2 to 7 years who missed some immunizations are prone to this infection during winter. The use of H. Influenzae vaccine caused a marked decrease in the incidence of this disease. Drooling, hoarseness, high fever, sore throat, the characteristic "sniffing dog" position and the rapidly progressive respiratory obstruction make the diagnosis a clinical one. Because of the rapid evolution to airway occlusion, a patient with suspicion of epiglottitis should receive immediate intubation under anesthesia in the operating room. Emergency cricothyrotomy is the alternative if there are no conditions to ensure a patent endotracheal airway. Intravenous ceftriaxone or cefuroxime provide an empiric antibiotic coverage. Lateral neck X-ray or other radiological examinations for a patient with epiglottitis in respiratory distress are considered a delay in the management. The classic "thumbprint sign" indicates obstruction of the airway by the swollen epiglottis. Throat examination is contraindicated in epiglottitis because it precipitates a laryngospasm. A cherry-red, swollen epiglottis on direct visualization during the intubation confirms the diagnosis. Aerosol ribavirin is used for patients with respiratory syncytial virus bronchiolitis, a severe infection in infants that can also progress to respiratory failure. Patients experience wheezing, rales, prolonged expiratory phase, rhinorrhea, nasal congestion, and respiratory distress. Antigen testing from nasal secretions provides a rapid diagnosis. Polyclonal and monoclonal antibodies are used for prophylaxis in wintertime. Nebulized racemic epinephrine is indicated in croup, an upper respiratory infection caused by parainfluenza virus. Nebulized racemic epinephrine has no effect in epiglottitis.

Case Ico-delete Highlights A 22-year-old man presents with a 3-day history of sharp, localized, intermittent back pain on the right side. The patient denies any physical trauma to the area. He states the pain intensifies with deep breathing, sneezing, and coughing. He also reports he is experiencing a concurrent viral respiratory illness, for which he has not sought treatment. He denies any PMH other than typical childhood illnesses. His vitals are: BP 125/76 mm Hg, pulse 85/min, temperature 97.8° F, respiration rate 16/min, height 67 inches, and weight 170 lb. Lung exam reveals occasional coarse rhonchi throughout all lung fields, without wheezes or rales. There is no increase or decrease in tactile fremitus in any lung fields, egophony is not present, and diaphragmatic excursion is equal bilaterally. Localized tenderness in the right back at the level of ribs 7, 8, and 9 is present with deep breaths and coughing, but tenderness is not elicited with palpation of the area. Skin exam reveals no rashes or other abnormal findings. The remainder of the physical examination does not demonstrate any other abnormal findings. Question Based on the most likely diagnosis for this patient, what treatment regimen would be most appropriate?

Correct answer: Indomethacin 25mg twice daily with a cough suppressant for 1 week Explanation Based on the symptomatology and physical exam findings, the most likely diagnosis is pleuritis. Pleurisy is characterized by an irritation of the parietal pleura; it causes an inflammatory condition that results in sharp, intermittent pain. It may be caused by viral or bacterial respiratory infections or fractured ribs; treatment consists of anti-inflammatory drugs and a cough suppressant. The other choices are not treatments for the patient's diagnosis of pleuritis. Azithromycin is an antibiotic used for bacterial infections, albuterol is used for patients with asthma and other reactive airway diseases, and valacyclovir is used for the treatment of herpes zoster. Diazepam is a benzodiazepine used as treatment for anxiety.

Case Ico-delete Highlights A 57-year-old man, who is well known to your practice, presents with a 6-month history of a daily productive cough. The patient is a nonsmoker, and he has worked in a local coal mine for the past 39 years. He very rarely comes in to see a healthcare provider. When asked if he is up to date with his vaccinations, he does not recall the last ones he received; he also does not recall when these may have been given. Question Given the most likely diagnosis at this time, what vaccinations would be recommended to help decrease any significant morbidity and mortality in this patient?

Correct answer: Influenzae and pneumococci Explanation The correct response is influenzae and pneumococci. It is very likely that the patient described above has an occupation pulmonary disease process, most likely coal worker's pneumoconiosis, or black lung disease. Most patients that develop this disease tend to have worked around coal dust for several years, if not decades. For this reason it is commonly diagnosed in patients who are over the age of 50. Smoking does not increase the risk of developing pneumoconiosis. The chronic ingestion of inhaled coal dust by macrophages in the alveoli leads to the formation of coal macules, usually 2 - 5 mm in diameter. These areas then appear on a chest radiograph as opacities, especially in the upper lung. Simple coal work's pneumoconiosis commonly is asymptomatic; at times a cough or even varying degrees of shortness of breath may be present. The patient will typically not have any significant abnormal pulmonary function results. More severe cases of this pathology are usually referred to as progressive massive fibrosis. There is no cure for coal worker's pneumoconiosis; therefore, prevention of excessive and long-term exposure is necessary. Once a patient has developed symptoms of this disease, such as the patient above, the healthcare provider must ensure the patient is up to date with all potential vaccinations to avoid any pulmonary co-morbidities. This should be viewed as a critical component of health maintenance. The influenzae and the Streptococcivaccines would be crucial for this patient to receive; consequences of these infections could be extremely dangerous. A zoster vaccination may be considered, but would not necessary be the best answer. This patient receiving a varicella vaccine would not be recommended at this time.

Case Ico-delete Highlights A 32-year-old man presents due to occasional shortness of breath and associated cough, especially when he is working outside. He has associated chest tightness that resolves within minutes when he sits down and rests. These symptoms occur 1-2 days a month. He is otherwise healthy and does not smoke. Blood pressure is 128/74 mm Hg, and pulse is 76, respiration is 14, and pulse oximetry is 100% on room air. His FEV1 is 96%. Question What is the treatment of choice?

Correct answer: Inhaled short-acting beta agonist Explanation This patient has intermittent asthma, which is defined as symptoms ≤2 days per week, nighttime awakenings ≤2 times a month, and no interference of normal activities with a normal FEV1 between exacerbations. The treatment of choice is inhaled short-acting beta agonist (SABA), albuterol, when needed. If the asthma occurs due to an allergen or in a predictable manner, the patient can use the SABA prior to exposure to avoid triggers as much as possible. A patient with persistent asthma requires treatment with an inhaled corticosteroid as a controller medication. The dose (low, medium, or high) depends on the severity of the asthma and the level of asthma control. A long-acting beta agonist may be added for controller medication. The patient would continue to use a SABA as needed for acute symptoms. Inhaled anticholinergic agents are used to treat chronic obstructive disease (COPD), but may be used as an adjunctive therapy to SABA for acute asthma exacerbations. Oral beta agonists have a delayed therapeutic response and are not useful for acute attacks. Inhaled SABA is more effective.

Case A 68-year-old man with a history of cirrhosis presents due to weight gain, increased girth, and shortness of breath. He denies fever or chills, cough, melena, hematemesis, hemoptysis, and confusion. He takes furosemide on a daily basis, and he has not missed any doses. On physical exam, his blood pressure is 120/80; pulse is 78, and respiratory rate is 18. He is alert and cooperative. Heart exam: regular rate and rhythm without murmurs. Lung exam: decreased breath sounds on right side. Abdominal exam: positive fluid wave, no tenderness to palpation. There is no hepatosplenomegaly appreciated. Chest X-ray shows a right-sided pleural effusion; it is free-flowing, as is evidenced on a decubitus film. Question What is the etiology of his pleural effusion?

Correct answer: Reduced intravascular oncotic pressure Explanation Pleural effusion, an abnormal collection of fluid in the pleural space, is related to an underlying pathology causing excess fluid production or decreased absorption. Effusions are classified as transudative or exudative depending on the characteristics of the fluid. Patients with liver disease/cirrhosis have decreased protein and albumin production, leading to decreased oncotic pressure of the intravascular fluid. This contributes to ascites and dependent edema. Peritoneal ascites may travel across the diaphragm and lead to pleural effusions (hepatic hydrothorax), most commonly on the right side. Pleural effusions caused by decreased oncotic pressure are transudative. Treatment is aimed at reducing the ascitic fluid using diuretics (loops and/or spironolactone). Severe cases may require treatment with a transjugular intrahepatic portosystemic shunt (TIPS) or liver transplant. Other causes of hypoalbuminemia, such as nephrotic syndrome, also cause pleural effusions due to reduced intravascular pressure. Altered permeability of the pleural membrane causing a pleural effusion is seen with localized inflammation, malignancy, or pulmonary embolism. Decreased lymphatic drainage is commonly seen in malignancy (lung, breast, or lymphoma). The effusion is exudative and almost always unilateral on the side with the associated pathology. Pleural effusions caused by congestive heart failure are due to increased hydrostatic pressure. They are usually bilateral and transudative. Reduced pressure in the pleural space prevents full lung expansion, allowing pleural fluid accumulation. Causes of this include atelectasis (transudative) or mesothelioma (exudative).

Case Ico-delete Highlights A 30-year-old African American man presents with a 2-month history of coughing and a 2-day history of coughing blood. He also mentions that he is losing weight and sweating at night. On physical examination, the patient is wasted, tachypneic, has bronchial breathing sounds on the right upper lobe, and there are crepitations on the left upper lobe and right mid zone. Refer to the image. Question What on this X-ray is helpful for diagnosis?

Correct answer: Left upper lobe cavitations and infiltrations Explanation This patient is presenting with typical pulmonary tuberculosis; symptoms include long-term cough, weight loss, night sweats, and sometimes hemoptysis. Physical examination also shows that he is wasted, with bronchial breathing on left upper lobe. In addition, tuberculosis in the United States is more common in the African American, Native American, and Eskimo populations, among socioeconomically disadvantaged people, and immigrants from underdeveloped countries. Typically, pulmonary tuberculosis involves upper lobes in adults. Mycobacteria are aerobic bacilli, and they grow better on more oxygenated upper lobes (apices) of the lungs. Infiltrations and cavitations on the upper lobes are typical X-ray findings on adult pulmonary tuberculosis. Hilar glands can be seen on pulmonary Tb X-ray; in and of themselves, however, they are not helpful for the diagnosis. They can be seen in the other conditions like sarcoidosis, lymphoma, and some other lung tumors. Middle and basal infiltrations also can be seen with pulmonary Tb, but they are not diagnostic; they can be seen with many other pulmonary infections. This X-ray has a nice air bronchogram on the right middle lobe, which is at the right side of the heart border. Air bronchogram is a sign of infiltration in the interstitial lung tissue, but it is not typical for pulmonary Tb. It can be seen with any other kind of infiltration.

Case You are evaluating a 12-year-old boy with cystic fibrosis (CF) at a periodic evaluation visit. Weight gain has been stable, but he has had 3 pulmonary exacerbations in the past year. His participation in physical activity has decreased during that time because his parents believe that exercise will be detrimental to him in his weakened condition; however, he has many friends who participate in athletics and physical activity and he would like to join them. Past medical history is otherwise unremarkable except for occasional episodes of sinusitis. Vital signs reveal a respiratory rate of 20 at rest, with scattered crackles and wheezes at both lung bases. Pulse oximetry is 93% at rest. Question What advice would you give?

Correct answer: Long-term, regular exercise training should be initiated. Explanation It is now generally accepted that regular physical activity should be an integral component of the treatment plan for individuals with cystic fibrosis (CF). Benefits include improved clearance of pulmonary secretions, slower decline of pulmonary function (e.g., of forced vital capacity), and better glycemic control and bone mineralization. A regular, long-term exercise program can maintain fitness even in the face of a decline in pulmonary function. The degree of exercise should be tailored to the tolerance and pulmonary function of the individual patient. Muscle strengthening, including the muscles of the chest wall, should be highlighted. Limitation of maximal exercise tolerance by symptoms does not appear to be a factor in mild to moderate pulmonary disease; it is only a factor when severe disease is present. Exercise capacity can be increased by exercise training in CF patients even with severe disease. CF itself is not a contraindication to competitive sports. The degree to which an individual with CF will be able to compete depends upon the nature of the sport and pulmonary function, among other factors. As indicated above, there are numerous benefits of physical activity and exercise training in patients with CF. Even with severe pulmonary disease, physical activity can be of benefit in CF. Parental encouragement of and parental participation in physical activity is important in initiating and maintaining a physical activity program in young CF patients.

Case A 69-year-old man presents with a 7 - 10 day history of increasing dyspnea and inspiratory chest pain. He has a 2-pack-per-day smoking history and abuses alcohol. He has hypertension, diabetes, coronary artery disease, and chronic kidney disease. He was discharged from the hospital 2-1/2 weeks ago; he was diagnosed with congestive heart failure after presenting with similar symptoms. The patient's breathing appears labored; there are diminished breath sounds on auscultation and dullness to percussion in the lower 1/2 of the lung fields, bilaterally. A chest X-ray reveals bilateral moderate pleural effusions. Thoracentesis reveals pleural fluid with the following characteristics: (1) turbid in appearance; (2) 2500 white blood cells/microliter; (3) glucose equal to serum levels; (4) ratio of pleural fluid protein to serum protein of 0.75; and (5) ratio of pleural fluid LDH to serum LDH of 0.72. Question Based on the information above, what is the most likely etiology of the pleural effusions?

Correct answer: Malignancy Explanation The correct response is malignancy. A pleural effusion is an abnormal accumulation of fluid in the pleural space. Patients with pleural effusions most often report dyspnea, cough, and pleuritic chest pain. Large pleural effusions are more likely to be symptomatic than smaller effusions. Physical findings are usually absent in small effusions. Larger effusions may present with dullness to percussion and diminished or absent breath sounds over the affected area. A diagnostic thoracentesis should be performed whenever there is a new pleural effusion and no clinically apparent cause, when there is an atypical presentation, or when an effusion fails to resolve as expected. Sampling allows visualization of the fluid in addition to chemical and microbiologic analyses to help identify the etiology of the effusion. The differential diagnosis of bilateral pleural effusions includes any cause of a transudative effusion. The most common of these is cardiac, renal or liver failure, and hypothyroidism. It also includes certain causes of exudative effusions, such as malignant disease, pneumonia, and tuberculosis. Pleural samples should be sent for measurement of protein, glucose, and LDH, in addition to total and differential white blood cell counts. These chemistry tests are used to classify effusions as a transudate or an exudate. This distinction is important because the differential diagnosis for each is entirely different. As defined by Light's Criteria, a pleural effusion is an exudate that has 1 or more of the following laboratory features: Ratio of pleural fluid protein to serum protein >0.5; Ratio of pleural fluid LDH to serum LDH >0.6; and Pleural fluid LDH >2/3 the upper limit of normal serum LDH. Additionally, samples with low glucose levels usually indicate a bacterial or significant inflammatory etiology. In contrast, transudative effusions have none of these features. Transudates are also distinguished by fewer than 1000 white blood cells/microliter and a pleural glucose level equal to serum. These types of effusions occur in the setting of normal capillary integrity and also suggest the absence of local pleural disease. The characteristics of the pleural fluid from this patient indicate an exudative pleural effusion. Malignancy is the most likely etiology in this patient, as a bacterial cause would typically present with a decreased glucose level in the pleural fluid compared with the serum; there is an equal glucose level in this case. Congestive heart failure accounts for >90% of transudative pleural effusions. Nephrotic syndrome and cirrhosis with ascites can also lead to transudative pleural effusions. Bacterial pneumonia and cancer are the most common causes of exudative effusions.

Case You receive notice that one of your patients, a 2-month-old boy, was found dead in his parents' bed. He was just seen for his 2-month well-baby check and seemed healthy and thriving, although he was recovering from a mild upper respiratory infection. His mother recently returned to work and the infant was being cared for by a nanny during the day. You remember counseling about "back to sleep" care and other risks for this condition. Although she is a smoker, she had cut back during her pregnancy and said she now only smokes outside. Question What is a risk factor for this condition?

Correct answer: Maternal smoking during pregnancy Explanation Sudden infant death syndrome (SIDS) is defined as the sudden death of an infant under the age of 1 that is unexplained by thorough case investigation, autopsy, and review of case history. The occurrence peaks between 2-3 months of age. Since 1994, when the Back to Sleep campaign began recommending placing infants in a supine position, there has been a consistent decrease in SIDS rate. SIDS is still responsible, however, for more infant deaths in the United States than any other cause of death during infancy beyond the neonatal period. The predominant hypothesis concerning the etiology remains that certain infants may have a maldevelopment or delay in maturation of the brainstem neural network responsible for arousal, which affects the physiologic response to life-threatening challenges during sleep. Independent risk factors for SIDS that have been consistently identified across studies are prone sleep position, lower socioeconomic class, overheating, late or no prenatal care, young maternal age, preterm birth and/or low birth weight, male sex, and maternal smoking during pregnancy. Breastfeeding is considered a protective factor against SIDS.

You receive notice that one of your patients, a 2-month-old boy, was found dead in his parents' bed. He was just seen for his 2-month well-baby check and seemed healthy and thriving, although he was recovering from a mild upper respiratory infection. His mother recently returned to work and the infant was being cared for by a nanny during the day. You remember counseling about "back to sleep" care and other risks for this condition. Although she is a smoker, she had cut back during her pregnancy and said she now only smokes outside. Question What is a risk factor for this condition?

Correct answer: Maternal smoking during pregnancy Explanation Sudden infant death syndrome (SIDS) is defined as the sudden death of an infant under the age of 1 that is unexplained by thorough case investigation, autopsy, and review of case history. The occurrence peaks between 2-3 months of age. Since 1994, when the Back to Sleep campaign began recommending placing infants in a supine position, there has been a consistent decrease in SIDS rate. SIDS is still responsible, however, for more infant deaths in the United States than any other cause of death during infancy beyond the neonatal period. The predominant hypothesis concerning the etiology remains that certain infants may have a maldevelopment or delay in maturation of the brainstem neural network responsible for arousal, which affects the physiologic response to life-threatening challenges during sleep. Independent risk factors for SIDS that have been consistently identified across studies are prone sleep position, lower socioeconomic class, overheating, late or no prenatal care, young maternal age, preterm birth and/or low birth weight, male sex, and maternal smoking during pregnancy. Breastfeeding is considered a protective factor against SIDS.

Case A 50-year-old woman presents with right-sided pleural effusion. Thoracentesis shows the presence of exudative serosanguineous pleural fluid and positive cytology. Question For what condition is this finding most typical?

Correct answer: Metastatic infiltrating ductal carcinoma Explanation Metastatic infiltrating ductal carcinoma is the correct response. A blood-tinged effusion in anyone past the age of 40 should put neoplasia into the differential diagnosis. Neoplastic pleural invasion usually involves both the visceral and the parietal surfaces and can cause a bloody pleural effusion. Breast cancers often metastasize to the lung and pleura. Carcinomas of the lung and breast are the most common primary sites from which pleural metastases arise. Stomach and ovarian carcinomas are next in frequency for the development of malignant pleural effusion. Spread of malignancy to the pleura from breast, stomach, and ovarian cancers usually occur indirectly from hepatic metastases. However, contiguous spread through the chest wall in breast cancer, and through the diaphragm in ovarian or stomach cancer, can occur occasionally. When malignancy is suspected, cytological examination of the fluid helps in establishing the diagnosis. Pleural biopsy, or pleural fluid immunocytochemistry, is used as an adjunct to cell morphology to aid diagnosis. Cor pulmonale results in peripheral edema, including pleural effusion. These are usually serous effusions, which are transudative in nature. Systemic lupus erythematosus (SLE) can be associated with exudative pleural effusions, but they are usually serous and bilateral. Bacterial sepsis due to S. aureus could lead to pleuritis with purulent exudates. Pulmonary embolism (a common cause of pulmonary infarction) results in bloody pleural effusions; however, the cytological examination will be negative.

Case Ico-delete Highlights An 18-year-old man has been diagnosed with asthma since he was 12 years old. The patient states he has been controlled using his rescue inhaler but for some reason his allergies this year are really acting up. You ask him how many times a week he has been using his inhaler. The patient responds "4 times a week." He also wakes up multiple times a month due to shortness of breath. He wheezes on a regular basis. His FEV1 is above 80% predicted. You determine you need to add additional therapy at this time. Question Based on the stepwise approach for asthma, what should you prescribe?

Correct answer: Mometasone furoate Explanation Momemtasone furoate is an inhaled low-dose corticosteroid, which is the preferred treatment for mild persistent asthma. This is a daily maintenance treatment that can be used up to twice a day to help control symptoms of asthma. Advair is a combined low-dose inhaled corticosteroid and long acting beta agonist. This would be used if therapy has not worked with a low-dose inhaled corticosteroid alone or for moderate persistent asthma. Serevent Diskus is a long-acting beta agonist. This would be used in addition to an inhaled corticosteroid. This would be used if therapy has not worked with a low-dose inhaled corticosteroid alone or for moderate persistent asthma. Omalizumab is used for severe asthma and reserved for those that have failed traditional therapies. This inhibits IgE binding to mast cells and basophils. Therapy cost can range from $6,000-$24,000 dollars a year. Oral prednisone is also used to treat severe asthma symptoms. Long term use is not recommended due to prolong osteoporosis, dermatologic changes, and adrenocortical and pituitary unresponsiveness.

Case Ico-delete Highlights A 17-year-old boy presents with intermittent bouts of shortness of breath, coughing, and chest tightness. The symptoms most often occur during football practice; sometimes, however, they also occur when he is just standing outdoors in cold weather. He denies palpitations, fever, and chills. Past medical history is noncontributory. He is a nonsmoker. Physical exam is unremarkable. Vital signs are as follows: BMI 19 kg/m2, BP 116/70 mmHg, HR 80 bpm, and SpO2 99% on room air. His physician orders pulmonary function testing (PFTs) and a chest X-ray (CXR). Question Based on the most likely diagnosis, what chest X-ray finding(s) is most likely?

Correct answer: Normal chest X-ray Explanation Asthma is a chronic condition that affects pulmonary function. The pathophysiologic process underlying asthma is inflammation. In response to specific triggers, an immune-mediated cascade occurs that leads to recruitment of inflammatory mediators and airway constriction. Although this is a normal response to airway irritants, the airway of patients with asthma is hyperresponsive. Bronchial constriction increases the work of breathing, particularly the exhalation phase. In between and even during an acute asthma attack, chest X-ray findings are usually normal. During severe exacerbations, the most common finding is lung hyperinflation. Hyperinflation and hyperlucency are also common in patients with emphysema. Bronchial wall thickening is usually associated with chronic obstructive pulmonary disease - specifically chronic bronchitis, which is marked by mucus hypersecretion. Perihilar fluffy infiltrates can be seen in a number of conditions, including infection with Pneumocystis carinii, lymphoma, or pulmonary edema. They are not seen in patients with straightforward asthma. Ground glass shadowing describes a diffuse hazy appearance to the lungs that may be caused by idiopathic interstitial pneumonia, eosinophilic pneumonia, interstitial fibrosis, or some types of lung cancer. It is not a feature of asthma.

Case A 45-year-old man presents with a fever accompanied by a productive cough. He has had the symptoms for several weeks. His temperature rises in the evenings, and he has experienced weight loss. The chest X-ray shows upper lobe cavitary lesions. Question Ico-delete Highlights What is the most likely diagnosis?

Correct answer: Secondary tuberculosis Explanation An upper lobe cavitary lesion in an adult person is classical of secondary tuberculosis. The granulomas of secondary tuberculosis are mostly present in the upper lobe, and they tend to cavitate when large. The onset of secondary tuberculosis is usually insidious, with the gradual development of general symptoms of cough and sputum. Hemoptysis, pleural pain, or a spontaneous pneumothorax may mark the onset; however, the diagnosis is now frequently made by radiography before the onset of any symptoms. Goodpasture's syndrome is associated with a pulmonary hemorrhage in persons with an anti-glomerular basement membrane antibody that also reacts with the pulmonary basement membrane. Clinically, the patient will present with acute renal failure or with pulmonary symptoms of hemoptysis (with significant impairment of the lung function). Pneumocystis carinii pneumonia is a common infection in immunocompromised individuals. Asbestosis will produce interstitial lung disease as well as pleural plaques. There will be a history of exposure to asbestos. Rarely, mesotheliomas can occur, particularly if associated with a history of smoking. Cor pulmonale is associated with pleural effusion.

Case A 12-year-old boy presents with sudden onset of dyspnea with wheezing. The patient had a similar appearance a month ago. An arterial blood gas shows hypoxemia, hypercapnia, and acidosis. The chest x-ray shows clear lung fields. He is afebrile. Question What is the most likely accompanying laboratory finding in this case? Answer Choices Ico-marker Ico-abct 1 Elevated sweat chloride Ico-marker Ico-abct 2 Numerous sputum eosinophils Ico-marker Ico-abct 3 Decrease CD4 lymphocyte count Ico-marker Ico-abct 4 Elevated white blood cell count Ico-marker Ico-abct 5 Positive tuberculin skin test

Correct answer: Numerous sputum eosinophils Explanation The characteristic dyspnea with wheezing suggests that it is an acute attack of asthma. In children, exposure to an allergen is often the inciting event for extrinsic asthma. This form of obstructive lung disease is episodic and limited, and so chronic lung changes are uncommon; therefore, the classical finding will be the raised eosinophil count, which is due to exposure to an allergen. Elevated sweat chloride is classical for cystic fibrosis. The chronic infections of cystic fibrosis are likely to produce pulmonary consolidations. A decreased CD4 count would suggest that he has HIV infection. He is unlikely that he has complications of AIDS, which are primarily infections in the lung; the x-ray shows clear lung fields. Elevated white count would occur in the context of an acute pneumonia producing wheezing and dyspnea. However, the patient is otherwise asymptomatic, including afebrile. In addition, pneumonia would not explain the patient's episode 1 month ago. Tuberculosis can often be primary and subclinical in children; there may be hilar adenopathy that could produce wheezing, although not in an episodic pattern.

Case A 58-year-old woman with a past medical history of hypertension, hyperlipidemia, and breast cancer is being evaluated in the emergency room after presenting with a 2-hour history of acute-onset severe left-sided pleuritic chest pain. The pain is associated with feelings of anxiety, hemoptysis, shortness of breath, and nausea. She "feels warm" but denies other complaints. She endorses a 30 pack-year smoking history. Upon physical exam, she is found to be febrile, normotensive, tachypneic, diaphoretic, and in acute painful distress. There is perioral cyanosis and a pleural friction rub to the left lung fields; the remainder of the exam is normal. A bedside electrocardiogram revealed the following image. Question What is the next correct step in the management of this patient?

Correct answer: Order a CT angiography of the chest Explanation This patient's presentation suggests a pulmonary embolism. EKG findings include an S1Q3T3 pattern, sinus tachycardia, T wave inversion in leads V1 - V3, Right Bundle Branch Block, and low amplitude deflections. Computed tomography angiography (CTA) is the initial imaging modality of choice for stable patients with suspected pulmonary embolism.The American College of Radiology (ACR) considers chest CTA to be the current standard of care for the detection of pulmonary embolism. The echocardiogram generally has limited accuracy in the diagnosis of pulmonary embolism. Few investigators have reported the feasibility of MRI in the evaluation of pulmonary embolism. However, the role of MRI is mostly limited to the evaluation of patients who have impaired renal function or other contraindications for the use of iodinated contrast material. Prescribing a proton pump inhibitor is not appropriate at this time, as the clinical presentation suggests pulmonary embolism. Thrombolytic therapy is not recommended for most patients. The role of thrombolytic therapy in the management of acute pulmonary embolism is controversial and has not demonstrated benefits in terms of reduced mortality rates or earlier resolution of symptoms when currently compared with heparin. The currently accepted indications for thrombolytic therapy include hemodynamic instability, such as systolic blood pressure less than 90 mmHg or systolic blood pressure drop of greater than 40 mmHg from baseline for at least 15 minutes, or right ventricular dysfunction demonstrated on echocardiogram. Thrombolytic therapy should not be used in patients with major contraindications due to bleeding risks.

Case A 3-year-old boy presents with a loud cough. Two days ago, the boy developed a runny nose and irritability; the next day, he began to cough loudly and felt warm. His mother tells you that the cough sounded like a wounded animal or a dog barking. The child is diagnosed with croup, and humidification is prescribed. Question What organism is most likely responsible for the patient's croup?

Correct answer: Parainfluenza virus Explanation The correct answer is parainfluenza virus, the most common cause of croup. Croup is more formally known as acute laryngotracheobronchitis. It occurs most often in children 2 to 3 years of age, and it has a distinctive 'seal barking' sound. This is caused by the marked edema in the respiratory tract. The same edema may show up on an A-P neck film as subglottic edema, or the "hourglass sign." This sign is also referred to as the "steeple sign." Group A streptococcus does not cause croup, but it is problematic in pharyngitis. Haemophilus influenzae type b is strongly associated with acute epiglottitis. The vaccine has been effective in reducing the incidence of Hib epiglottitis. Influenza virus can cause croup, but does so less commonly than the parainfluenza virus. It is a more common cause of pneumonia than croup. Therefore, parainfluenza is the most likely organism responsible for this patient's symptoms. Rubella virus infection presents with a rash as well as malaise and upper respiratory symptoms. It does not cause croup.

Case A 72-year-old man is evaluated at the bedside following hospital admission for a 1-year history of progressive dyspnea, weight loss, low-grade fevers, fatigue, and myalgias. His past medical history is remarkable for atrial fibrillation, for which he takes amiodarone, hypercholesterolemia, and recurrent urinary tract infections, for which his urologist prescribed nitrofurantoin on a chronic, prophylactic basis. He denies any cigarette use, history of murmurs or coronary artery disease, chest pain, wheezing, hemoptysis, syncope, abdominal pain, rashes, peripheral edema, diaphoresis, or vomiting. Bedside echocardiogram and electrocardiograms are unremarkable for abnormalities. A chest x-ray revealed peripheral reticular opacities at the lung bases and a generalized honeycombing pattern (refer to image). Question Which of these statements regarding the clinical presentation of this patient is correct?

Correct answer: Physical exam findings of pulmonary hypertension commonly occur. Explanation This patient's most likely diagnosis is idiopathic pulmonary fibrosis. It is critical to obtain a complete history, including medication history, social history, occupational history, exposure history, and review of systems, to ensure other causes of interstitial lung disease are excluded. Amiodarone, bleomycin, and nitrofurantoin are notable medications associated with pulmonary fibrosis. Most patients with idiopathic pulmonary fibrosis present with a gradual onset, often for longer than 6 months duration. Dyspnea, which is the most prominent symptom in idiopathic pulmonary fibrosis (IPF), usually begins insidiously and is often progressive. Approximately 5% of patients have no presenting symptoms when idiopathic pulmonary fibrosis is diagnosed. The clinical symptoms of idiopathic pulmonary fibrosis are nonspecific; symptoms often precede the diagnosis by a median of 1-2 years. Most patients present with exertional dyspnea and a nonproductive cough. Pulmonary hypertension is a common comorbidity in patients with idiopathic pulmonary fibrosis, and an estimated 20-40% of patients with idiopathic pulmonary fibrosis who are evaluated or listed for lung transplantation have pulmonary hypertension at rest. Patients may have a loud P2 component of the second heart sound, a fixed split S2, a holosystolic tricuspid regurgitation murmur, and pedal edema. As right ventricular hypertrophy ensues, a right ventricular heave may be palpated at the lower left sternal border and increased right atrial pressure may cause elevation of the jugular venous pressure. The chest radiograph lacks diagnostic specificity for idiopathic pulmonary fibrosis. In most patients with idiopathic pulmonary fibrosis, the physical examination reveals fine bibasilar inspiratory crackles (Velcro crackles). Additionally, digital clubbing is seen in 25-50% of patients with IPF.

Case A 69-year-old woman presents with shortness of breath. She states it has been worsening over the last 3 - 4 days; she also is experiencing increased fatigue. The patient is not on any daily medications other than over-the-counter multivitamins, and she has no pertinent past medical history. Physical examination is significant for an oral temperature of 101.5° F, and during auscultation, there are absent breath sounds noted in the right lower lung field. Tactile fremitus reveals an absent result in that same lung field, and percussion over that area creates a dull percussion note. Question What is the most likely diagnosis?

Correct answer: Pleural effusion Explanation The correct response is pleural effusion. The key findings are absent breath sounds, specifically in the lower lung field during auscultation, percussion that produces a dull note in the same area, and tactile fremitus absent over that location. All these signs indicate that fluid accumulation is highly likely in the pleural space in that area (which separates the air-filled lung from the chest wall) and blocks the transmission of sound. Another sign is a notable fever (in an elderly patient) indicating some type of infectious process. Symptoms further lead to respiratory or cardiology involvement. During auscultation, breath sounds are many times obscured by wheezing; percussion will reveal resonant and hyper-resonant notes. Tactile fremitus will be absent. Fever is not a feature of asthma. Chronic obstructive pulmonary disease (COPD) is not the correct choice. Although during auscultation breath sounds would be potentially decreased to absent, these will be diffusely noted (not just concentrated in 1 area). There also may be crackles, wheezes, or rhonchi heard with this particular diagnosis. When percussion is performed in a patient with COPD, there will be diffusely hyper-resonant percussion notes produced. Tactile fremitus would be decreased, but this would be throughout the lung fields. Fever could potentially be an associated finding in COPD. Early left-sided heart failure is not the correct choice. In this pathology, there is increased pressure in the pulmonary veins which causes congestion and interstitial edema around the alveoli; bronchial mucosa may also become edematous. For this reason, auscultation will potentially reveal vesicular breath sounds; there may also be late inspiratory crackles in dependent portions of the lung fields (and some wheezing). Tactile fremitus will be normal; percussion will reveal resonant notes. Fever is not a sign of this diagnosis. Pneumothorax occurs when there is air leaking into the pleural space, typically unilaterally, causing the lung to recoil away from the chest wall. Pleural air blocks the transmission of sound (as in this patient), but will produce different physical examination findings; during percussion, the affected area with produce a hyper-resonant (or even tympanic) note over the affected area, auscultation will be absent, but tactile fremitus may be decreased or potentially absent. Fever is not a common finding.

Case A 72-year-old man presents with longstanding but increasing dyspnea, and a 3-day history of shortness of breath, coughing, unilateral sharp chest pain and mild temperature elevation. Pain is worse when he takes a deep inspiration and when he coughs. PMH is significant for a smoking history of 2 packs/day for 40 years (80 pack-years). The patient has other co-morbidities, including CAD, COPD, CHF, hypertension, and dyslipidemia. He denies recent travel, sick contacts, occupational exposure, and drug or alcohol use. Vital signs are BP 190/96, HR 140, RR 48, 02 68% on room air. Chest examination of the left posterior chest reveals a dull percussion note, inaudible bronchovesicular breath sounds, decreased tactile fremitus, a pleural friction rub on inspiration and diminished voice sounds below the sixth intercostal space. Significant bilateral lower extremity edema is also noted. A chest X-ray reveals blunting of the costophrenic angle. Question What is the most likely diagnosis?

Correct answer: Pleural effusion Explanation This patient's signs and symptoms are consistent with pleural effusion. The key finding is blunting of the costophrenic angle on chest X-ray. Fluid in the pleural space pushes the lung upward resulting in a blunting of the costophrenic angle that is normally seen as a sharply pointed downward indentation. Results of the chest examination show decreased or absent breath sounds, dullness to percussion over affected area, decreased tactile fremitus, and friction rub. The most common cause of pleural effusion is left-ventricular failure. Pneumothorax is characterized by hyperresonance of percussion note, JVD due to compression of the superior vena cava, falling O2 saturation, and hypotension. Chest X-ray shows a visceral pleural line and/or lung retraction from the chest wall. Pulmonary thromboembolism is characterized by sudden onset dyspnea, pleuritic chest pain, hemoptysis, tachypnea, unilateral lower extremity edema, hypoxia, and hypocarbia. Pulmonary fibrosis is associated with exertional dyspnea, non-productive cough, and crackles heard on inspiration. CT scan shows patchy opacities at lung bases with honeycombing. Bronchiectasis is associated with dilation of the bronchi. Patients present with productive cough with yellow or green sputum, dyspnea, hemoptysis. Exam reveals rales, rhonchi, wheezes, purulent mucus. Chest X-ray reveals increased bronchovascular markings, tram lines, and areas of honeycombing.

Case A 76-year-old man with hypertension, coronary artery disease, and poorly-controlled left ventricular congestive heart failure presents with a 3-day history of insidious chest pain. Pain is made worse when he takes a deep breath in and when he coughs. He denies any relation of pain to position, activity, or food intake. He denies fever, chills, palpitations, sputum production, wheezing, abdominal pain, nausea, vomiting, diarrhea, or peripheral edema. His physical exam reveals a widespread friction rub upon inspiration, absent lung fremitus, and reduced lung sounds over the thoracic cavity. Question What finding would be anticipated upon further diagnostic testing?

Correct answer: Pleural fluid N-terminal pro-brain natriuretic peptide levels greater than 1500 pg/mL Explanation This patient's presentation is most consistent with a pleural effusion. This patient is demonstrating pleuritis, which is caused by damage and inflammation to the pleura of the lung. The most common cause of pleural effusion is left ventricular failure, which causes a transudative effusion. The characteristics of pleural fluid transudate are a yellow appearance, specific gravity of less than 1.016, absolute protein of less than 3 grams per 100 mL, a pleural protein to serum ratio of less than 0.5, a pleural to serum LDH ratio of less than 0.6, a serum to pleural glucose ratio of less than 1, and very low WBCs. A pleural fluid N-terminal pro-brain natriuretic peptide (NT-proBNP) >1500 pg/mL is virtually diagnostic of an effusion secondary to congestive heart failure. The earliest sign of pleural effusion on the frontal view of a chest radiograph is a blunting of the lateral costophrenic angle. A large free pleural effusion appears as a dependent opacity with lateral upward sloping of a meniscus-shaped contour. The diaphragmatic contour is partially or completely obliterated, depending on the amount of fluid (silhouette sign). Lobar consolidations with air bronchograms, patchy airspace opacities, or diffuse alveolar or interstitial opacities are common radiographic findings consistent with pneumonia. Hyperinflation, hyperlucency, and depressed diaphragms upon chest X-ray assessment suggest asthma. Increased lung volumes and increased retrosternal airspace may also be observed.

Case A 62-year-old man presents with a 2-month history of worsening fatigue and shortness of breath. He has a past medical history of emphysema attributable to his 85 pack-year cigarette smoking history. The patient complains of nearly passing out while climbing the stairs in his house. He tells you that he feels like his heart races. He reports chronic shortness of breath and cough, but he now he feels like his dyspnea is dramatically worse; he can no longer sleep in his bed. He has been trying to sleep propped up in a chair at night. He is also experiencing fatigue. He has gained about 15 pounds, and he notes that he can no longer lace up his shoes. He denies fever, chills, and chest pain. His cough produces some mucus, but no hemoptysis. His vitals are shown in the table. Weight 212 lb Height 69" Body mass index (BMI) 31.3 Pulse 108 Blood pressure 140/88 mm Hg Temperature 98.2°F Pulse oximetry 88% On physical exam, you see a man in mild respiratory distress; he is sitting upright and leaning forward, and he uses accessory respiratory muscles for breathing. The exam is significant for reduced air movement and mild rales bilaterally in the lungs, distended neck veins, mild tachycardia with prominent P2, lower extremity edema, and right upper quadrant abdominal tenderness with hepatomegaly. Question Once this patient's current condition is stabilized, what health maintenance recommendation is most appropriate for this patient based on the most likely diagnosis?

Correct answer: Pneumococcal polysaccharide vaccination Explanation Normally recommended for all individuals 65 and over, the pneumococcal polysaccharide vaccination is recommended for younger patients who have chronic lung diseases, such as this patient. Blood pressure screening of adults is recommended by multiple organizations. A recommended interval is every 2 years for those with normal blood pressures and every year for those with mildly elevated blood pressures (systolic 120-139 mm Hg and/or diastolic 80-89 mm Hg). This patient has a systolic blood pressure of 140 mm Hg and other cardiovascular complications, however, so he should be monitored more frequently. If high blood pressure remains elevated, he should be treated. Colonoscopy is a colon cancer screening technique routinely recommended for all men and women 50 and over; it may be recommended earlier if a family history of colon cancer is present. If he has not yet had one, this patient should undergo a screening colonoscopy, but this does not directly relate to his presenting condition (emphysema/chronic obstructive pulmonary disease, progressed into cor pulmonale). Preventive services are most effective when tailored to the patient's specific needs. In the patient's case, a pneumococcal vaccination is a higher priority preventive service than colonoscopy. Prostate-specific antigen (PSA) testing for prostate cancer has been controversial. Currently, the US Preventive Services Task Force is neither for nor against PSA screening. Given his severe pulmonary disease, PSA testing is not a high priority for this patient. This patient's current BMI is over 30, indicating obesity, but the patient's history reveals very recent weight gain attributable to fluid retention. Before any aggressive weight loss is recommended, the edema should be addressed through sodium and fluid restriction and most likely diuretic medications. Furthermore, aggressive physical activity for weight loss would likely exacerbate his condition, so it is not recommended.

Case Ico-delete Highlights A 2-year-old girl presents with cough, fever, decreased oral intake, and irritability for a few days. Prior to presentation, she had a 1-week history of an upper respiratory infection. On exam, she is febrile to 102°, has a respiratory rate of 28, a pulse of 110, and blood pressure of 88/56. She appears tired, but she is responsive, interactive, and in no acute respiratory distress. On exam, there are decreased breath sounds at the bases and crackles. Question What is the most likely diagnosis?

Correct answer: Pneumonia Explanation Fever and cough combined with the abnormal lung exam make pneumonia the most likely diagnosis. Foreign body aspiration may occur in this age group but is less likely to be associated with fever and global changes on lung exam. Asthma exacerbation usually presents with more prominent respiratory distress and wheezes on lung exam. Fever is not present unless there is an underlying infection. Pulmonary embolism is rare in healthy children. It would not cause fever. Congenital heart disease may cause pleural effusion, marked by decreased breath sounds and dullness to percussion. It is unlikely to be associated with fever and more likely to be associated with a history of failure to thrive.

Case A 5-year-old girl has paroxysms of cough that increase in severity and duration. Some coughing episodes are followed by a whoop-like, high-pitched inspiratory noise, and vomiting has also occurred after paroxysms. Question What laboratory test could lead to the earliest confirmation of the likely diagnosis?

Correct answer: Polymerase chain reaction assay and antigen detection Explanation The correct response is polymerase chain reaction assay and antigen detection. The characteristics presented by this child suggest B. pertussis or parapertussis infection. The diagnosis of pertussis is still primarily clinical, and laboratory results only play a small role in patient management. Chest X-ray cannot confirm the suspected diagnosis. White blood count is likely to be elevated, with the lymphocytes composing 70-80% of the total count, but this finding is nonspecific. The infection can be directly confirmed by culture for Bordetella pertussis, but it takes several days, and there is a high rate of false negative results. Immunofluorescent staining of nasopharyngeal washing can be done, and it is significantly more sensitive than culture. PCR assay and antigen detection are increasingly used to assist in diagnosing pertussis. Advantages include greater sensitivity, more rapidly available results, and use later in the disease course or after antimicrobial therapy because the tests do not rely on the isolation of viable organisms.

Case A 72-year-old man presents with an 8-month history of progressive dyspnea, accompanied by a dry and persistent hacking cough. While the dyspnea now occurs at rest, he denies fever, chills, palpitations, chest pain, or peripheral edema. He states that he has worked for many years at a local chemical plant. His physical exam is remarkable for digital cyanosis and clubbing, while his pulmonary exam reveals diffuse, fine, dry inspiratory crackles. His cardiac exam was positive for a prominent pulmonary valve closure sound (P2) and an elevated jugular venous pressure of 6 cm. A chest X-ray noted small lung volumes, with increased densities in the lung periphery and a honeycombing pattern; pulmonary function testing measured reductions in TLC, FEV1, and FVC with a preserved FEV1/FVC ratio. Question What is the best treatment for this patient at this time?

Correct answer: Prednisone Explanation This patient most likely has hypersensitivity pneumonitis (HP), also known as extrinsic allergic alveolitis. This is an inflammatory disorder of the lung involving alveolar walls and terminal airways that is induced by repeated inhalation of a variety of organic agents. The chronic form of HP typically results from low-grade or recurrent exposure over many months/years, and the lung disease may already be partially or completely irreversible. These patients are usually advised to avoid all possible contact with the offending agent. In addition to identifying and avoiding the causative agent, institution of glucocorticoid treatment is indicated. Prednisone at a dosage of 1 mg/kg per day or its equivalent is continued for 7-14 days, then tapered to 0.25-0.5 mg/kg and is maintained at this level for an additional 4-12 weeks at a rate that depends on the patient's clinical status. While patients with chronic HP may gradually recover without therapy following environmental control, a trial of prednisone may be useful to obtain maximal reversibility of the lung disease. Following initial prednisone therapy (1 mg/kg per day for 2-4 weeks), the drug is tapered to the lowest dosage that will maintain the functional status of the patient. Many patients will not require or benefit from long-term therapy if there is no further exposure to the antigen. Improvement of lung function may continue over a few months to years. If the patient's condition continues to decline on glucocorticoids, a second agent should be introduced while lowering or maintaining the prednisone dose at 0.25 mg/kg per day. Glucocorticoid therapy is recommended for symptomatic interstitial lung disease (ILD) patients with eosinophilic pneumonia, cryptogenic organizing pneumonia, connective tissue diseases, sarcoidosis, hypersensitivity pneumonitis, acute inorganic dust exposures, acute radiation pneumonitis, diffuse alveolar hemorrhage, and drug-induced ILD. In organic dust disease, glucocorticoids are recommended for both the acute and chronic stages.

Case A 9-year-old boy presents with a productive cough for the past few days. The patient also has been having fevers of 101°F. The patient has had recurrent bouts of pneumonia. The patient's neonatal course was complicated by meconium ileus. A sweat chloride test is positive and sputum examination reveals gram-negative rods that are oxidase positive. The culture has a greenish tint. Refer to the image. Question Based on the patient's underlying condition and the sputum sample, this infection is most likely caused by what organism?

Correct answer: Pseudomonas aeruginosa Explanation This child who has recurrent bouts of pneumonia, a history of meconium ileus, and a positive sweat chloride test most likely has cystic fibrosis. Cystic fibrosis is an autosomal recessive disorder affecting chloride channel protein. Children with cystic fibrosis are very susceptible to infections. Most notable is their susceptibility to infection with Pseudomonas aeruginosa. Pseudomonas is a gram-negative rod that is strictly aerobic and oxidase-positive. Some species produce pigments such as pyocyanin and pyoverdine, which cause a greenish tint on agar growth media. Staphylococcus species appear as gram-positive clusters, while Streptococcus species appear as gram-positive diplococci. Haemophilus influenzae is a small gram-negative (coccobacillary) rods. It causes a variety of diseases, including sinusitis and pneumonia. Listeria monocytogenes is a small gram-positive rod. It is most commonly associated with meningitis and sepsis in newborns and immunocompromised individuals, not pneumonia.

Case A 45-year-old man presents with progressive dyspnea and chest discomfort. He is 2 days status post total hip arthroplasty, which was uncomplicated. He denies any trauma to the chest. On exam, his pulse is 110 BPM; respirations are 40/min. The chest radiograph reveals a wedge-shaped opacity. Lab studies reveal a higher D-dimer level. Question What is the most likely cause of his chest pain?

Correct answer: Pulmonary embolism Explanation Pulmonary embolism pain is often abrupt in onset and pleuritic in nature. It can last several minutes to a few hours. Cough and hemoptysis may also be present. There may be predisposing factors such as deep venous thrombosis or long periods of immobilization. Orthopedic surgery is also a risk factor for pulmonary embolism. On examination, patients are dyspneic and tachypneic with a tachycardia. Chest radiograph findings that are suggestive of a pulmonary embolus are the Hampton hump and the Westermark sign. The Hampton hump is a wedge-shaped opacity that has a tip pointing towards the hilus. The Westermark sign is decreased vascularity. D-dimer is a measure of fibrin degradation products. A normal value reliably rules out the diagnosis of pulmonary embolism, while an elevated value suggests the diagnosis of pulmonary embolism. Pneumothorax pain is usually described as the sudden onset of pleuritic chest pain. Other signs and symptoms of pneumothorax such as dyspnea, tracheal deviation, and decreased breath sounds may also be present. Chest radiograph findings of pneumothorax include a mediastinal shift and the presence of air without lung markings in the affected side. Esophageal reflux pain is a substernal pain usually described as burning. It may last for up to 1 hour. It is relieved by antacids and may be aggravated by alcohol and postprandial recumbency. Aortic dissection pain is of abrupt onset and usually described as severe and tearing. It may radiate from the anterior chest wall posteriorly to the interscapular region. On exam, blood pressure may be elevated and a pericardial rub and diastolic murmur of aortic insufficiency radiating along the right sternal border may be auscultated. Unstable angina pain is usually described as tightness or pressure on the chest and lasts for 10-20 minutes. It is usually left-sided or retrosternal and may radiate to the jaw and neck. It often occurs at rest, but it may be precipitated by physical exertion and emotional stress. On examination, a systolic murmur of mitral regurgitation may be auscultated during an episode of pain.

Case A 33-year-old African American woman with no significant past medical history, who is in her 38th week of a normal pregnancy, presents with a 1-hour history of shortness of breath. She does not recall any precipitating activities or events that may have provoked these symptoms and recalls that she was laying in her bed for several hours since waking when the symptoms developed. She admits to an associated sharp, non-radiating pleuritic chest pain, as well as lower extremity swelling, which she states has been "persistent throughout the course of her pregnancy." She denies palpitations, chest pressure, cough, sputum, fever, chills, changes in weight, rashes, diaphoresis, abdominal pain, nausea, or a history of allergies. Her physical exam is noteworthy for tachypnea and tachycardia, but the rest of the vital signs are normal. Her lungs are clear to auscultation bilaterally, without wheezing, rhonchi, or crackles. Her lower extremities are remarkable for 2+ pitting edema up to the level of her knees; there is no calf tenderness, venous cords, or Homan's sign appreciated. Her skin and mucous membranes were without diaphoresis or cyanosis. A bedside EKG reveals sinus tachycardia at 120 bpm with prominent S waves in lead I and Q waves in lead III. Question What is the most likely diagnosis?

Correct answer: Pulmonary embolism Explanation This patient has a presentation most consistent with pulmonary embolism. Nearly all PEs arise from deep venous thrombosis (DVT) in the lower extremity or pelvic veins. Risk factors for DVT and PE are similar in children and adults and include conditions that impair venous return, conditions that cause endothelial injury or dysfunction, and underlying hypercoagulability disorders. Bed rest and confinement without walking, even for a few hours, are common precipitators. Specific risks include an age over 60, atrial fibrillation, cigarette smoking (including passive smoke), estrogen receptor modulators, exogenous estrogens, and progestins (including oral contraceptives and estrogen therapy), extremity or pelvic trauma, heart failure, hypercoagulability disorders, immobilization, indwelling venous catheters, myeloproliferative disorders, nephrotic syndrome, obesity, pregnancy and postpartum states, sickle cell anemia, recent surgeries, and prior venous thromboembolism. Larger emboli cause acute dyspnea, pleuritic chest pain, or both. Dyspnea may be intermittent or occur only with exercise. Less common symptoms include cough and hemoptysis. The most common signs of PE are tachycardia and tachypnea. Less commonly, patients have hypotension, a loud second heart sound (S2) due to a loud pulmonic component (P2), and crackles or wheezing. In the presence of right ventricular failure, distended internal jugular veins and a right ventricular heave may be evident, and right ventricular gallop (third and fourth heart sounds [S3 and S4]), with or without tricuspid regurgitation, may be audible. ECG most often shows tachycardia and various ST-T wave abnormalities, which are not specific for PE. An S1Q3T3 or a new right bundle branch block may indicate the effect of abrupt rise in right ventricular pressure on right ventricular conduction; these findings are moderately specific but insensitive for PE. Pulmonary edema typically causes severe dyspnea, the production of pink and frothy sputum, and diaphoresis and cyanosis. Rales are present in all lung fields, as are generalized wheezing and rhonchi. The chest X-ray usually indicates vascular redistribution, blurriness of vascular outlines, increased interstitial markings, and, characteristically, the butterfly pattern of distribution of alveolar edema. The patient's younger age, quality of chest pain, lack of gastrointestinal and skin findings, and absence of characteristic EKG findings make acute myocardial infarction a less likely diagnosis. The rapid onset of symptoms and denial of fever, cough, and sputum suggest a diagnosis other than bacterial pneumonia, as does the absence of an allergic history and adventitious lung sounds.

Case A 44-year-old healthy man, who is a non-smoker, has a 3-cm coin lesion in the right upper lobe. The lesion was revealed on his chest radiograph. The patient did not have any clinical symptoms. The fine-needle aspiration did not reveal any cells. Question Ico-delete Highlights What is the most likely tumor in this case?

Correct answer: Pulmonary hamartoma Explanation A peripheral 'coin lesion' could be a granuloma, carcinoma, or a hamartoma. The most likely malignancy in a non-smoker would be adenocarcinoma. Pulmonary hamartoma is the most common benign lung tumor, although it is a relatively uncommon lesion. It is likely an acquired lesion from peribronchial mesenchyme. The patients are generally asymptomatic. The lesion is almost always solitary and is usually peripheral; hence, it does not reveal any cells on fine-needle aspiration. Histologically, hamartomas are composed of both mesenchymal and epithelial elements. A bronchial carcinoid is (usually) a central benign tumor with some of the properties of malignant growth. Mesothelioma is a bulky pleural mass. It is rare, even in smokers with asbestos exposure. Metastatic adenocarcinoma usually produces multiple lung nodules. Large cell undifferentiated carcinoma is a large peripheral mass.

Case Ico-delete Highlights A 47-year-old Caucasian woman, previously healthy, presents for evaluation of a 4-week history of dyspnea. Her symptoms started approximately 1 month ago when on vacation in Colorado. She initially attributed these symptoms to the altitude; however, upon return to her hometown, she continued to have shortness of breath with mild activity, such as walking more than 100 feet, walking upstairs 1 flight, vacuuming, or sweeping. Her symptoms resolved with rest. She also reports mild exertional chest tightness and easy fatigability. She denies paroxysmal nocturnal dyspnea, orthopnea, edema, palpitations, and syncope. Past medical history includes the usual childhood illnesses. She has no previous surgeries and no known allergies. Medications include a daily multivitamin and occasional Tylenol for headaches. She is a non-smoker, rarely uses alcoholic beverages, and denies the use of illegal or illicit drugs. Physical exam reveals an alert white woman in no acute distress. Her vitals reveal temp 96.9°F, pulse 80 and regular, respirations 16, and BP 136/82. O2 sat is 96% on room air. There is no obvious jugular venous distention. Respirations are non-labored. Lung fields are clear to auscultation and percussion. No rhonchi, rales, or wheezes are present. Heart shows RV heave present, normal S1 with fixed, split S2 with prominent P2 component. Grade II/VI systolic murmur is present at the left upper sternal border at the second intercostal space. The remainder of a complete physical examination is within normal limits. CBC and BMP are unremarkable. Free T4 and TSH are within normal limits. EKG shows normal sinus rhythm with right ventricular hypertrophy, right atrial enlargement, and right axis deviation. There is an RSR in leads v1 and v2. Question What do you suspect is the primary cause of her dyspnea?

Correct answer: Pulmonary hypertension Explanation The correct answer is pulmonary hypertension. The reduced oxygen concentration present at high altitudes can trigger (or worsen) the development of pulmonary hypertension in patients who were previously asymptomatic. Symptoms of pulmonary hypertension include dyspnea, associated chest tightness, and easy fatigability. Palpitations or dizziness may be associated symptoms. With more severe pulmonary hypertension, symptoms of right-sided heart failure (pedal or peripheral edema, ascites, hepatomegaly, or jugular venous distention) may develop. Angina secondary to coronary artery disease is incorrect. Angina most commonly presents as exertional chest tightness alleviated by rest, but it often presents as exertional dyspnea. This patient has both of these symptoms; however, angina alone would not account for the abnormalities on her physical exam (cardiac murmur, widely fixed and split S2) and EKG (right ventricular hypertrophy, right atrial enlargement, right axis deviation, and incomplete right bundle branch block). This patient also does not have typical risk factors for CAD, such as hypertension, diabetes, high cholesterol, or smoking. Aortic stenosis is incorrect. Aortic stenosis can cause exertional dyspnea and typical exertional angina; however, aortic stenosis would not cause right ventricular hypertrophy. It could, however, cause left ventricular hypertrophy. Atrial septal defect is incorrect. Atrial septal defect is a congenital heart defect present at birth. If an atrial septal defect were the principal cause of her dyspnea, the patient would have had dyspnea her entire life. This patient did not develop dyspnea until four weeks ago, due to the development of pulmonary hypertension after high-altitude exposure. Atrial septal defect is associated with a widely-fixed split S2 and cardiac murmur and could cause right ventricular hypertrophy and right atrial enlargement. Atrial septal defect can cause long-standing right ventricular volume and pressure overload, which can certainly be contributing factors to the development of pulmonary hypertension. cause right ventricular hypertrophy and right atrial enlargement. Idiopathic pulmonary fibrosis is incorrect. Idiopathic pulmonary fibrosis could certainly cause dyspnea and fatigue; however, it typically would be associated with bibasilar inspiratory lung crackles on physical examination. This patient had normal auscultation of the lungs. Idiopathic pulmonary fibrosis rarely affects patients younger than 50.

Case A 69-year-old man with a 50 pack-year smoking history, COPD for 12 years, and a myocardial infarction 2 years ago has been experiencing increased exertional dyspnea for 4 months. There is associated easy fatigability, exertional chest discomfort, and lightheadedness. He denies fever, chills, palpitations, cough, wheezing, abdominal pain, nausea, vomiting, and diarrhea. Physical exam findings were remarkable for a right ventricular heave, widely split S2 with an accentuated pulmonic component, a pulmonary ejection click, an S3 and 1+ pitting edema to the bilateral lower extremities. There was also evidence of a 5-cm jugular vein distention. Question What diagnostic test results would be expected in this patient?

Correct answer: Right axis deviation, R wave greater than S wave in V1, and peaked p-waves on EKG Explanation This patient's presentation is consistent with secondary pulmonary arterial hypertension. Common ECG findings include right axis deviation, R > S in V1, S1Q3T3, and peaked P waves. Common chest X-ray findings include enlarged hilar vessels that rapidly prune into the periphery, enlargement of central pulmonary arteries, attenuation of peripheral vessels, oligemic lung fields, and a right ventricle that fills the anterior airspace on lateral view. Given this patient's prior history of COPD, an obstructive pulmonary function test pattern is expected, not normal FEV1 to FVC ratios. Echocardiography reveals signs of chronic right ventricular pressure, such as increased thickness with paradoxical bulging of the septum into the left ventricle during systole. In later stages, right ventricular dilatation occurs, leading to hypokinesis. Right atrial dilatation and tricuspid regurgitation are also present. A mean pulmonary arterial pressure of >25 mm Hg in the absence of an underlying disorder identifies pulmonary hypertension (most patients with pulmonary arterial hypertension present with substantially higher pressures).

Question A New Orleans resident invites his friend to partake in a Mardi Gras tradition: the King cake. Baked into the sweet cake is a small trinket, which is shaped like a baby. The tradition is as follows: whoever gets the piece of cake with the baby in it has to buy the king cake the following year. Unfortunately, his friend accidentally bites into the trinket and inhales a small fragment of it (about 1 cm x 1 cm) while laughing. Where is the trinket most likely to lodge?

Correct answer: Right main bronchus Explanation The trachea has a diameter of approximately 2 cm. The right main bronchi is more vertical and wider (and shorter) than the left main bronchi, so things are more likely to lodge there. When children swallow things that end up in the respiratory system, they are more likely to lodge in the right main bronchi. The bronchioles are smaller than 1 mm in diameter and are too small for something this size to be lodged. With a diameter less than 0.5 mm, the terminal bronchioles are the smallest bronchioles; they are too small for something this size to become lodged.

Case A 63-year-old man with a past medical history of hyperlipidemia presents with an 8-month history of intermittent flushing and warmth sensation to his face and neck areas, alternating watery, frothy, and bulky stools associated with steatorrhea, weight loss, and wheezing. His urine demonstrated increased 5-hydroxyindoleacetic acid (HIAA) levels. Scintigraphy revealed innumerable foci of intense activity scattered throughout the liver. Question What agent is highly effective at reducing symptoms?

Correct answer: Sandostatin (Octreotide) Explanation This patient's diagnosis is consistent with carcinoid tumor. Carcinoid tumors are of neuroendocrine origin and derived from primitive stem cells in the gut wall, but they can be seen in other organs, including the lungs,mediastinum, thymus, liver, pancreas, bronchus, ovaries, prostate, and kidneys. Carcinoid tumors have high potential for metastasis. Neuroendocrine tumors contain somatostatin receptors. This feature allows for the localization of primary tumors and tumor metastases by scintigraphy with the radiolabeled somatostatin analog octreotide. It can be used to detect certain types of cancer arising from the neuroendocrine system. As a somatostatin analog, sandostatin is highly effective in reducing symptoms. Survival advantage with the use of this drug has not yet been proven. Sandostatin reduces the amount of the growth factor produced, thus theoretically impairing growth. Danocrine suppresses pituitary-ovarian axis by inhibition of pituitary gonadotropin output. It is indicated in the treatment of endometriosis, fibrocystic breast disease, and hereditary angioedema. Cytarabine is an antimetabolite chemotherapeutic agent that inhibits DNA polymerase during the S phase; it is indicated in the treatment of acute myelogenous leukemia and acute lymphocytic leukemia.

A 45-year-old woman presents with insidious onset of increasing dyspnea. A chest X-ray revealed nodular infiltrates and marked hilar lymphadenopathy. The transbronchial biopsy demonstrated non-necrotizing granulomas. Question What is the most likely diagnosis? Answer Choices Ico-marker Ico-abct 1 Asbestosis Ico-marker Ico-abct 2 Idiopathic pulmonary fibrosis Ico-marker Ico-abct 3 Coccidioidomycosis Ico-marker Ico-abct 4 Hypersensitivity pneumonitis Ico-marker Ico-abct 5 Sarcoidosis

Correct answer: Sarcoidosis Explanation Sarcoidosis is a multisystem granulomatous disease. Histologically, it is characterized by the presence of non-necrotizing granulomas and hilar lymph node involvement occurs in 100% of the cases. Any infection has to be ruled out. 30% of the patients of sarcoidosis are asymptomatic, and the disease is commonly detected by an abnormal chest radiograph revealing bilateral hilar lymphadenopathy in an asymptomatic patient. However, with extensive lung involvement, there may be exertional dyspnea or cough. The diagnosis can be made confidently from the clinical and radiological feature, and it can be confirmed histologically from the biopsy of a superficial lymph node or skin lesion. Asbestosis leads to the formation of interstitial lung disease, pleural plaques, and (rarely) mesothelioma. Granulomas are not a feature of asbestosis. Idiopathic pulmonary fibrosis leads to interstitial lung disease and not granulomatous disease. Coccidioidomycosis is the result of infection with dimorphic fungi Coccidioides immitis. These produce granulomas that are usually caseating. Hypersensitivity pneumonitis is an extrinsic allergic alveolitis.

Case A 3-year-old boy presents with a 4-hour history of respiratory distress, dysphagia, and fever. On examination, temperature is 104.3°F, pulse 150, and respirations 32 and shallow. Marked inspiratory stridor with an open-mouth appearance and sialorrhea is present. Question The initial step in management is:

Correct answer: Securing the airway Explanation The above description is characteristic of acute upper airway obstruction due to an infection, with epiglottitis being the most likely cause. All diagnostic and therapeutic measures must be held in abeyance until an airway is secured. This is ideally done in the operating room under a "double setup," with preparation to perform endotracheal intubation or tracheostomy as needed. Blood gases or anything else that can excite the child should be deferred until the airway is secured. While antibiotics are important in treatment of epiglottitis, the airway must be secured first. Lateral neck radiographs can confirm the diagnosis in doubtful cases, but they should be performed only after the airway is established. Attempts to directly visualize the epiglottis without the ability to immediately intubate or perform a tracheotomy can lead to total airway obstruction, and they should not be attempted.

Case Ico-delete Highlights A 65-year-old retired man worked for 30 years at a mineral plant that processes diatomaceous earth. The area surrounding the plant is pretty; however, the residents have long complained that the plant produces a white dust that has coated the terrain. Over time, the man develops difficulty breathing. Finally, he sees his family doctor. He is a nonsmoker, and he denies any joint pain. Pulmonary tests show a mixed obstructive/restrictive pattern. A CBC with differential is done, and the results are all within normal limits. X-rays are done; they show eggshell calcifications of the hilar nodes. X-rays also show nodules in the upper lung fields. A biopsy of the lung shows doubly refractile particles (when viewed with polarized light). Question What is the most likely diagnosis?

Correct answer: Silicosis Explanation Workers in the mining industry, because of exposure to silica dust, can develop silicosis. Diatomaceous earth is primarily crystalline silica. Eggshellcalcificationin the hilar lymph nodespoints towards a diagnosis of silicosis. Doubly refractile (birefringent) particleswith polarized light are seen with silicosis. Doubly refractile particles with polarized light are seen with silicosis, not with bronchogenic carcinoma. Caplan's syndrome is seen with pneumoconiosis. It is characterized by nodules in the lung and rheumatoid arthritis. Since he denies joint involvement, this is not the case here. Hypersensitivity pneumonitis is an allergic lung disease. The symptoms of hypersensitivity pneumonitis include fever, cough, dyspnea, and chills. There would be an elevated white blood cell count, which is not the case here. Doubly refractile particles with polarized light can be found with silicosis, not with hypersensitivity pneumonitis. Pleural mesothelioma is a malignancy associated with asbestos exposure. There is no increased risk of mesothelioma with silicosis. X-rays would show pleural changes or pleural effusion.

Case Ico-delete Highlights A 52-year-old man has a past medical history of smoking 40 packs of cigarettes per year; he presents for a follow-up. He notes that, over the past week, he has developed increased nonproductive cough and shortness of breath. He has had a chronically-progressive cough and shortness of breath upon exertion over the past year. His occupational history reveals coal mining for 25 years. He denies any alcohol consumption, travel history, or sick contacts. He also denies fever, chills, sore throat, otalgia, chest or abdominal pain, peripheral edema, rashes, and pruritus. A chest radiograph is performed, which reveals the following imaging. Question What is true regarding health maintenance recommendations in this patient?

Correct answer: Smoking cessation strategies should be strongly encouraged Explanation This patient's most likely diagnosis is silicosis. Patients should be strongly advised to quit smoking; they should be assisted in their smoking cessation efforts. Furthermore, patients should be immunized against influenza and pneumococcal pneumonia. Latent tuberculosis infection (i.e., positive tuberculin skin test result without active disease) should be treated with isoniazid. A 10-mm induration is considered a positive test result in this population. Patients with disease should avoid any additional exposure to silica dust. No dietary or activity restrictions are necessary in cases of silicosis.

Case A 69-year-old man with a past medical history of obstructive sleep apnea, hypertension, and COPD presents to his local medical office with dyspnea on exertion that has been slowly progressive over the course of the last year. He notes impairment in climbing stairs and walking short distances. His review of systems is positive for fatigue, palpitations, intermittent retrosternal chest pain, swelling of his lower extremities, dizziness, and "feeling faint." His associated symptoms are also known to occur upon exertion. His cardiac exam is remarkable for an oxygen saturation of 90%, accessory muscle usage, an increased pulmonic component of the second heart sound (P2), wide, inspiratory splitting of S2 over the cardiac apex, right-sided S3 and S4 gallops, a left parasternal lift, a loud diastolic murmur that increases with inspiration and diminishes with the Valsalva maneuver, prominent "A" waves in jugular venous pulsations, and increased JVD. He has an enlarged liver with hepatojugular reflux, peripheral edema, and ascites. A bedside EKG and chest x-ray were performed, which revealed the following image. Question What is expected to be the most beneficial intervention for this patient?

Correct answer: Spironolactone Explanation This patient has a presentation most consistent with cor pulmonale secondary to chronic obstructive pulmonary disease (COPD). The electrocardiogram demonstrates right axis deviation, right ventricular hypertrophy, and right atrial enlargement. The chest x-ray indicates cardiac enlargement, with prominence of the pulmonary artery, right atrium, and right ventricle. Treatment should center upon treatment of the underlying contributory disorder. For patients with COPD, bronchodilation and infection treatment should be considered. Oxygen therapy is of great importance in patients with underlying COPD,especially when administered on a continuous basis. Oxygen therapy relieves hypoxemic pulmonary vasoconstriction, which then improves cardiac output, lessens sympathetic vasoconstriction, alleviates tissue hypoxemia, and improves renal perfusion. Prostaglandin I2 (prostacyclin) is the main product of arachidonic acid in the vascular endothelium, has relatively specific vasodilatory effects on the pulmonary microvasculature along with antiplatelet effects, and, over time, may also promote vascular remodeling. Prostacyclin has been the cornerstone in the medical treatment of IPAH but is ineffective in the larger group of patients with severe COPD or interstitial pulmonary fibrosis or when hypoxemia is the cause for PAH (pulmonary artery hypertension). Current guidelines suggest low-level graded aerobic exercise, being careful to avoid heavy physical exertion or isometric exercises because they may cause syncope. PAH and cor pulmonale patients should follow a sodium-restricted diet of less than 2400 mg per day. Immunosuppressive therapy is often administered to patients with connective tissue diseases and those with interstitial lung disorders, although only a minority of patients in the latter category respond. Although there are reports of improvements in pulmonary function and oxygenation in patients who are receiving interferon-γ1b injections, recently published trials failed to show any improved survival with this therapy. To manage right ventricular volume overload, patients often respond well to a combination of a loop diuretic, such as furosemide, and the potassium-sparing diuretic spironolactone

Case Ico-delete Highlights A middle-aged man is an immigrant to the United States; he presents with a chronic cough, blood-stained sputum, and night sweats. Physical examination reveals a cachectic man with bronchial breathing in the right upper lobe. His sputum examination under microscope shows acid-fast staining rods, and his PPD is 15 mm. Question What is the best therapeutic approach?

Correct answer: Start isoniazid+rifampicin+pyrazinamide+ethambutol immediately and reassess the patient's treatment after the culture result. Explanation The choice for tuberculosis treatment is always multiple drug therapy to prevent resistance during the long treatment (6-9 months). Isoniazid (INH), rifampicin, pyrazinamide, and ethambutol is the most used combination to start in newly diagnosed TB patients. The patient described in this question has all the symptoms of pulmonary TB. His PPD skin test is 15 mm, and his sputum is also positive for acid-fast bacilli. PPD is used as an antigen in the tuberculin test, which is a delayed hypersensitivity reaction. The standard tuberculin test is performed by injecting 0.1 mL of diluents containing 5 TU (tuberculin unit) of PPD. It is injected intradermally, and induration is checked in 48-72 hours. 0-5 mm is accepted as negative; 5-10 mm can be due to a previous BCG vaccination. If a patient has not had the vaccine and was exposed to TB, however, 5-10 mm indicates they are probably infected. In AIDS patients, a 5 mm reaction is considered positive. Even if they had the BCG vaccine previously, patients with 15 mm or more induration are assumed to be infected with M. tuberculosis. This patient's PPD is 15 mm, and his sputum is positive for AFBs, so the proper approach should be to send a culture to identify the type of Mycobacterium, but treatment should not be delayed waiting for culture results. Multiple drug therapy should be started and the patient's treatment should be assessed after receiving the culture result. If it is not a resistant strain, pyrazinamide and ethambutol can be stopped after 2 months. The patient should continue with INH and rifampin and complete 6 months of treatment.

Question A 66-year-old man with moderately well-controlled type II diabetes mellitus presents for the management of community-acquired pneumonia. He is currently being treated with intravenous levofloxacin. He is afebrile; pulse is 93, blood pressure is 130/90 mm Hg, respirations are 18/min, and oxygen saturation is 92% on room air. What is the most common cause of bacterial pneumonia among non-institutionalized individuals with no significant co-morbid conditions?

Correct answer: Streptococcus pneumoniae Explanation The correct response is Streptococcus pneumoniae. Community-acquired pneumonia (CAP) is a major cause of morbidity and mortality worldwide, particularly among patients age 65 and older. The pathogens associated with bacterial pneumonia may be divided into the typical and atypical species. Approximately 85% of the cases are caused by 'typical bacteria', most frequently Streptococcus pneumoniae, Haemophilus influenzae, and Moraxella catarrhalis. Among these, infection with Streptococcus pneumoniae remains the most common in the United States. The remaining 15% of CAP cases are caused by atypical bacteria (e.g., Legionella species, Chlamydia pneumoniae, and Mycoplasma pneumoniae). Hematogenous spread of Staphylococcus aureus, causing the development of pneumonia, is more often recognized in specific high-risk patients (e.g., intravenous drug abusers and patients with implanted prosthetic devices). Staphylococcus aureus is also relatively more common among patients suffering from post-viral bacterial pneumonia. Except in patients with cystic fibrosis or other causes of chronic bronchiectasis, Pseudomonas aeruginosa is an uncommon cause of CAP.

Case A 6-week-old male infant presents with a 4-day history of cough and nasal congestion. According to his mother, he occasionally has a bluish tint around his lips while sleeping. There is no history of fever. His older siblings have an upper respiratory infection. The patient's appetite has been decreased somewhat, mostly due to the copious nasal secretions; however, he has been maintaining a normal urine output. He was delivered pre-term at approximately 34 weeks gestation. He had mild respiratory distress syndrome, spending 2 days on a ventilator in the neonatal intensive care unit (NICU). He went home in 10 days and has done well since. He has had no immunizations. A physical exam reveals an infant in mild respiratory distress, respirations of 52/min, with slight intercostal retractions. Temperature is 100.2 F with a HR of 130/min. Perioral duskiness is seen. Oxygen saturation at room air is 83%, and HEENT exam is otherwise normal. His chest exam shows coarse rhonchi and expiratory wheezes. Heart rate and rhythm are regular. No murmurs appreciated. Abdomen is soft and non-tender. Neurological is intact. Chest X-ray shows mild hyper expansion, but no consolidation. Nasal swab for respiratory syncytial virus (RSV) is positive. Question What is the most appropriate treatment?

Correct answer: Supplemental oxygen and supportive therapy Explanation Respiratory syncytial virus (RSV) is the most important etiology of viral lower respiratory disease in infants and children; bronchiolitis is the most common manifestation. Infection is initiated in the upper respiratory tract and may spread to the lower tract; this causes obstruction of smaller airways by edema, necrotic tissue, and inflammatory cells. This leads to the typical findings of nasal discharge, cough, retractions, and wheezing or rhonchi. The peak incidence is during the first 2 years of life. In the United States, it occurs during the winter months in an epidemic fashion. RSV is a member of the Paramyxovirus family. Complications include apnea in the very young and premature infant, pneumonia, croup, respiratory failure, otitis media, and dehydration. The severity of bronchiolitis is diagnosed and assessed based on history and physical examination. Infants with mild bronchiolitis can be treated symptomatically at home. The following are the criteria for hospitalization in RSV bronchiolitis: Age less than 12 weeks Gestational age at birth of less than 34 weeks Cardiopulmonary disease or immunodeficiencies Wheezing and respiratory distress associated with oxygen saturation below 92 percent on room air (hypoxemia) History of significant apnea before assessment. The management in moderate-to-severe bronchiolitis involves supplemental oxygen therapy and supportive measures such as prevention of dehydration and respiratory support. Supplemental oxygen is the single most useful therapy, usually delivered via nasal prongs when oxygen saturation (SpO2) falls persistently below 90% in previously healthy infants. Mechanical ventilation may be needed for respiratory failure or severe apnea. Ribavirin, an antiviral agent, has not been shown to reduce hospital stays or mortality. The agent is very expensive and may be teratogenic in humans. It may still have a place for treatment of infants at significant risk for complications (i.e., those with cardiopulmonary disease). Bronchodilators, such as albuterol, have not been shown to be effective; however, their empirical use in the setting of the hospitalized patient is still commonly seen. Likewise, corticosteroids are not effective, and their use is contraindicated. Antibiotics, such as ampicillin and cefotaxime, are not routinely used in the majority of patients, but they are indicated in cases of clinical evidence of superimposed bacterial pneumonia or acquired nosocomial infection during the hospitalization.

Case A 2-week-old neonate has been reported to have an elevated Immunoreactive Trypsinogen (IRT) level on her newborn screening sample that was sent from the newborn nursery after birth. The state newborn screening lab also ran additional tests to screen for the 40 most common mutations that are known to cause Cystic Fibrosis (CF). The result of that mutation screening was negative. You explain that this means a greater than 98% chance that the newborn does not have CF. Question What should be the next step in the evaluation of this positive newborn screening result for Cystic Fibrosis if the parents ask for definitive testing?

Correct answer: Sweat chloride test Explanation The patient in this vignette is being evaluated for Cystic Fibrosis. Since there are more than 1,900 mutations that are known to cause CF, the absence of 40 of the most common mutations does not completely rule out CF as a possibility. Further confirmation requires a sweat chloride test, which measures the sweat chloride concentration by pilocarpine iontophoresis. An elevated value of >40mmol/L in infants less than 6 months and >60mmol/L in those infants greater than 6 months is considered diagnostic of CF. Sweat conductivity testing is considered unreliable and is not useful for establishing the diagnosis. Nasal epithelial potential difference measurement is a research tool to assess the basic defect in CF at the level of the nasal epithelium and is not used in clinical practice. Throat swab may help screen for any pathogens that may be colonizing the airway in CF patients, but this usually does not help with confirming the diagnosis of CF. Similarly, fecal elastase level can be measured to diagnose pancreatic insufficiency once the diagnosis of Cystic Fibrosis is established. Normal levels of fecal elastase do not rule out Cystic Fibrosis, as many patients can be pancreatic sufficient at birth.

Case Ico-delete Highlights A 54-year-old Caucasian man presents with sudden severe shortness of breath. The patient is well known to you due to a 5-year history of severe emphysema. He states that he has been a basketball player all his life; he was practicing about 1 hour prior to presentation, at which point he experienced sudden chest pain and immediate shortness of breath. It is still bothering him. He describes the chest pain in the middle of the chest, more so on the right anterior side. The patient admits to smoking 2.5 packs of cigarettes daily. The patient has had at least 5 episodes similar to this one since he was 19 years old. Physical examination reveals a tall, thin, well-developed man in moderate distress. Other abnormalities include mild tachycardia (120 beats per minute) and diminished breath sounds in the posterior right lower lobe. There is noted wheezing and crackles heard throughout the rest of the lung fields in both the inspiratory and expiratory phases of breathing. Question Based upon the most likely diagnosis at this time, what pharmaceutical option may be used to help decrease this occurrence in the future?

Correct answer: Talc sclerotherapy Explanation The correct response is Talc sclerotherapy. The clinical scenario is highly suggestive of a (recurrent) secondary pneumothorax. Components that lead to this diagnosis include unilateral, sudden chest pain with dyspnea and minimal physical exam findings, in this case, the mild tachycardia and diminished breath sounds. The above scenario most likely fits what is considered a secondary pneumothorax, which usually occurs as a complication of various pathologies, including COPD, asthma, cystic fibrosis, tuberculosis, Pneumocystis pneumonia, and various interstitial lung diseases. Diagnosis is usually confirmed with chest radiograph. Treatment is typically observation and potential surgical intervention after about 5 days if the pneumothorax fails to resolve itself. However, the ability of the patient to withstand a surgical intervention also must be considered. There are various thought processes in terms of management of recurrent pneumothorax especially. The correct option out of the above is the talc sclerotherapy; this is commonly reserved for patients who may not necessarily be surgical candidates. This is also known as pleurodesis and is performed to obliterate the pleural spaced to prevent recurrent pleural effusions or persistent pneumothorax. Talc is an effective sclerosant is instilled into the pieural space to cause inflammation and fibrosis. Intravenous antibiotics would be correct if the pneumothorax were actually caused by the sequelae of staphylococcal pneumonia. Oral antibiotics, short-acting β2-agonist, and inhaled corticosteroids are inappropriate in the management of a secondary pneumothorax

Question A 40-year-old man presents with a 3-month history of a productive cough; the cough produces a scant amount of yellow sputum. He has also had an evening temperature rise (101 F) for the past 3 months. On occasion, the sputum was blood-streaked. He has lost a significant amount of weight during this period. Patient is a non-smoker. He immigrated from India 5 years ago, and his medical records are not available. Coarse upper lobe crackles and rhonchi are heard bilaterally. X-ray shows multiple bilateral upper lobe cavities with surrounding infiltrate. What is the probable diagnosis?

Correct answer: Tuberculosis Explanation The classical clinical presentation is of tuberculosis. Chest X-ray confirms the diagnosis. As tuberculosis is a common disease in developing countries, the patient being an immigrant from a developing country strengthens the diagnosis. Lobar pneumonia has an acute onset of 3-4 days. It would not be missed on X-ray because of the consolidation present in a particular affected lobe. With Asthma, the patient would have a history of acute onset of symptoms of breathlessness. A chest X-ray will not show significant changes in asthma. The presence of bilateral wheeze and rhonchi all over the chest wall with the ABG are important in the diagnosis of asthma. Emphysema has symptoms of weight loss, but it will usually have a dry cough and X-ray will show bullae, hyper-translucent lung fields with peripheral vascular markings. Bronchitis will have symptoms of breathlessness, but usually there is no weight loss and the Chest X-ray does not show any significant changes. Sometimes, there might be changes due to concomitant emphysema.

Case Ico-delete Highlights A 4-year-old boy presents with a 3-day history of fever with chills, cough, and fast breathing. His parents report decreased oral intake and increased difficulty breathing with retractions over the last 24 hours. His initial vital signs are: heart rate 144 bpm; respiratory rate 32/min; temperature 101.3 degrees Fahrenheit (38.5 degree Celsius); and oxyhemoglobin saturation 89% on room air. He is immediately started on supplemental oxygen, and his oxyhemoglobin saturation improves to 95%. Subsequent evaluation, including a chest X-ray, is suggestive of right middle and lower lobe pneumonia. Question What is the mechanism for the low oxyhemoglobin saturation in this patient?

Correct answer: Ventilation/Perfusion mismatch Explanation Of all the causes of hypoxemia in children, ventilation perfusion mismatch is the most common cause in most clinical situations. In a child with pneumonia, the middle and lower lobes of the right lung will continue to receive pulmonary blood flow, but the exchange of gases will be impaired due to poor air entry in those parts of the lung. This generates a mismatch between the ventilation (which is reduced) and perfusion (which is maintained), causing most of the blood from those areas to return to the left atrium in a relatively deoxygenated state. This leads to an overall drop in the percentage of hemoglobin molecules that are saturated with oxygen, thereby manifesting as a drop in the measured oxyhemoglobin saturation. Hypoventilation occurs in patients with neuromuscular weakness and can cause significant elevation of carbon dioxide levels before it causes hypoxemia. Diffusion block is associated with lung disorders, such as idiopathic pulmonary fibrosis and interstitial lung diseases, which increase the thickness of the alveolar basement membrane, thereby reducing the exchange of gases. Shunt causes hypoxemia due to the mixing of deoxygenated blood with oxygenated blood when the former has bypassed the lung completely (such as right to left shunt in a patient with Tetralogy of Fallot). A patient with pneumonia will not have an increase of dead space in the lungs.

Case A 40-year-old woman undergoing chemotherapy develops fevers and respiratory symptoms. A chest radiograph is ordered, and it reveals bilateral fluffy pulmonary infiltrates. A bronchoscopy with biopsy is performed, and the specimen is found to contain septate hyphae with acute-single branching. Question What treatment should the patient receive?

Correct answer: Voriconazole Explanation The patient's symptoms are most indicative of a pneumonia-like infection, most likely stemming from the fungus Aspergillus fumigatus. Voriconazole is considered a first-line treatment for invasive aspergillosis. It is available in both oral and intravenous forms and has good bioavailability. It is hepatically cleared and has less toxicity compared to amphotericin B, which is also used to treat aspergillosis. Vancomycin was first isolated from Streptomyces orientalis. It was introduced in 1958 for the growing problem of penicillin-resistant staphylococci. By 1960, it was superseded by Methicillin. In the last 10 years, the use of vancomycin has significantly increased due to the emergence of Methicillin-resistant Staphylococcus aureus (MRSA), Clostridium difficile, and enterococcal disease. The antibiotic complexes to the D-alanyl-D-alanine portion of the growing cell wall and inhibits peptidoglycan and transpeptidation. Vancomycin is poorly absorbed orally and thus is given by IV. It is widely distributed throughout the body. Adverse reactions are not common; however, ototoxicity is a serious concern and blood levels must be monitored. Erythromycin is a macrolide and was originally isolated from a strain of Streptomyces erythreus in 1952 by McGuire, et. al. Erythromycin interferes with protein synthesis. Oral erythromycin base is destroyed by the acid in the stomach; as a result, it is manufactured with an acid-resistant coating that is dissolved in the duodenum, which is where it is liberated in its active form. The antibiotic is distributed widely in tissue and is concentrated in the liver and spleen. Levels of the antibiotic persist in tissue longer than in serum, and the antibiotic does not enter the CSF in the absence of inflammation. Toxicity is low. Erythromycin was formerly the drug of choice for the treatment of Legionella; but now azithromycin is preferred. Tetracycline compounds were first described in the 1940s. Tetracyclines inhibit protein synthesis. They are usually used orally and are absorbed by the stomach. Dairy products and aluminium or magnesium salts such as antacids will negatively affect absorption by the stomach; therefore, they must be avoided. The antibiotic distributes widely in the body, with the exception being the central nervous system. Gastrointestinal disturbances can be a side effect of the antibiotic. However, a far more serious side effect occurs due to the ability of the antibiotic to bind to divalent cations such as magnesium and calcium. Because of this, they are rarely used in children. In children, they bind to developing teeth and bone, resulting in gray or brown teeth or affect long bone development. Tetracyclines are used for some "unusual" infections such as Mycoplasma, Ureaplasma, nocardiosis, plague, Rocky Mountain Spotted Fever, Scrub typhus, and Q fever. Penicillin was first isolated from the fungus Penicillin notatum. Penicillin is the antibiotic that is considered to have ushered in the modern era of antimicrobials. The basic penicillin ring consists of a thiazolidine ring and a B-lactam ring, which is a 4-member ring with amide linkage joining carbonyl and nitrogen. The mechanism of action of penicillins is through penicillin-binding protein (PBP) binding with subsequent inhibition of transpeptidation reaction and activation of the autolytic enzymes. Administration is orally and IM or IV. Food interferes with absorption, so it should be given 1 hour before or 2-3 hours after meals. Penicillin is widely distributed in the body, but there is poor penetration in some spaces, such as the eye, pericardium, and cerebral spinal fluid. The antibiotic is considered non-toxic, but it can cause allergic reactions in 3-10% of the patient population. Penicillin is most active against Streptococcus spp., Neisseria gonorrhoeae, Treponema pallidum, and Listeria spp. It is used clinically to treat syphilis, Streptococcus pyogenes pharyngitis, and as a prophylactic agent after rheumatic heart disease.

Case A 38-year-old man presents with pneumonia. He is a construction worker who is otherwise healthy and has no allergies. His temperature is elevated, he has a nonproductive cough, and he states he "feels bad" and lacks energy. Question What is the best course of action?

Correct answer: You obtain a specimen for a Gram-stain and sputum cultures and prescribe erythromycin 500mg PO q6h for 14 to 21 days for an empiric therapy against Mycoplasma pneumoniae. Explanation The patient presents with symptoms of community-acquired pneumonia, not classic symptoms of a S. pneumonia infection. He is most likely immunocompetent, and your patient does not show the classical symptoms of being critically ill. Hospital admission is unnecessary. In such a case, the best course of action is to obtain a Gram-stain and sputum cultures before any treatment is initiated. Then, an empiric treatment is started immediately with erythromycin; treatment should be focused on the most likely non-viral pathogen, which is either S. pneumoniae, S. aureus, or Mycoplasma pneumoniae. Erythromycin is effective against both pathogens, as well as other atypical organisms such as Chlamydia and Legionella. Have the patient drink plenty of fluids for adequate hydration, and supply oxygen if needed. This treatment should improve the patient's condition within a few days. If not, you will have the results of the sputum cultures available to adjust the antibiotic therapy accordingly. Fluoroquinolones are used if there are underlying comorbidities such as chronic lung disease. Doxycycline is another first-line choice in uncomplicated CAP. Starting an empiric therapy immediately is wrong because cultures have to be collected before antibiotic treatment is initiated, or they may be useless. Vancomycin as the antibiotic of choice is wrong because an empiric therapy should be focused on the most likely pathogen. Erythromycin should be prescribed for Mycoplasma pneumoniae, and penicillin G or erythromycin (for penicillin-allergic patients) should be prescribed for the treatment of S. pneumoniae.


Kaugnay na mga set ng pag-aaral

AWS Certified Developer Associate 3

View Set

Real Estate Course: Exam Prep Master

View Set

ATI PEDS Ch. 6 Growth and Development of the School-age Child

View Set

Unit 7 - AP Environmental Science

View Set

Ch. 14 Contract Law and Performance

View Set

General Review Spanish Test 2 - set 28

View Set